Exercicios Informatica CESPE 2010 250 Questoes

45
Informática Geral CESPE Questões de Concursos de 2010 (250 questões) Prof. Belfort [email protected] Brasília/DF 01/12/2010

Transcript of Exercicios Informatica CESPE 2010 250 Questoes

Page 1: Exercicios Informatica CESPE 2010 250 Questoes

Informática Geral

CESPE Questões de Concursos de 2010

(250 questões)

Prof. Belfort [email protected]

Brasília/DF 01/12/2010

Page 2: Exercicios Informatica CESPE 2010 250 Questoes

�������������� ����������������������� ���� ����� !"#$%&'%$%&()&!*+,)&-+&..&%&./0123456717839717:6;<;=>715971:?43=7178;<<7@715>3A3B7=@CD<;1C1E=>;6=;>1FG:AC6;61H1IEFHJKLC1M5;18C=8;6=;1719C@C<1@;15>3A3B7NOC1@;1>;8=CAC437<P12;6679;=>7<P7:A387>3QC<1;1:6C8;@39;=>C<17<<C837@C<171E=>;6=;>1;1RSTUVSWTP1;8C=<3@;67=@C17123456717839717:6;<;=>7@7P1X5A45;1C<13>;=<1<;453=>;<KYY 0C1<;18A38761C1ZC>OC1 P1C<1[WW\]1@71:?43=71<;6OC1;G3Z3@C<KWW\_15918C=>;a@C126;M5;=>;9;=>;17>57A3B7@C1;1:5ZA387@C:C61591<b>3CKYc 0C1<;18A3876171C:NOC1 P1<;6OC17:6;<;=>7@7<1@3Q;6<7<C:Nd;<P1;=>6;1;A7<171M5;1:;693>;1;=Q376171:?43=71C51<;51eRSf1:C68C66;3C1;A;>6g=38CKYh i1>;G>C1@71:?43=71:C@;1<;61;@3>7@C1;1:5ZA387@C15>3A3B7=@CD<;71C:NOC1jklml1=C1n386C<C2>1oC6@1;1<7AQ7=@CDC18C9C1pqnrK016;<:;3>C1@;18C=8;3>C<1;19C@C<1@;15>3A3B7NOC1@;17:A387>3QC<1;1@;<3<>;97<1C:;6783C=73<P1X5A45;1C<13>;=<1@;1.s&71./P171:76>361@712345677:6;<;=>7@7KYt u7<C1<;1@;<;X;18C:3761591>;G>C1@71:?43=71Q3<57A3B7@718C91CEFH1:767159176M53QC1;@3>7@C18C91C1o63>;61@C1v6i2238;P1@;Q;D<;1<;A;83C=761C1>;G>C1;17A>;6761712C=>;1:7671591>3:C1@3<:C=bQ;A=C1v6i2238;P15971Q;B1M5;1;G3<>;13=8C9:7>3Z3A3@7@;1@;12C=>;;=>6;1C<17:A387>3QC<1@C1o3=@Cw<1;1@C1v6i2238;K Yx y76717Z636P1;9179Z3;=>;1o3=@Cw<P15971:A7=3Az718637@71;979Z3;=>;1r3=5GP11=;8;<<?6371713=<>7A7NOC1@;1591{e|}RS7:6C:637@C1@;18C=Q;6<OC1=C18C9:5>7@C61M5;15<71C1o3=@Cw<KY~ �;P17C1<;18A387618C91C1ZC>OC1@36;3>C1@C1��|]W1=713974;91M5;;<>?1=71:?43=71Q3<57A3B7@718C91C1EFHP1���m�l12C615971@7<C:Nd;<17:6;<;=>7@7<P1;=>OCP17C1<;18A38761;<<71C:NOCP1713974;9<;6?18C:37@71:767171?6;71@;1>67Z7AzC1;1:C@;6?1<;613=<;63@71;9591>;G>C1;@3>7@C18C91C1o63>;61@C1v6i2238;K08;6871@;18C=8;3>C<1@;1<3<>;97<1@;13=2C697Nd;<1;1@;1<;4567=N71@73=2C697NOCP1X5A45;1C<13>;=<1<5Z<;M5;=>;<K1Y� �V�f|{P1591:6C8;@39;=>C1M5;1@;Q;1<;616;7A3B7@C19;=<7A1C57=57A9;=>;P1>;917123=7A3@7@;1@;14767=>36171:6C>;NOC1@;1@7@C<P3=8A5<3Q;18C=>6717>7M5;<1@;1��U�]Kc� 013=2C697NOC115917>3QC1M5;P18C9C1M57AM5;61C5>6C17>3QC39:C6>7=>;1:7671C<1=;4�83C<P1>;91Q7AC61:767171C647=3B7NOC1;P:C613<<CP1@;Q;1<;617@;M57@79;=>;1:6C>;43@7K1

Agnaldo
Nota
C
Agnaldo
Nota
C
Agnaldo
Nota
E
Agnaldo
Nota
E
Agnaldo
Nota
E
Agnaldo
Nota
E
Agnaldo
Nota
E
Agnaldo
Nota
C
Page 3: Exercicios Informatica CESPE 2010 250 Questoes

��������������� ���������������������� ������� !"#$%&'(!%()! (*()!*#!+,-$-.%&'(!*#!,#/0($(1-%)2!3#""% #0,%)2%4$-/%,-5()!#!4"(/#*- #0,()!%))(/-%*()!6!70,#"0#,2!8+$1+#!()!-,#0)!%)#1+-"2!%!4%",-"!*%!3-1+"%!%/- %2!9+#! (),"%!+ %!4:1-0%!%/#))%*%+,-$-.%0*(;)#!(!70,#"0#,!�<4$("#"!=!>7�=?@!AA B( !%!%,-5%&'(!*%!(4&'(!CDEFGDHIJKLMNOPEDQFK0(!RSTU2!-03(" %&V#)!*%!)#))'(!*#!0%5#1%&'(2!,%-)!/( (WXXYZS[2!%"9+-5()!*#!70,#"0#,!,# 4(":"-()!#!\-),]"-/(2!)'(#</$+*%)!9+%0*(!(!7�!_!3#/\%*(@A a!) b($(! !%4"#)#0,%*(!0%!4:1-0%!-$+),"%*%!0%!3-1+"%-0*-/%!#""(!0%!0%5#1%&'(!*#/(""#0,#!*#!%!4:1-0%!,#"!)-*(!/"-%*%4%"%!0%5#1%&'(!# !)-),# %!c-0+<!#!#),%"!)#0*(!%/#))%*%!# /( 4+,%*("!/+8(!)-),# %!(4#"%/-(0%$!_!(!d-0*(e)@Af g,-$-.%0*(;)#!(!/% 4(!hijOFkkIJKlFKmnkoDK#<-b-*(!0#))%4:1-0%2!_!4())5#$!$(/%$-.%"2!0(!),-(!*%!p*5(/%/-%;q#"%$!*%g0-'(!>pqg?!(+!# !(+,"()!),-()!*%!70,#"0#,2!-03(" %&V#)!9+#/(0,#0\% !%!#<4"#))'(!*-1-,%*%!0#))#!/% 4(@p/#"/%!*#!/(0/#-,()!#! (*()!*#!+,-$-.%&'(!*#!%4$-/%,-5()!#!*#)-),# %)!(4#"%/-(0%-)2!8+$1+#!()!-,#0)!*#!rs!%!rt@Au a!,% %0\(!*%!3(0,#!# !+ %!4:1-0%!5-)+%$-.%*%!/( !(!70,#"0#,�<4$("#"2!# !+ !/( 4+,%*("!/+8(!)-),# %!(4#"%/-(0%$!_!(d-0*(e)!vw2!4(*#!)#"!%+ #0,%*%!4("! #-(!*%!(4&'(!xJMQFk0(!4%-0#$!*#!/(0,"($#@ Ay g %!4:1-0%!9+#!#),-5#"!)#0*(!5-)+%$-.%*%!/( !(!70,#"0#,�<4$("#"!4(*#!)#"!)%$5%!/( (!+ !%"9+-5(!*(!d"-,#"!*(zO{||PoF2!/%)(!#),#!)#8%!(!#*-,("!*#!,#<,(!-0),%$%*(!0(/( 4+,%*("@A} p(!)#!/$-/%"!%!,#/$%!~!*(!,#/$%*(!*(!/( 4+,%*("2!+ %"_4$-/%!*%!- %1# !0%!,#$%!*(! (0-,("!)#":!/(4-%*%!4%"%!%!:"#%*#!,"%b%$\(!#!4(*#":!)#"!-0)#"-*%!# !+ !,#<,(!9+#!#),#8%!)#0*(#*-,%*(!/( !(!d"-,#"!*(!�"a33-/#@�(!9+#!)#!"#3#"#!%!/(0/#-,()!*#!)-),# %)!*#!-03(" %&V#)!#!*#)#1+"%0&%!*%!-03(" %&'(2!8+$1+#!()!-,#0)!)+b)#9+#0,#)@A� g !%"9+-5(!/"-4,(1"%3%*(!3-/%!4"(,#1-*(!/(0,"%!/(0,% -0%&'(4("!5"+)@� p!"#%$-.%&'(!*#!+ !��WYU�2!(+!/]4-%!*#!)#1+"%0&%2!/(0)-),## !/(4-%"!*%*()!*#!+ !*-)4()-,-5(!*#!%" %.#0% #0,(!4%"%(+,"(2!*#! (*(!9+#!#))#)!*%*()!4())% !)#"!"#),%+"%*()!# /%)(!*%!4#"*%!*()!*%*()!("-1-0%-)!4"(5(/%*%2!4("!#<# 4$(2!4("%4%1% #0,(!%/-*#0,%$!(+!/(""+4&'(!*#!*%*()@2

Agnaldo
Nota
C
Agnaldo
Nota
E
Agnaldo
Nota
E
Agnaldo
Nota
E
Agnaldo
Nota
E
Agnaldo
Nota
E
Agnaldo
Nota
E
Agnaldo
Nota
C
Page 4: Exercicios Informatica CESPE 2010 250 Questoes

������������������ ������������������������������ � ��������!�!� "�#�"$%&'()*+,-./01234./564/.4/.47648/91:4.4;<98/<:4=>0./9?91<9?0./?0/4?@<0:@9A?0/BCDEFGHICHJDFKLG/?4/MNOPOQRMRS/-..@;9A4/9/012T0/48/564/0..@;9@./?4/10;<692T0/.T0/481:479?0./=0::4<984;<4SU -/V09/;0<W=@9/19:9/0./;0X0./V4;4Y@=@Z:@0./?0/1:07:4..0/?0/19W.[/564/0/.9AZ:@0/8[?@0/?4/5648/4.<Z/4;<:9;?0/;0/84:=9?0910;<9/<4;?\;=@9/?4/9A<9]/=08/0/=>989?0/9684;<0/:49A/_Z?4.=0;<9?0/0/abcdSe a0/fA<@80/<:@84.<:4]/>06X4/9684;<0/?4/P]ghi/48/:4A92T0/9084.80/14:W0?0/?4/QRRN]/=08/0/X9A0:/8[?@0/.9@;?0/?4jk/hlQ]mg/19:9/jk/lMn]hRS/oZ/1:[email protected]./?4/564/0/19W.]/9<@;_968/=:4.=@84;<0/9;69A/;9/=9.9/?0./Q/8@A>34./?4/10.<0./;0/9;0?4/QRMRSp q9@./74;<4/481:479?9]/1:0?6r@;?0/.@7;@Y@=9/14:.14=<@X9/?474:92T0/?4/:@564r9]/=:4.=@84;<0/;0/1:0?6<0/@;<4:;0/V:6<0/[email protected];<4/0=619?9/4/A0;74/?0/?4.4.14:0/4/?9./<4;<9234./?0/=:@84]68/10X0/=08/9/96<04.<@89/48/9A<9Ss aT0/Y9A<98/4.<6?0./564/80.<:98]/68/84A>0:/?4.4814;>04.=0A9:/?9./=:@9;29./569;?0/4A9./;T0/<\8/?4/X4:/0/19@/48/=9.9]?4/V:920./=:6r9?0.]/9V9<@?0]/10:564/;T0/<48/689/Y0;<4/?4:4;?9/19:9/.6.<4;<9:/.69/Y98WA@9St aT0/[/1:[email protected]/[email protected]:/86@<0/19:9/14:=4V4:/564]/=08/[email protected];<4/<:9V9A>9;?0]/0./79.<0./=08/9..@.<\;=@9/.0=@9A/4/84.80=08/:48[?@0./=0;<:9/?04;29./?4/Y6;?0/;4:X0.0]/.T0.@7;@Y@=9<@X984;<4/:4?6r@?0./569;?0/9./14..09./4.<T0/[email protected]=619?9./=08/<:9V9A>0S$%&'()*+uv-./01234./564/.4/.47648/91:4.4;<98/1:010.<9./?4/<:4=>0./?419:4=4:S/-..@;9A4/9/012T0/=6_0/<4w<0/=0::4.10;?4/90/564/1:4=4@<6989./;0:89./?4/:4?92T0/0Y@=@9ASU a0..0./4.<6?0./<[=;@=0./?480;.<:98/564/9/=:x;@=9/?0/_070/;0y:9.@A/[/:41A4<9/?4/4w481A0./564/?4.9=0;.4A>98/9/A479A@r92T0]=080/9/X@0A\;=@9/?9./79;764./564/=0;<:0A98/4A4]/A9X9748/?4?@;>4@:0/4/=001<92T0/?4/96<0:@?9?4./19:9/Y9r4:48/X@.<9/7:0..9?@9;<4/?9./@A479A@?9?4.Se -=:4?@<980./564/0/10?4:/?0/?@;>4@:0/.6_0/4/;0_4;<0/?0/_070;T0/<48/A@8@<4.S/b0:/.69/X4r]/9./@;.<@<6@234.]/.46./z:7T0./4Y6;=@0;Z:@0./;T0/.T0/@814:84ZX4@./{/=0::612T0/564/=0;<98@;90/.@.<489/9?8@;@.<:9<@X0S/..0/[/689/14;9Sp |V.4:X9}.4/564/?4.?4/0./9;0./NR]/569;?0/0./=929};W564@./4/0.V@;70./@;X9?@:98/9./=@?9?4.]/;T0/Y9A<98/[email protected]?@0./19:9/80.<:9:9/X6A;4:9V@A@?9?4/?0./974;<4./?0/10?4:/1fVA@=0/90/=9;<0/?9.4:4@9/564/4=09/?0./=0Y:4./4810:=9A>9?0./?9/_079<@;9Ss ~/@;=0;<ZX4A/0/;f84:0/?4/10A@=@9@./=9;9A>9.]/<:919=4@:0./4X979V6;?0./@;=A6.@X4/?4/9A<0./4.=9A34.d/48/<0?0/0/19W.]A@79?0./{/=0;<:9X4;2T0/{/V9;?@?9748St |./4;X0AX@?0./;0/_070/;T0/>4.@<98/48/914A9:/19:9/9/X@0A\;=@94/9/4A@8@;92T0/YW.@=9S/-A[8/[email protected]]/0/.4=:4<Z:@0/;9=@0;9A9;<@?:079./?9/b:4.@?\;=@9/?9/j41fVA@=9/@?4;<@Y@=9/;0.456@1984;<0./4A4<:x;@=0./?4/_070./?4/9r9:/689/Y0:89/?4A479A@r92T0/?0/?@;>4@:0/?0/;9:=0<:ZY@=0/@;<4:;9=@0;9AS $%&'()*+uuc08/:4A92T0/90/.@.<489/014:9=@0;9A/�@;6w]/9..@;9A4/9/012T0=0::4<9SU �8/689/?@.<:@V6@2T0/?0/.@.<489/�@;6w]/[/10..WX4A/4;=0;<:9:��������/?4.<@;9?0./9/?@X4:.9./Y@;9A@?9?4.]/=080/19:9/1:0X4:.4:X@20/?4/9=4..0/{/;<4:;4<S/a0/98V@4;<4/�@;6w]/<98V[8/[email protected]/?4/689/Z:49/?4/<:9V9A>0/��d/19:9/6.0/?0/.@.<489014:9=@0;9A/9/19:<@:/?4/689/@;<4:Y9=4/7:ZY@=9Se b4A0/Y9<0/?4/.4:/68/��������/1:01:@4<Z:@0]/569A564:/6.6Z:@010?4/Y9r4:/9A<4:9234./;0/98V@4;<4/4/=0A9V0:9:/19:9/9/84A>0:@9?0/.@.<489/�@;6wSp |/=z?@70}Y0;<4/?0/.@.<489/014:9=@0;9A/�@;6w/;T0/10?4/.4:9A<4:9?0�/10:/4..9/:9rT0/4A4/;T0/[/?@.<:@V6W?0/.0V/9/A@=4;29/�b�06/�a�]/564/[/1fVA@=9/4/14:8@<4/80?@Y@=9234./;0/=z?@70Ss ���/c0;<:0A/c4;<4:/[/9/Z:49/?4/<:9V9A>0/?0/�@;6w/14A9/569A/.4Y9r/9=4..0/9/91A@=9<@X0./@;.<9A9?0./;0/=0816<9?0:]/=080/0y:|YY@=4/4/06<:0.St |/�@;6w/;T0/14:8@<4/564/.4_98/@;.<9A9?0./06<:0./.@.<489.014:9=@0;9@./;9/84.89/8Z56@;9]/10@./@..0/9Y4<9:@9/0?4.4814;>0/?0/=0816<9?0:]/<0:;9;?0}0/A4;<0S/$%&'()*+u�-..@;9A4/9/012T0/=0::4<9/=08/:4Y4:\;=@9/9/4?@2T0/?4/<4w<0.]1A9;@A>9./4/91:4.4;<9234./48/98V@4;<4/�@;6wSU |/y:|YY@=4/;T0/10..6@/@;<4:014:9V@A@?9?4/=08/06<:0.98V@4;<4.�/10:/@..0]/19:9/564/0./9:56@X0./74:9?0./;0/yj|YY@=4.4_98/A47WX4@./48/06<:0./=0816<9?0:4.]/[/;4=4..Z:@0/.9AXZ}A0.;0/Y0:89<0/S1?Ye y9.4/[/0/��������/6<@A@r9?0/19:9/.4/=:@9:/1A9;@A>9./4A4<:x;@=9.9/19:<@:/?4/?9?0./@810:<9?0./?4/06<:0./91A@=9<@X0.]/=080/0q9<>/4/0/c9A=Sp |/81:4../14:8@<4/9/=:@92T0/?4/@8974;./X4<0:@9@./?4.4;>0.d]9./569@./10?48/.4:/@81:4..9./10:/84@0/?4/569A564:/91A@=9<@X0?0/y:|YY@=4S/s |./91A@=9<@X0./?0/yj|YY@=4/14:8@<48/9/=:@92T0/?4/9:56@X0.;0./Y0:89<0./<W1@=0./?4/06<:0./98V@4;<4.]/=080]/10:/4w481A0]?9/.6W<4/q@=:0.0Y</q�d/|YY@=4S/�4..9/Y0:89]/4..4./9:[email protected]?48/.4:/9V4:<0./4/9A<4:9?0./14A0/91A@=9<@X0/=0::4.10;?4;<4?0/q�/|YY@=4St |/�:@<4:]/0/4?@<0:/?4/<4w<0./?0/y:|YY@=4]/[/=919r/?4/74:9:9:56@X0./;0/Y0:89<0/S?0=/06/0?<]/89./;T0/14:8@<4/9/4?@2T0/069A<4:92T0/?4/9:56@X0./48/><8AS3

marcusbelfort
Rectangle
Agnaldo
Nota
C
Agnaldo
Nota
C
Page 5: Exercicios Informatica CESPE 2010 250 Questoes

������������������ ������������������������������ � ��������!�!� "�#�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` .53603/<7/213/59353W58A5347107853453ZH[\]365W10323M102340812W2<N53453=7;45>6?3809_6032/0665323Q923R2;0<23PQ03/5;8L947O0162637;E5192@a0634536768092?3/595323PQ2;874240340306:2@5093476/53Q87<7F242?3093EQ;@A53456321PQ7O56303:15T12926;0<037;682<2456Bb c3:566XO0<3<5/2<7F213Q9321PQ7O53PQ030680R232192F0;2453;20681Q8Q12340347108d17563453=7;45>6?323:218713423E011290;82efghijgkl?3476:5;7W7<7F2423253603/<7/213/59353ZH[\]K65W1035W58A53mnjojklB3Y66235:@A535E010/03Q923R2;0<23453T010;/7245134021PQ7O56?3/595353=7;45>63YS:<5101?3/593/29:53:21247T782@A53453801953/51106:5;40;803:2123:06PQ76235Q3;59006:0/XE7/5345321PQ7O5Bp q3=7;45>63r:42803L3Q9310/Q165340328Q2<7F2@A534536768092=7;45>6?353PQ2<35E010/035:@a06340310;5O2@A53403<7/0;@2340Q65303829WL9340328Q2<7F2@A53453\HJstuv]32;87OX1Q6306:0/XE7/5423w7/1565E8B$%&'()*+,x.667;2<03235:@A53/511082323106:07853403/5;/07856?3E011290;826?2:<7/287O56303:15/04790;8563403y;801;08BC .3y;801;083L3E7;2;/72423:0<53/Q68534530;O75340390;62T0;60<081z;7/26?3263PQ27636A53/5;82W7<7F24263:0<563:15O0451063402/066533y;801;0830310:26624263:212353Q6QM175323:2187134236Q2/5;82380<0Ez;7/2?3459L687/235Q309:1062172<BU D21232/0665323Q92310403403/59Q;7/2@A53:51390753403Q923<7;N280<0Ez;7/235Q3403Q9367680923403{|323/2W5?3L3;0/066M175353Q654530PQ7:290;85340;597;2453ZH}]ZB` {2;85353y;801;083YS:<51013/595353~55T<03VN15903:019780932047@A53032<8012@A5340321PQ7O53;53E5192853N89<35Q3N89Bb D2123PQ03563424563812E0TQ093E2/7<90;803;23y;801;08?3Q87<7F29_602:0;263563:1585/5<563{VD�yD3:21232/0665331040?3030;O75340]�Zu�G30321PQ7O56Bp D513PQ068a06340360TQ12;@23453/59:Q82451?3Q92390;62T09340/51107530<081z;7/536590;803:540360132W01823603N5QO013\HJstuv]2;87OX1Q637;682<2453;239MPQ7;2B3 $%&'()*+,�V5;674012;45323E7TQ1232/792?3PQ037<Q68123Q923:MT7;230<081z;7/2?2667;2<03235:@A53/5110823/59310<2@A53253Q653453y;801;083YS:<5101�yY�3423w7/1565E8BC .:d6360347T78213530;4010@53403Q923:MT7;232360132/066242?35W58A53 3:0197803/5;E01713603066030;4010@53068M35Q3;A5/511085BU .3:218713453Z]�[3 3453yY?3L3:566XO0<3O76Q2<7F21PQ2<PQ013:MT7;23PQ030680R2321PQ7O2423;536X8753PQ03603068M2/0662;453;539590;85?303;A532:0;26323:MT7;237;7/72<��HZ]�uI]�B` D21236032/0662134080197;24536X875?3L3;0/066M17537;60171360Q0;4010@53;23W211234030;4010@56?35Q30;8A5?3Q87<7F2;45_603403Q990/2;76953403WQ6/23�/5;E5190353423E7TQ12395681242�?3WQ6/21Q93<5/2<3:513:2<2O12_/N2O035Q30;8A53:513Q93;590306:0/XE7/54536X87534060R24535Q340360Q3/5;80�45Bb .3E7TQ1232/79232:1060;82323:MT7;23:17;/7:2<34536X8753~55T<0?PQ03:54036013Q87<7F2453:2123:06PQ76263;23y;801;08?3/26535/59:Q8245130680R23/5;0/8245?35Q3:019780323WQ6/23407;E5192@a063<5/2<90;80?360353/59:Q824513453Q6QM1753;A50687O013/5;0/8245331040Bp q3W58A53 3476:5;7W7<7F23EQ;/75;2<7424063PQ0:01978093O76Q2<7F213Q923<76823403:MT7;263927632/06624263:0<5yY3;23y;801;08B$%&'()*+,�./01/234263EQ;@a0637;6878Q/75;2763423�D?32667;2<03235:@A53/511082BC q37;681Q90;853403812;62@A535Q3/5;/7<72@A5310E010;42453:513Q9�D3:2662323O2<013/59538X8Q<530S0/Q87O53RQ47/72<BU .3�D3:540328Q213093E2O513403:066523RQ1X47/2?382;85309:15/066532497;7681287O53/5953RQ47/72<?30938542632637;68�;/7265147;M17263030S8125147;M1726B` c3O042453253�D328Q213;563RQ7F2456306:0/7276Bb �2632@a063:0;276?323/2:2/742403:568Q<28d1723453�D340/51100S/<Q67O290;803423:15/Q12@A535Q851T2423:0<23OX87923253�D4067T;2453:212353/265Bp .3�D340O01M32/59:2;N2137;PQL178563:5<7/7276?37;/<Q67O0326:176a063093E<2T12;80?380;45353:10653/5;6878QX45324O5T2455Q3;A5B4

marcusbelfort
Rectangle
Agnaldo
Nota
C
Agnaldo
Nota
C
Agnaldo
Nota
C
Page 6: Exercicios Informatica CESPE 2010 250 Questoes

���������������� �������������������� ���������������� �!�"����#��$� ��%�&�'() ���& �!�*�!+,-./0123456789:;<=57>79?@8A@A8;7B57@9C@5797;57D5E;FA:G8H57I9:9796J89K;B5L;??HI;:97;75J<=57E5889@;MN O7J;:;D8;7PQRPSTRUVW79?@G7H6J:XEH@;7:5K57;JY?757D5EGFA:5Z;:KAI?[7I57@89E\57Z]HE;IB57;:KAI?7;:H;B5?7B57J85E9??5B9EH?Y8H5[7_MabcadeMf O7?AF?@H@AH<=57B97Z?5F89[7_Mbde7J587WVg7;:@98;8H;757?9I@HB558HKHI;:7B57@9C@5L76;?76;I@98H;7?A;7E598hIEH;797?A;7E5889<=5K8;6;@HE;:Mi j7@89E\579I@897@8;D9??k9?7I;?7:HI\;?7la797lb79CJ:HE;7;9CJ89??=57Z@5B5?75?79ID5:DHB5?[7_MllclaeMm j7D5EGFA:57Z89E58B9[7_Mlde7@;6Fn67J5B98H;7?987E5889@;69I@9K8;];B57E567;E9I@57o7pqrVpRsMt O7?AF?@H@AH<=57B97ZE56J5?@;7B9[7_Mlue7J587rVQWvPvUTRwxRs;E;889@;8H;7J89AXy57B97?9I@HB57;57@9C@5M+,-./012z45I?HB98;IB57;?79?@8A@A8;?7B57@9C@5L7;??HI;:97;75J<=57E5889@;7I57{A9BHy789?J9H@57>7E5IE58B|IEH;MN O7;:@98;<=57B97Z?9;67K;8;I@HB;?[7_Ma}e7J;8;7Ws~wx�wpwQvPRVI=57HI@98]9897I;7E5889<=57K8;6;@HE;:7B57J98X5B5Mf O?7]586;?7D98F;H?7ZK;8;I@96[7_Mble797Z5F8HK;6[7_MbaeE5IE58B;67E567Z9:9H<k9?7J98HYBHE;?[7_MbleMi O7HI?98<=57B;7]586;7D98F;:7�wQvsps�7I57:AK;87B97Z6;I@98[L967Z6;I@987A675:\57I;75JHIH=57B57J5D5[7_MbbeL7;E;889@;8H;J89AXy57?HI@G@HE57;57@9C@5Mm O758;<=57s�PWvPwxw��UQWx�wvVpsWxPQSPwgP�P�wRVpsWxJ;8;]8;?9H;B9765B57K8;6;@HE;:69I@97E5889@5757@89E\57Z;:KAI?7];@589?7?9;J89?9I@;8;67E5657HIDH;FH:Hy;B589?[7_Muc�eMt OHIB;7{A9757D5EGFA:57ZI9E9??HB;B9[7_Ml}e79?@HD9??9]:9CH5I;B57I57J:A8;:L7;7]586;7D98F;:7ZHB9I@H]HE5A[7_MldeB9D98H;7J986;I9E987I57?HIKA:;8M+,-./012��45I?HB98;IB57;?7I586;?7B9789B;<=575]HEH;:L7;??HI;:97;75J<=5E5889@;MN �5B5?75?79CJ9BH9I@9?75]HEH;H?7B9D967?9KAH87;769?6;BH;K8;6;<=5Mf j7A?57B57J;B8=57EA:@57B;7:XIKA;7B9D97?9879DH@;B57I;?789B;<k9?5]HEH;H?L7J5H?7BH]HEA:@;7579I@9IBH69I@57B5?7@9C@5?7J587J;8@97B;J5JA:;<=57967K98;:Mi O789B;<=57B97A67B5EA69I@575]HEH;:7B9D97DH?;8J8H658BH;:69I@97579I@9IBH69I@57B;769I?;K967J9:57?9A8969@9I@9L7J;8;7{A96L7B97];@5L79:;7n7H6J58@;I@9Mm j789KH?@857B97H6J89??k9?7J9??5;H?7I;?7E56AIHE;<k9?75]HEH;H?I=57B9D975E58898L7;HIB;7{A97579CJ9BH9I@97;J89?9I@97?HKI;@G8H597B9?@HI;@G8H57HB9I@H]HE;B5?7I56HI;:69I@9Mt 45657]586;7B97AIH]586Hy;87;?7E56AIHE;<k9?75]HEH;H?L96J89K;c?975769?657D5E;@HD57967@5B;?79:;?LHIB9J9IB9I@969I@97B97?9A7B9?@HI;@G8H5M +,-./012��456789:;<=57;57?H?@96;75J98;EH5I;:797;57;6FH9I@97�HIACL7;??HI;:9;75J<=57E5889@;MN j7������7n7A67��������7{A97?97HI?@;:;7B9I@857B57�HIAC797];yE567{A9757�HIAC7J5??;7?987BH?@8HFAXB57K8;@AH@;69I@9Mf j7�HIAC75]989E97;75J<=57B97{A97A67I5D57A?AG8H57J5??;7;F8H8A6;7?9??=57B97A?57B57;6FH9I@97J;8;7A@H:Hy;87?9A?7;J:HE;@HD5?69?657{A975A@857A?AG8H579?@9;7:5K;B57I57?H?@96;Mi j7�HIAC7J5B97?987;E9??;B57;7J;8@H87B;7G89;7B97@8;F;:\57B5�I5697;J9I;?L7J5H?757���7n7A6;7���7{A97?Y7J5B97?98A@H:Hy;B;7967E56JA@;B587{A97@HD987HI?@;:;B5757�HIB5�?7J;8;A?57?H6A:@|I95Mm �9FH;I7n7A6;7B;?7BH?@8HFAH<k9?7B57�HIAC76;H?7A@H:Hy;B;?7I56AIB5�7I579I@;I@5L7?A;7HI@98];E97I=57?AJ58@;7;7:XIKA;J58@AKA9?;L7?9IB57I9E9??G8H57E5I\9EH69I@57B97HIK:h?7J;8;;E9??5Mt j7�HIAC75]989E97];EH:HB;B97B97HI@98;<=579I@897��������7B9BHD98?;?7J:;@;]586;?�7I579I@;I@5L7I=57J986H@97{A97?9;6E8H;B5?7��� ���7B97E5I]HKA8;<=57J;8;7{A975A@85?7¡�������J5??;6785B;87I57�HIACM+,-./012�¢O??HI;:97;75J<=57E5889@;7;789?J9H@57B;79BH<=57B97@9C@5?L7J:;IH:\;?79;J89?9I@;<k9?7967;6FH9I@97�HIACMN �67;8{AHD579BH@;B57J9:57£¤j]]HE97967]586;@57I;@HD57J5B97?98;F98@57J587{A;:{A9875A@857;J:HE;@HD57B57698E;B5L7J5H?79:97]5H]9H@57J;8;7?987;F98@5797E56J;@XD9:7E567@5B5?75?7;6FH9I@9?L7I59I@;I@5757E5I@8G8H57I=57n7D98B;B9H85Mf j74;:E7n7A6;7]988;69I@;7¥@H:7I;7E5I?@8A<=57B97]Y86A:;?6;@96G@HE;?7J;8;7EG:EA:5L7J5??AH7;?769?6;?7]AIEH5I;:HB;B9?B57�CE9:7B;7¦HE85?5]@L76;?7J5??AH7?HI@;C97B97]Y86A:;?BH]989I@97B;?7A@H:Hy;B;?7J5879?@9Mi j7£¤j]]HE97n7A6;7?AX@97B97;J:HE;@HD5?7{A97J5??AH7D98?k9?9?J9EX]HE;?7J;8;7�HIAC797J;8;7�HIB5�?�7I579I@;I@5L7J;8;9C9EA@Gc:57I57�HIB5�?L7n7I9E9??G8H57];y987��§���797;E9??;875�HIAC7J;8;7A@H:HyGc:5Mm 57�6J89??L7n7J5??XD9:7E8H;87A6;7I5D;7;J89?9I@;<=57B97������97?;:DGc:;7I57]586;@57J;B8=57I;@HD57B97;8{AHD5?7J;8;79??9;J:HE;@HD5Mt O@A;:69I@9L757]586;@57J;B8=57B5?7;8{AHD5?7B57�8H@987n75©ª�L7967?AF?@H@AH<=57;57j��L7{A97;J89?9I@;D;7BHD98?;?:H6H@;<k9?7B97E56J;@HFH:HB;B9M75

marcusbelfort
Rectangle
Agnaldo
Nota
C
Agnaldo
Nota
C
Page 7: Exercicios Informatica CESPE 2010 250 Questoes

���������������� �������������������� ���������������� �!�"����#��$� ��%�&�'() ���& �!�*�!+,-./0123456789:;<=>68<68?@68:8A@86B=C:@8D68E6FG;;<8HG9:I6JK8L:MD68N6768O<@:8<8IGP?9<8<NG7<8Q?:876@L9<8?7<8BRPGM<8<N:@@<D<8N678:@@:8M<S:P<D69K<@@GM<;:8<86B=>68N699:L<TU 5<@68?78N<D:<D68I:NV<D68LGS:@@:8@GD68<B9:@:ML<D68M6896D<BW8D:@@<8BRPGM<K8:;:8GMDGN<9G<8Q?:868HG9:I6J8:@LR8N6M:NL<D68<8?78@:9SGD69XYZ8Q?:8B96S[8@?B69L:8<68\]]8@:P?96T_ @<MD6868O6L>68 K8W8B6@@aS:;8L:98<N:@@68G7:DG<L68<6@8D:F8@aLG6@87<G@8<N:@@<D6@8:78L6D<8<8bML:9M:L8:8IGN<98B698D:ML968D6@B9GMNGB<G@8<N6ML:NG7:ML6@T8]<7OW78W8B6@@aS:;8N6MIGP?9<98:@@<86B=>68B698<@@?ML686?8R9:<8L:7RLGN<K8B<9<8@:8;G@L<986@8B9GMNGB<G@8@aLG6@N699:;<L6@Tc d<86B=>68efghijkljmnop8D68qYrs8 K8W8B6@@aS:;8L:98<N:@@68<68NtDGP68uvwx8D68D6N?7:ML68:8I<F:98<;L:9<=C:@8M:N:@@R9G<@K8N<@6V<y<8<;P?78:9968D:8N<99:P<7:ML68M<8BRPGM<Tz 698B<D9>6K8<68@:8N;GN<9868O6L>68 K8@:9R8<O:9L<8?7<8M6S<8L:;<K8N678687:@768N6ML:{D68D<8L:;<876@L9<D<8M<8IGP?9<T| }86B=>68 8D<8O<99<8D:8qYrs~�:JGO:8<@8S:9@C:@8<ML:9G69:@8D<@8BRPGM<@8D68@aLG68D<8� K8yR8B?O;GN<D<@T+,-./0123�56789:;<=>68<6@8<B;GN<LGS6@8B<9<8:DG=>68D:8L:JL6@K8B;<MG;V<@8:<B9:@:ML<=C:@8:78<7OG:ML:8�GMD6�@K8<@@GM<;:8<86B=>68N699:L<TU d6876D68D:8<B9:@:ML<=>68D:8~���Y~8D68E�6�:96GMLK8Q?:B6D:8@:98<NG6M<D68B6987:G68D<8L:N;<88�86?8D68O6L>68 KW8B6@@aS:;8SG@?<;GF<986@8~���Y~8D:8?7<8<B9:@:ML<=>68:7N6My?ML6@8D:8Q?<L968~���Y~8B698L:;<T8_ }68@:8GM@:9G98:78?78D6N?7:ML68D68�69D8?78aMDGN:8<M<;aLGN6KW8M:N:@@R9G68Q?:8L6D<@8<@8:ML9<D<@8D68aMDGN:8:@L:y<787<9N<D<@B6987:G68D:8?78:@LG;68:@B:NaIGN6Tc �8N67<MD68lf�����8D:8?7<8L<O:;<8D68�69D8B:97GL:8<NG6M<968�JN:;8B<9<8Q?:8B6@@<8@:98GMN;?aD<K8D:ML968D68D6N?7:ML6K8?7<L<O:;<8D68�JN:;8N678N<7B6@8B9W�I697<L<D6@8B<9<8NR;N?;6@M?7W9GN6@Tz }8:JL:M@>68D:8<9Q?GS6@8T����8D68�GMD6�@8W8?7<8<;L:9M<LGS<B<9<8Q?:8@:y<78@<;S6@8L:JL6@8Q?:8B6@@<78@:98<O:9L6@8:786?L96@<7OG:ML:@86B:9<NG6M<G@K8N676868�GM?JK8B698:J:7B;6T8| d68�69DK8B6D:�@:8GM@:9G98?7<8Q?:O9<8D:8BRPGM<8<Bt@8<8{;LG7<;GMV<8DGPGL<D<8L:N;<MD6�@:8@G7?;L<M:<7:ML:88� 8:8�T +,-./0123�}@@GM<;:8<86B=>68N699:L<8<89:@B:GL68D:8N6MN:GL6@8OR@GN6@KI:99<7:ML<@K8<B;GN<LGS6@8:8B96N:DG7:ML6@8D:8bML:9M:LTU @<MD68<8I:99<7:ML<8]:;M:LK8B6D:�@:8S:9GIGN<98@:8?7<87RQ?GM<:@LR86?8M>68M68<9K8:8<LW87:@7686OL:9868:MD:9:=68b8D:@@<7RQ?GM<T_ �8?@68D6876D:;68��b8B:97GL:8?7<87:;V698GML:9N6M:J>68:ML9:6@8DGS:9@6@8B96L6N6;6@8D:89:D:@K8Q?:8@>68:@L9?L?9<D6@8:78@:L:N<7<D<@K8DGSGDGD<@8:78L9[@8P9?B6@�8:ML9<D<K8B96N:@@<7:ML68:@<aD<Tc �8N699:G68:;:L9�MGN68W8?78@:9SG=68D:8L96N<8D:87:M@<P:M@8D:L:JL6K8Q?:8B6D:78N6ML:98<9Q?GS68<M:J<D6T8�@@:8@:9SG=68?LG;GF<?78B96L6N6;68:@B:NaIGN68D:M67GM<D68H]Tz 78q��Yq8}���8W8?78:Q?GB<7:ML68Q?:8B:97GL:8Q?:8?7<;GMV<8L:;:I�MGN<8@:y<8N67B<9LG;V<D<8@G7?;L<M:<7:ML:8B69L9RI:P68<M<;tPGN68D:8S6F8:86?L968DGPGL<;8D:8D<D6@T| �:8<8N6ML<8D:8Y�q���8:@LR8;6N<;GF<D<8:78?78@:9SGD698D68LGB6�JNV<MP:8�:9S:9K8Q?<MD6868?@?R9G68<N:@@<98<@8@?<@87:M@<P:M@K:;<@8@>68<?L67<LGN<7:ML:8O<GJ<D<@8B<9<8<87RQ?GM<8?@<D<8B:;6?@?R9G68B<9<8I<F:9868<N:@@68A8N6ML<K8M>68IGN<MD68NtBG<8D<@7:M@<P:M@8<N:@@<D<@8M6@8@:9SGD69T6

Agnaldo
Nota
c
Agnaldo
Nota
c
Page 8: Exercicios Informatica CESPE 2010 250 Questoes

��������������� ������������������������������ �� !����"�#$% &�'(�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aVbXA67<=YA=9689;<=>5?89;86<7J=A89D8=9;<A89?596<P8>A5EF89?<7[74C8:9D=]D=A8:9689;<=>5?895T<=789<9?59>86><::F89?<<;D=\:7A;8:9589c<:84=89X5>A865CGd V5T<9589aVbX9<9M9V8;A::F89?<9e5C8=<:9W8TACAJ=A8:9fVeWg<:75T<C<><=968=;5:9K4<9?A:>ADCA6<;95:957AYA?5?<:9?5:9T8C:5:9?<Y5C8=<:9<9?8:9>8==<78=<:9?<9@46?8:9DhTCA>8:G)*+,-./01ia9=<:D<A789?89>j<K4<B95::A65C<9598DEF89>8==<75GH X59jAD]7<:<9?<9A6:4@A>AQ6>A59?<9@46?8:9?89>8==<67A:75B95C<PA:C5EF8957=AT4A9589T56>89:5>5?8959=<:D86:5TACA?5?<9D<C8D5P5;<6789?89>j<K4<9589>=<?8=G9U I9>j<K4<9?<Y8CYA?89D8=9@5C759?<9@46?8:9659D=A;<A=55D=<:<675EF898T=AP5989T56>8959A6>C4A=9:<49<;A7<67<9689>5?5:7=8?<9<;A7<67<:9?<9>j<K4<:9:<;9@46?8:GZ I9>j<K4<95?;A6A:7=57AY8B97[74C89<;A7A?89D<C89D=]D=A89T56>8BD8?<9:<=9<;A7A?89589D8=75?8=G^ a9C<PA:C5EF89K4<9?A:>ADCA65989>j<K4<96F895?;A7<989>j5;5?8<6?8::89:<;9P5=567A5B9D<C89K45C989<6?8::567<9<RD=<::5;<67<:<9<RA;<9?598T=AP5EF89?<9D5P5;<6789?897[74C89<;9@5Y8=9?8?<Y<?8=9D=A6>AD5CGd _<95>8=?89>8;959k4=A:D=4?Q6>A59?89l4D<=A8=9c=AT465C9?<m4:7AE5B9595D=<:<675EF89567<>AD5?59?<9>j<K4<9D=\N?575?8>5=5>7<=AL59?5689;8=5CG)*+,-./01na9<R<>4EF89?8:9:<=YAE8:9T56>J=A8:9?<9A67<=<::<9?89P8Y<=689@<?<=5CB5::A;9>8;89?<9A67<=<::<9?<9:45:95475=K4A5:B9>5T<B9D=A6>AD5C;<67<BH M9VeWGU 58:9T56>8:9>88D<=57AY8:GZ 589aVbXG^ 58956>89?89=5:AC9lGaGd 589 56>89X5>A865C9?89_<:<6Y8CYA;<6789c=AT47J=A89<oA656><A=8G )*+,-./0pqX89=5:ACB95:9A6:7A74AEO<:9@A656><A=5:9DhTCA>5:9:F89>86:A?<=5?5:H <67A?5?<:9=<:D86:JY<A:9D<C59<R<>4EF89?59D8C[7A>597=AT47J=A59?8:P8Y<=68:GU ]=PF8:954RACA5=<:9?59<R<>4EF89?59D8C[7A>59?<9>=\?A789?8P8Y<=689@<?<=5CGZ <67A?5?<:9@A:>5CAL5?8=5:9?89lA:7<;59oA656><A=89X5>A865CG^ <67A?5?<:9D=<:75?8=5:9?<9:<=YAE8:9?89aVbXGd ]=PF8:9<R<>478=<:9?59D8C[7A>59?<9A6Y<:7A;<678:9?89P8Y<=68@<?<=5CG)*+,-./0prV8;9=<C5EF89M9k56<C59?89sA6?8t:9bRDC8=<=9;8:7=5?59D5=>A5C;<67<5>A;5B95::A65C<9598DEF89>8==<75GH X59@5C759?<9>86<RF89?89>8;D475?8=9>8;959u67<=6<7B989sA6?8t:bRDC8=<=9@A>59A6?A:D86[Y<C9D5=59894:4J=A8G9U a9D5=7A=9?89[>86< B9\9>8==<78A6@<=A=9K4<989>8;D475?8=9<;94:897<;95D<65:94;5946A?5?<9?<<67=5?59D5=59_e_B9<96F89:4D8=759@8=;578:9P=5YJY<A:9<;9V_GZ I9T87F89 B9j5TACA75?89659k56<C59AC4:7=5?5B9\947ACAL5?89K456?8:<9?<:<k59YA:45CAL5=95:9D5:75:9>8;9A6@8=;5EO<:9?<75Cj5?5:9?<75;56j8B97AD89<9?5759?<9;8?A@A>5EF8G^ a9D5=7A=9?59k56<C59;8:7=5?5B9\9>8==<789A6@<=A=9K4<\989?A=<7]=A89=5AL9?<9;5A:95C759jA<=5=K4A5968:A:7<;598D<=5>A865C9sA6?8t:G9X<::<9?A=<7]=A8B9<:7F89>867A?8:78?8:98:9?<;5A:B9>8;89vwxyz{|}x~w��|y9<9vwxz}|~�x��{|�BYA:[Y<A:9D<C8:9T87O<:9?<9<RD56?A=9<9=<>45= Gd I9[>86< A6?A>5989?A:>89=[PA?89?8>8;D475?8=B986?<9@A>5;9P=5Y5?8:98:95=K4AY8:9?89:A:7<;58D<=5>A865C9sA6?8t:G7

marcusbelfort
Rectangle
marcusbelfort
Rectangle
Agnaldo
Nota
c
Page 9: Exercicios Informatica CESPE 2010 250 Questoes

��������������� ������������������������������ �� !����"�#$% &�'(�&)*+,-./012 3456789:;<6953<3=8>?;<3<@8A<B3C?:38D?7E;<3<3F<6:D<3GHIJKLMNKMOPJQRPLKS953T8695U7B3<7786<D:3<35VWX53@5;;:E<YZ [5;3A:8539:3=?6@856<D89<9:73:6@56E;<9<736<3F<6:D<3<775@8<9<3<53\@56:3 B3]3V577\:D35_E:;386=5;A<W:73>:;<87375_;:353787E:A<5V:;<@856<DB3_:A3@5A53@56=8>?;<;353V;5@:77539:35_E:6WX53:3867E<D<WX539:3<E?<D8a<W:73953T8695U7Y3b c537:3@D8@<;353\@56:3 B37:;d3<_:;E<3?A<3F<6:D<3953e6E:;6:E3fgVD5;:;B353C?:3V:;A8E:353<@:7753<3?A<3Vd>86<3hijB3V5;3A:859:3;:9:73@<_:<9<735?37:A3=85B39:79:3C?:B3653@<AV53<V;5V;8<9539:77<3F<6:D<B37:F<386=5;A<95353:69:;:W53@5AVD:E539<3Vd>86<3C?:37:9:7:F<3<@:77<;B37:A3:;;5739:398>8E<WX5Yk [5;3A:853953\@56:3 B3]3V577\:D3<@:77<;398;:E<A:6E:3<73@56E<739:3ilmnop3:3<73d;:<739:3<;A<a:6<A:6E53:7V:@\=8@<73957?7?d;8573@<9<7E;<9573653@5AV?E<95;3:A3?75Yq c73=?6@856<D89<9:73;:D<@856<9<73<53\@56:3 9::A37:;3?E8D8a<9<737:AV;:3C?:37:3<98@856<35?3;:A5:3C?<DC?:;3V;5>;<A<C?:3:7E:F<3867E<D<953653@5AV?E<95;3:A3?75B39:3=5;A<3<36X53@<?7<;3867E<_8D89<9:3653787E:A<35V:;<@856<DYr 4<7537:39:7:F:35?8;3As78@<35?3<@:77<;353<A_8:6E:39:3d?9853V<;<3:g:@?E<;3<;C?8573953E8V53Y<8B3]3V577\:D3=<atuD53V5;3A:8539:=?6@856<D89<9:73:6@56E;<9<73<537:3@D8@<;353\@56:3 Y)*+,-./01vc3;:7V:8E53953<A_8:6E:3w86?gB3<7786<D:3<35VWX53@5;;:E<YZ x3<@:7753y3e6E:;6:E3653<A_8:6E:3w86?g3]3;:<D8a<95B3V5;V<9;X5B3V5;3A:853953<VD8@<E853z:;6:DYb {53<A_8:6E:3w86?gB3V59:3|<:;398:;7573E8V5739:86E:;=<@:73:3?7539:3D8@:6W<7Y3}A<39:77<7386E:;=<@:73]35z~fB3C?:3?E8D8a<3<3D8@:6W<3�{}Yk c3987E;8_?8WX53~:_8<63953w86?g3E:A3<37?<3?E8D8a<WX5D8A8E<9<3V5;36X535=:;:@:;3=:;;<A:6E<73<9A8687E;<E8<76:A386E:>;<WX53@5A35?E;573<A_8:6E:73@5AV?E<@856<87Yq x3w86?g3]3?A3787E:A<3>;<E?8E539:7:65D8953653�;<78D@5A3535_F:E8539:3;:7E;86>8;353?7539:3787E:A<75V:;<@856<8739:3<DE53@?7E5B3@5A5353T8695U7B3657@5AV?E<95;:739573�;>X573>5:;6<A:6E<87Yr cV<@|:3]3<39:65A86<WX539:3?A<3@5A?689<9:39:9:7:65D:95;:739:3����hn�i3>;<E?8E53V<;<3<@:7753y3T:_BC?:3=583=5;A<9<36573f7E<9573}6895739<3cA];8@<B3A<7|5F:3:7Ed39877:A86<9<3:A3d;8573V<\7:7B386@D?78:365�;<78DY )*+,-./01�45A3;:D<WX53y73=:;;<A:6E<73953V<@5E:3��x==8@:B3<7786<D:3<35VWX5@5;;:E<YZ fA3?A3<;C?853@;8<953V:D53<VD8@<E8534<D@B3]3V577\:D3<;A<a:6<;365Adg8A53E;t73VD<68D|<73:D:E;�68@<7B3<73C?<873V59:A3@56E:;3=�;A?D<79:3@dD@?D573:3�o�i�po���3V<;<35?E;<73;:=:;t6@8<73:3_<6@5739:39<957Yb x73<;C?8573>:;<9573V:D53��x==8@:3657398:;7573=5;A<E573V59:A7:;3@56=8>?;<9573V<;<38AV;:77X53V:D53eAV;:77B39:3<@5;953@5A3<7V;:=:;t6@8<73953?7?d;853:3@56789:;<6953<73@<;<@E:;\7E8@<739<8AV;:775;<3@56:@E<9<3<53@5AV?E<95;Yk x3T;8E:;3]353<VD8@<E853953��x==8@:3:C?8<D:6E:3<53T5;939<�8@;575=EB3V:;A8E86953;:<D8a<;3V;<E8@<A:6E:3<73A:7A<73<W:739:;:9<WX53:3:98WX539:3E:gE57378AVD:73:3A?DE8A\98<B3A<7B3<53@56E;d;85953T5;9B36X53V:;A8E:3C?:37:F<A37<D573<;C?8573653=5;A<E53Y[~�B:A3=?6WX539:386@5AV<E8_8D89<9:3@5A:;@8<DYq {573�po�i�3:D<_5;<9573V5;3A:853953eAV;:77B3]3V577\:D386@D?8;365E<7V<;<3:98WX539:3E:gE573C?:35;8:6E:A3<386E:;V;:E<WX53953@56E:s95395�po�iY3x3E:gE539<365E<36X53=8@<387\:D3V<;<3<3<?98t6@8<3C?<695357�po�i�3:7EX537:6953:g8_89573653A5953<V;:7:6E<WX539:3�po�i�Yr �<7:3]353<VD8@<E853953��x==8@:3C?:3V:;A8E:3<3@;8<WX539:39:7:6|57:E5;8<873:A3<A_8:6E:3w86?gB3E:6953<73A:7A<73=<@8D89<9:7:6@56E;<9<7365345;:D~�cT3:3653[<86E39<3�8@;575=EY8

Agnaldo
Nota
c
Agnaldo
Nota
c
Agnaldo
Nota
c
Page 10: Exercicios Informatica CESPE 2010 250 Questoes

��������������� ������������������������������ �� !����"�#$% &�'(�&)*+,-./01123456789:473;:;<6=>9:;:?6@:A;B>8;6C>5D9:;>@:;E:48C:;73;F397;7:;5>GD8;H6?9353<D;I<<6?8A;?3@;>@;73?>@84D3;8@;J93?8553;78;876KL3A:5564:C8;:;3JKL3;?3998D:MN I;?34E>4D3;78;3JKO85;78;<39@:D:KL3;765J34GP865;8@; ;J89@6D8;78<6469A;43;<39@:D3J:79L3;J:9:;:;F8QA;:5;?34<6=>9:KO85;78;R6J89D8SD3;8;D:@QT@;78;P65>:C6U:KL3;73;73?>@84D3;8@;876KL3MV W:;56D>:KL3;7:;E:48C:;@35D9:7:A;T;?3998D3;:<69@:9;B>8;:;8SJ9855L3;XWIYZ[\;][;W_I Hab2cd;85De;58C8?634:7:;8;D8@;:;<34D88@;48=96D3M;f:9:;58;8C6@64:9;3;<39@:D3;48=96D3;7855:;8SJ9855L3A;T;5><6?684D8;?C6?:9;3;Q3DL3; A;4:;Q:99:;78;<899:@84D:5Mg c;J:9D69;7:;<6=>9:;@35D9:7:A;T;?3998D3;64<8969;B>8;D373;3;D8SD3;76=6D:73;85De;?3@;:;<34D8;8@;48=96D3;8;B>8;3;73?>@84D3;8@;876KL3;D8@:5;7>:5;@:9=845A;85B>897:;8;76986D:A;E>5D6<6?:7:5Mh I;G?348; A;B>8;:J:98?8;C3=3;:Q:6S3;73;73?>@84D3A;6476?:;B>8;Re;?3998KO85;78;=9:<6:;:;5898@;?34<8967:5;43;D8SD3;76=6D:73M;i f39;@863;78;<>4?634:C67:785;765J346Q6C6U:7:5;43;jklm; A;T;J355GP8C;?34<6=>9:9;3;D:@:4R3;7:;<34D8A;:;765J356KL3;735J:9e=9:<35;8;@:9=845A;:556@;?3@3;6458969;B>8Q9:5;78;C64R:A;4n@893;78;Je=64:;73;73?>@84D3;3>;3>D9:5;<39@:D:KO85;78;85D9>D>9:M)*+,-./01oc?89?:;735;?34?86D35;78;4D8948D;8;plqrslkqA;:5564:C8;:;3JKL3?3998D:MN [@;>@:;plqrslkqA;:;D93?:;78;7:735;84D98;?C684D85;8589P673985;T;98:C6U:7:;?3@;3;>53;73;J93D3?3C3;F6t_6MV [@;=9:4785;e98:5;78;>@:;?67:78A;?3@3;:893J39D35;8uvwxxply;zklqkruA;439@:C@84D8;58;84?34D9:;765J34GP8C:;D8?43C3=6:;FcWA;B>8;T;>@;D6J3;78;9878;58@;<63;J:9::?8553;{;4D8948DMg I;:?8553;:;Je=64:5;|k}A;7:;4D8948D;3>;78;>@:plqrslkqA;T;<86D3;?3@;3;>53;73;~bbf;3>;J93D3?3C3;78R6J89D8SD3Mh \Hbf;T;>@:;D8?43C3=6:;>D6C6U:7:;:J84:5;4:;plqrslkqAB>:473;4L3;8S65D8;?348SL3;?3@;:;4D8948DM;i _bf;T;3;J93D3?3C3;78;84P63;8;98?8Q6@84D3;78;k�jsp�>D6C6U:73;B>:473;:5;@845:=845;78;?399863;8C8D9�46?34L3;J355>8@;98@8D84D85M )*+,-./01��>:4D3;{;39=:46U:KL3;78;:9B>6P35;76=6D:65A;:5564:C8;:;3JKL3;?3998D:MN I5;:9B>6P35;78;645D:C:KL3;78;J93=9:@:5;78P8@;<6?:9;:9@:U84:735;8@>@;7698D�963;D8@J39e963;73;565D8@:;3J89:?634:CA;:;<6@;78;<:?6C6D:9;:5>:;98@3KL3;B>:473;3?3998;J93QC8@:5;78;8S8?>KL3M;W8558;?:53AT;5><6?684D8;78C8D:9;3;98<89673;7698D�963;D8@J39e963MV W:;9878;78;?3@J>D:73985;78;>@:;8@J985:A;98?3@847:t58;B>8;35:9B>6P35;?3@;64<39@:KO85;6@J39D:4D85;58E:@;=9:P:735;:J84:5;4:5@eB>64:5;735;>5>e9635A;J365A;@85@3;8@;?:53;78;<:CR:;4:;9878A;3>5>e963;J378;?34D64>:9;D9:Q:CR:473;58@;64D899>JKO85Mg W3;:@Q684D8;F6473�5A;J:9:;58;=9:P:9;>@;:9B>6P3;4:;J:5D:���������������A;T;48?855e963;B>8;3;:9B>6P3;85D8E:;43;<39@:D3;M73?A;EeB>8;855:;J:5D:;T;8S?C>56P:;J:9:;:9@:U84:9;73?>@84D35;?96:735;43F397A;8;4L3;J89@6D8;=9:P:KO85;78;:9B>6P35;?3@;3>D935;<39@:D35M;h f39;@863;73;F6473�5;[SJC3989A;J:9:;58;?3J6:9;D3735;35;:9B>6P35;78>@;7698D�963;J:9:;3>D93;785D643A;T;5><6?684D8;98:C6U:9;3;58=>64D8J93?876@84D3�;58C8?634:9;D3735;35;:9B>6P35;73;7698D�963�;43;jklm������A;?C6?:9;:;3JKL3;�������;58C8?634:9;3;7698D�963;785D643;7858E:73�?C6?:9A;43;jklm;������A;:;3JKL3;�����Mi W3;:@Q684D8;�64>SA;J:9:;8P6D:9;J93QC8@:5;78;7858@J84R3;73?3@J>D:739A;98?3@847:t58;B>8;>@;7698D�963;?34D84R:;:9@:U84:735;:DT��;:9B>6P35A;?:7:;>@;78C85;?3@;D:@:4R3;78A;43;@eS6@3A;��;H�M9

Agnaldo
Nota
c
Agnaldo
Nota
c
Agnaldo
Nota
c
Page 11: Exercicios Informatica CESPE 2010 250 Questoes

��������������� ������������������������������ �� !����"�#$% &�'(�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a b;<67E?;NO4<8;<F;59D;<A46E:;8;B<V<@4::9E4<75=9:7:<C?9<4<6\E74<@4::96K45895E9<;4<9589:9N4<cEEK6deefffP@;7J;P>4WPM:<K:4W;W9DA95E996E;W;<=4:;<84<;:B<54<A4A95E4<9A<C?9<4<;@9664<=47<E95E;84Pg _<A484<hiijklY<V<?E7D7H;84<K;:;<5;W9>;NO4<9A<KL>75;6<89<klmnolYm<;K95;6<K;:;<?6?L:746<@;8;6E:;846<K9D4<;8A7576E:;84:<8;<:989<K4:A974<89<?A;<695c;<K9664;D<9<75E:;56=9:\W9DPp G96A4<9A<A484<hiijklYB<V<K466\W9D<?E7D7H;:<4<G4H7DD;<I7:9=4J<K;:;<W76?;D7H;:<?A;<KL>75;<XYZ<9A<=4:A;E4<cEAD<4?<9C?7W;D95E9B<C?996E9F;<;:A;H95;8;<54<@4AK?E;84:<9A<?64P)*+,-./01qR6675;D9<;<4KNO4<@4::9E;<;<:96K97E4<89<@9:E7=7@;NO4<87>7E;DPQ R?E4:78;89<@9:E7=7@;84:;<V<;<8954A75;NO4<89<?6?L:74<C?9<E9AK489:96<89<;@9664<r6<75=4:A;Ns96<@45E78;6<9A<?A;<A956;>9A;6675;8;B<K:7W;8;<9<@9:E7=7@;8;P^ R<;?E4:78;89<:9>?D;84:;<E9A<;<=?5NO4<89<9A7E7:<@9:E7=7@;84687>7E;76B<=?5@745;584<@4A4<?A<@;:E:74<8;<t5E9:59EPa _<tut<St56E7E?E4<b;@745;D<89<u9@54D4>7;<8;<t5=4:A;NO4U<VE;AMVA<@45c9@784<@4A4<R?E4:78;89<39:E7=7@;84:;<v;7Hw:;67D97:;Pg xyt<4?<t3x<V<4<54A9<8;84<;4<@9:E7=7@;84<C?9<=47<9A7E784<K4:?A;<;?E4:78;89<@9:E7=7@;84:;P<p zA<@9:E7=7@;84<87>7E;D<V<K9664;DB<75E:;56=9:\W9D<9<5O4<K466?78;E;<89<W;D78;89P )*+,-./0{|R@9:@;<89<;6675;E?:;<87>7E;DB<;6675;D9<;<4KNO4<@4::9E;PQ x;:;<;6675;:<?A;<A956;>9A<87>7E;DB<4<:9A9E95E9<?6;<?A;<@c;W9K:7W;8;P<^ _<896E75;EL:74<?6;<;<@c;W9<K:7W;8;<84<:9A9E95E9<K;:;<:9@9M9:?A;<A956;>9A<87>7E;D<;6675;8;Pa x;:;<;6675;:<?A;<A956;>9A<87>7E;DB<4<896E75;EL:74<?6;<?A;@c;W9<K[MD7@;Pg zA;<A956;>9A<87>7E;D<64A95E9<K489<69:<;6675;8;<K9D4896E75;EL:74<8;<A96A;Pp R<@c;W9<K[MD7@;<V<?A;<@c;W9<C?9<K9:A7E9<;M:7:<E48;6<;6;6675;E?:;6<87>7E;76<846<?6?L:746<89<?A;<9AK:96;P10

Agnaldo
Nota
c
Agnaldo
Nota
c
Page 12: Exercicios Informatica CESPE 2010 250 Questoes

www.pciconcursos.com.br

11

Agnaldo
Nota
c
Agnaldo
Nota
E
Agnaldo
Nota
E
Agnaldo
Nota
E
Agnaldo
Nota
E
Agnaldo
Nota
E
Agnaldo
Nota
C
Agnaldo
Nota
E
Agnaldo
Nota
E
Page 13: Exercicios Informatica CESPE 2010 250 Questoes

���������������� ������������������������ ��������� �� ����� !"#$"%&' ('# )*+(,-"+&'# '$"!,.%'+,%# " $,.'&"# ('# #%#&"-,#/%+('0# " 1%+*23 4*56*" '# %&"+# 7*" #" #"6*"-89: ; #%#&"-, '$"!,.%'+,5 < *-, $5,&,)'!-,3 '* #"4,3 *-, "#$<.%"(" =,#" #'=!" , 7*,5 #>' "2".*&,('# '# $!'6!,-,# *#,('# "-*- .'-$*&,('!8 �5<- (%##'3 &!,(*? ,# &,!"),# !"7*%#%&,(,# $"5'*#*@!%' '* $'! $!'6!,-,# $,!, *-, 5%+6*,6"- 7*" '.'-$*&,('! .'-$!""+(,89A B=*+&* < *- #%#&"-, '$"!,.%'+,5 .'- =,#" "- 1%+*2("#"+C'5C%(' $,!, DEFGHEEIJ3 KGJIFELJ " #"!C%('!"#89M ; #%#&"-, '$"!,.%'+,5 /%+('0# < -,%# #"6*!' (' 7*" ' 1%+*23*-, C"? 7*" ' 1%+*2 $'##*% .N(%6' ,="!&' " ),.%5%&, ,'.'!!O+.%, (" C*5+"!,=%5%(,("#8 99 �# )*+PQ"# (' R!;))%." #>' #%-%5,!"# S# )*+PQ"# (' $,.'&";))%." (, T%.!'#')& +' 7*" #" !")"!" S "(%P>' (" &"2&'# "$5,+%5U,# "5"&!V+%.,#8 W'+#%("!,+(' , )%6*!, ,.%-,3 7*" -'#&!, *-, &"5, (' X+&"!+"&Y2$5'!"! Z8[3 '# .'+."%&'# =@#%.'# " '# -'('# (" *&%5%?,P>' ("&".+'5'6%,#3 )"!!,-"+&,#3 ,$5%.,&%C'# " $!'."(%-"+&'# ,##'.%,('# SX+&"!+"& " \DF]DGF3 4*56*" '# %&"+# , #"6*%!89_ �' #" .5%.,! , '$P>' 3 #"!@ ,.%'+,(' *- #%#&"-, ("#"6*!,+P, 7*" 6,!,+&" , $!'&"P>' .'+&!, C!*# " F]EaDJ8 9b �' .5%.,! , '$P>' 3 ' *#*@!%' &"- ,."##' , *-,)"!!,-"+&, (" =*#., 7*" $"!-%&" 5'.,5%?,!3 $'! "2"-$5'3%-,6"+# !"5,.%'+,(,# S $,5,C!, �cYY18 9d �' #" .5%.,! , )"!!,-"+&, 3 #"!@ )"%&, , ("#.'+"2>' (,X+&"!+"&8 9e f,!, #" "+C%,! , $@6%+, ,$!"#"+&,(, $'! Ggh\i3 < #*)%.%"+&".5%.,! , )"!!,-"+&, 3 (%6%&,! ' Ggh\i (' ("#&%+,&@!%' ".5%.,! , '$P>' jklmno8 9p �' #" .5%.,! , )"!!,-"+&, 3 ,# $@6%+,# (, �cYY1 " ('q''65" #"!>' "2%=%(,# "- -%+%,&*!,8 c' 7*" #" !")"!" ,'# $'("!"# ,(-%+%#&!,&%C'# " ,'# $!%+.$%'# 7*"!"6"- , ,(-%+%#&!,P>' $r=5%.,3 4*56*" '# %&"+# #*=#"7*"+&"#89s t" ,.'!(' .'- ' $!%+.$%' (, 5"6,5%(,("3 , ,(-%+%#&!,P>'$r=5%., #'-"+&" $'(" ),?"! ' 7*" , 5"% 5U" $"!-%&"8_: W'-' (".'!!O+.%, (, !"5,P>' U%"!@!7*%., $!"#"+&" +' u-=%&'(, ,(-%+%#&!,P>' $r=5%.,3 *- N!6>' (" U%"!,!7*%, #*$"!%'!$'(" ,C'.,! ,&!%=*%PQ"# (" *- N!6>' #*='!(%+,('3 ("#(" 7*""#&,# +>' #"4,- (" .'-$"&O+.%, "2.5*#%C,8v*56*" '# $!N2%-'# %&"+#3 , !"#$"%&' (' #"!C%P' $r=5%.' " ('.'+&!,&' ,(-%+%#&!,&%C'8_A ; $!%+.$%' (, .'+&%+*%(,(" (' #"!C%P' $r=5%.'3 #"6*+(' '7*,5 ' #"!C%P' $r=5%.' +>' $'(" #"! %+&"!!'-$%('3 < ,$5%.@C"5,' "2"!..%' (, )*+P>' $r=5%.,3 -,# +>' ,'# .'+&!,&'#,(-%+%#&!,&%C'#8_M ;# #"!C%P'# $!"#&,('# $'! $"##',# 4*!(%.,# "- !"6%-" (".'+."##>' '* $"!-%##>'3 ,%+(, 7*" $,!, #,&%#),?"! ,#+"."##%(,("# .'5"&%C,#3 +>' #>' .'+#%("!,('# #"!C%P'# $r=5%.'#34@ 7*" +>' #>' $!"#&,('# (%!"&,-"+&" $"5' Y#&,('8W'- !"5,P>' ,' %+#&%&*&' (, 5%.%&,P>' " ,'# ,&'# ,(-%+%#&!,&%C'#34*56*" '# %&"+# 7*" #" #"6*"-8_9 ; $!"6>' .'+#&%&*% -'(,5%(,(" (" 5%.%&,P>' $,!, ,7*%#%P>' ("#"!C%P'# .'-*+#3 7*,57*"! 7*" #"4, ' C,5'! "#&%-,(' (,.'+&!,&,P>'8__ � !"C'6,P>' (' ,&' ,(-%+%#&!,&%C'3 7*" %-$5%., "2&%+P>' (" *-,&' C@5%('3 $!'(*? ")"%&'# !"&!',&%C'#8v*56*" ' $!N2%-' %&"-3 !"5,&%C' ,' $!'."##' ,(-%+%#&!,&%C'8_b ; $!%+.$%' (, '="(%O+.%, S )'!-, " ,'# $!'."(%-"+&'# &"-,$5%.,P>' ,=#'5*&, +' $!'."##' ,(-%+%#&!,&%C'3 !,?>' $"5, 7*,5'# ,&'# (' $!'."##' ,(-%+%#&!,&%C' #"-$!" ("$"+("!>' (")'!-, ("&"!-%+,(,8� !"#$"%&' ('# $!%+.$%'# )*+(,-"+&,%#3 ('# (%!"%&'# " (,# 6,!,+&%,#)*+(,-"+&,%#3 4*56*" '# %&"+# , #"6*%!8_d ;# C,5'!"# #'.%,%# (' &!,=,5U' " (, 5%C!" %+%.%,&%C,3 ,.'+#&!*P>' (" *-, #'.%"(,(" 5%C!" 4*#&, " #'5%(@!%, " , 6,!,+&%,(' ("#"+C'5C%-"+&' +,.%'+,5 .'+#&%&*"- )*+(,-"+&'# (,w"$r=5%., x"("!,&%C, (' R!,#%58_e t" ,.'!(' .'- , W'+#&%&*%P>' x"("!,5 (" yzZZ {Wx|3 +,U%$N&"#" (" %-%+"+&" $"!%6' $r=5%.'3 , ,*&'!%(,(" .'-$"&"+&"$'(" *#,! $!'$!%"(,(" $,!&%.*5,!3 ,##"6*!,+('}#" ,'$!'$!%"&@!%'3 +' .,#' (" (,+'3 , %+("+%?,P>' *5&"!%'!8� !"#$"%&' (, '!6,+%?,P>' $'5&%.'},(-%+%#&!,&%C, (' Y#&,('=!,#%5"%!' " (, ,(-%+%#&!,P>' $r=5%.,3 4*56*" '# %&"+# #"6*%+&"#8 _p � Wx ,(-%&" , %+.'!$'!,P>'3 , #*=(%C%#>' '* '("#-"-=!,-"+&' (" "#&,('#8_s f,!, ")"%&'# (" !"-*+"!,P>' (" $"##',5 (' #"!C%P' $r=5%.'3 ,Wx $"!-%&" , C%+.*5,P>' '* "7*%$,!,P>' (" 7*,%#7*"! "#$<.%"#!"-*+"!,&N!%,#8v*56*" ' %&"- ,=,%2'3 !"5,&%C' ,' f'("! Y2".*&%C'8b: ; $!"#%("+&" (, w"$r=5%., +>' (%#$Q" (" .'-$"&O+.%,.'+#&%&*.%'+,5 $,!, .'+."("! %+(*5&'3 $'! #" &!,&,! (".'-$"&O+.%, "2.5*#%C, (' f'("! v*(%.%@!%'812

marcusbelfort
Rectangle
Agnaldo
Nota
C
Agnaldo
Nota
C
Agnaldo
Nota
E
Agnaldo
Nota
C
Agnaldo
Nota
E
Agnaldo
Nota
C
Agnaldo
Nota
E
Agnaldo
Nota
E
Agnaldo
Nota
C
Page 14: Exercicios Informatica CESPE 2010 250 Questoes

��������������� ����������������������������������������������� �!��"�������#��� ����$%��� ��&'�������"��� ��&() �*�+�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a9E9D<;09-/83D/a9E3D/1/?E10:93F?./6383/C./8-D/J07R9.-D/9<1EDA/;/3J01D1E<383/C.3/D9<C345-T9J-<;<963A/D1:C983/81/C.3/3DD10<9L3/3/D10/BC2:383/6-./012345-/DJ1E3298381D/J01L9D<3D/E3/@19/bFccdecffgF[G h-5-A/D10L98-0/JOP296-/83/=>??@A/<1L1/DC3/81.9DD5-9EL3298383/J-0/8169D5-/38.9E9D<03<9L3F/>1DD3/D9<C345-A/h-5-81L10S/D10/019E<1:038-/3-/630:-/3E<109-0.1E<1/-6CJ38-A1D<3E8-/DC3/3J-D1E<38-093/3C<-.3<963.1E<1/DCB19<3/3/63DD345-F[[ i3C2-A/1./UCE45-/83/019E<1:0345-/81/C./6-21:3A/D10S016-E8CX98-/3-/630:-/MC1/3E<109-0.1E<1/-6CJ3L3A/63P1E8-W2T181L-2L10/3-/10S09-/-D/1.-2C.1E<-D/J1061P98-D/E-/J10Z-8-F>1DD3/D9<C345-A/63D-/i3C2-/E5-/U343/3/81L-2C45-/8-D/01U1098-D1.-2C.1E<-D/E-/J03X-/81/E-L1E<3/893DA/121/1D<30S/DCB19<-/DCDJ1ED5-/1/3-/J3:3.1E<-/81/.C2<3/89S093FjC3E<-/3-D/3<-D/81/9.J0-P98381/38.9E9D<03<9L3A/BC2:C1/-D/9<1ED/3D1:C90F[k ,-ED<9<C9/1E09MC169.1E<-/92Z69<-/-/3<-/81/C./3:1E<1/JOP296-0161P10/J303/D1C/U92T-/C./3C<-.7L12/X10-/MC92l.1<0-/6-.-J01D1E<1/81/C./1.J01DS09-/MC1/<1ET3/<98-/9E<101DD1/8901<-3.J3038-/J-0/-.9DD5-/816-001E<1/83D/3<09PC94N1D/81DD1/3:1E<1JOP296-/6-.-/D10L98-0/JOP296-F[m ,-ED98101/MC1/i180-/<1ET3/81ECE6938-/-/3:1E<1/JOP296-/h-5-J-0/3<-/81/9.J0-P98381A/D3P1E8-/MC1/1D<1/103/9E-61E<1F/>1DD163D-A/i180-/J10810S/3C<-.3<963.1E<1/DC3/UCE45-/JOP2963/1/<10SD1CD/89019<-D/J-2Z<96-D/DCDJ1ED-DA/32;./81/D10/6-E81E38-//J1E381/0162CD5-/1/3-/J3:3.1E<-/81/.C2<3F =/01DJ19<-/81/;<963/1/.-032A/BC2:C1/-/9<1./3P39R-F[n =/;<963/E-/D10L94-/JOP296-/89X/01DJ19<-/D-.1E<1/D/01234N1D/816-.3E8-/1/-P189KE693A/1EMC3E<-/3/.-032/3J2963WD1/D/01234N1D81/6-.J0-.9DD-/1/01DJ19<-/3-D/J09E6ZJ9-D/1/L32-01DF,-./012345-/3-D/3.P91E<1D/1/D/U1003.1E<3D/o9E8-pD/1/@9ECRABC2:C1/-D/9<1ED/3/D1:C90F[H ?./3.P91E<1/o9E8-pDA/-/369-E3.1E<-/D9.C2<qE1-/83D/<1623Dr A/s /1/t/J10.9<1A/1E<01/-C<03D/34N1DA/U9E329X30C.3/<301U3/J1E81E<1/E-/6-.JC<38-0A/J-0/1R1.J2-A/1E610030/C.3J2963<9L-/MC1/E5-/1D<1B3/01DJ-E81E8-A/6-.-/-/o-08/-C/-uE<10E1</?RJ2-010F[Q i303/D1/01329X30/3/<0-63/81/CDCS09-/E-/o9E8-pDA/D1./81D29:30-/6-.JC<38-0A/81L1WD1/D12169-E30/3/-J45-/vwxyz{|}~}��A1E6-E<0383/E-/����/�����wzA/3/U9./81/1E610030/3/D1DD5-/3<C32/81<03P32T-/1/819R30/-/6-.JC<38-0/89DJ-EZL12/J303/-C<0-/CDCS09-F[V =/DCZ<1/81/3J2963<9L-D/8-/@9ECR/�<30IUU961A/89DJ-EZL12:03<C9<3.1E<1/E3/uE<10E1<A/J10.9<1/3/18945-/81/8-6C.1E<-D/6-.89U101E<1D/J380N1DA/-D/MC39D/J-81./D10/3P10<-D/J303.-89U9634N1D/1./3J2963<9L-D/o9E8-pDF[Y >-/a960-D-U</o-08A/J303/D1/3P090/C./8-6C.1E<-/189<38-/E-IJ1EIUU961A/;/E161DDS09-/:1030/C./30MC9L-/E-/U-0.3<-/F-8</1AJ-0/.19-/83/-J45-/���}z�wz{�}���y��}A/1E6-E<0383/E-/�����z����}{�}{�}z�A/6-EL10<10/-/01U1098-/30MC9L-/J303/-/U-0.3<-F8-6Fk\ ID/LZ0CD/81/.360-/MC1/83E9U963./8-6C.1E<-D/8-/o-08/J-81.D10/129.9E38-D/6-./3/9ED<32345-/8-/3J2963<9L-/�9DC32/�3D96/U-0=JJ2963<9-EDA/8-/o9E8-pDFhC2:C1/-D/9<1ED/DCPD1MC1E<1DA/3/01DJ19<-/81/6-E619<-D/1/U1003.1E<3D81/uE<10E1<FkG =/D9E<3R1/81/1E81014-D/81/6-0019-/121<0lE96-/E3/uE<10E1</<1./-D1:C9E<1/J3805-�/E-.1/8-/CDCS09-A/D1:C98-/8-/DZ.P-2-/�/1A/3D1:C90A/-/8-.ZE9-/-C/����A/MC1/;/-/1E81014-/8-/D10L98-0/MC1DCJ-0<3/-/D10L94-/81/������Fk[ =/29E:C3:1./��a@A/C<929X383/J303/60930/8-6C.1E<-DT9J10<1R<-DA/J-81/D10/9E<10J01<383/J-0/MC32MC10/E3L1:38-0/1E5-/J0169D3/D10/6-.J92383Fkk  �i/;/C./J0-<-6-2-/81/6-.CE96345-/MC1/J10.9<1/-/1EL9-/8130MC9L-D/3E1R-D/3/.1ED3:1ED/81/6-0019-/121<0lE96-A/D1./3E161DD98381/81/6-.J36<30/1DD1D/30MC9L-DFkm ¡������¢£;/C./<9J-/81/3<3MC1/E3/uE<10E1</MC1/<1E<3/9E8CX90A/J-0.19-/81/.1ED3:1ED/81/������/-C/DZ<9-D/.32969-D-DA/-D/CDCS09-D3/9EU-0.301./838-D/J1DD-39D/-C/6-EU981E6939DF>-/MC1/D1/01U101/3-D/J-8101D/38.9E9D<03<9L-D/1/3-D/J09E6ZJ9-D/MC101:1./3/38.9E9D<0345-/JOP2963A/BC2:C1/-D/9<1ED/DCPD1MC1E<1DFkn I/J09E6ZJ9-/83/.-03298381/38.9E9D<03<9L3/<1./1R9D<KE6933C<lE-.3/E-/-081E3.1E<-/BC0Z896-/E369-E32/1/81L1/D10-PD10L38-/E5-/D-.1E<1/J12-/38.9E9D<038-0/JOP296-A/6-.-<3.P;./J12-/J30<96C230/MC1/D1/012369-E3/6-./3/38.9E9D<0345-JOP2963FkH ,-./UCE83.1E<-/E-/J-810/89D69J29E30A/3/38.9E9D<0345-JOP2963A/3-/<10/6-ET169.1E<-/81/J0S<963/81/U32<3/J-0/D10L98-0JOP296-A/J-81/1D6-2T10/1E<01/3/9ED<3C0345-/-C/E5-/81J0-6189.1E<-/81D<9E38-/3/J0-.-L10/3/6-001DJ-E81E<1/3JC0345-83/9EU0345-F13

marcusbelfort
Rectangle
marcusbelfort
Rectangle
marcusbelfort
Rectangle
Agnaldo
Nota
C
Agnaldo
Nota
C
Agnaldo
Nota
E
Agnaldo
Nota
E
Agnaldo
Nota
E
Agnaldo
Nota
C
Agnaldo
Nota
C
Agnaldo
Nota
E
Agnaldo
Nota
C
Page 15: Exercicios Informatica CESPE 2010 250 Questoes

UnB/CESPE – CAIXA/NM1

Cargo: Técnico Bancário Novo Caderno ALFA – 12 –

QUESTÃO 50

Acerca dos modos de utilização de aplicativos do ambienteMicrosoft Office, assinale a opção correta.

A A barra de ferramentas de formatação dos aplicativos da suíteOffice permite que seja aplicada a determinado conteúdo umasérie de ações, tais como salvar, copiar, colar e apagar.

B O recurso denominado Selecionar tudo, acionado a partir doconjunto das teclas � + �, permite que, com o apoio domouse, o usuário selecione trechos arbitrários de documentos,planilhas ou apresentações editados, respectivamente, noWord, no Excel e no PowerPoint.

C Ao ser inserida em um documento editado no Word, umaplanilha criada utilizando-se o Excel é convertida em tabela,que deixa de ter vínculo com a planilha original e, por isso,alterações na tabela não afetam o conteúdo da planilha, nemvice-versa.

D No Excel, o recurso de mesclar células de uma planilhapermite criar uma célula de planilha a partir de células vizinhasselecionadas.

E Ao ser inserida em um documento editado no Word, umafigura deve ser dimensionada no tamanho desejado, pois,uma vez inserida no documento, ela não poderá serredimensionada.

QUESTÃO 51

Acerca dos aplicativos da suíte BR Office, assinale a opção correta.

A A opção de exportar um documento editado no Writer comoPDF (portable document format) permite que esse documentoseja visualizado utilizando-se aplicativo não pertencente à suíteBR Office.

B No Writer, a opção que permite salvar o conteúdo de umdocumento como mensagem de e-mail só estará ativada se ocomputador no qual o aplicativo estiver sendo executadopossuir acesso à Internet.

C O Calc é um aplicativo da suíte BR Office que permite, noambiente Linux, a realização de operações matemáticassimples, como adição e subtração, e é equivalente ao aplicativoCalculadora do Windows.

D No Impress, a opção de duplicar slide permite que seja inseridoum novo slide em branco imediatamente após o slide emedição.

E As réguas existentes para edição de texto, acessíveis pelo menuExibir, não podem ser ocultadas na suíte BR Office, de forma agarantir que o espaçamento entre caracteres e a disposiçãoespacial das páginas sejam visíveis.

QUESTÃO 52

Com relação ao Windows, assinale a opção correta.

A Por meio do menu Iniciar do Windows XP, tem-se acesso àopção Executar, que permite abrir arquivos editados emaplicativos da suíte Microsoft Office, sem que os respectivosaplicativos sejam previamente executados.

B A opção Definir acesso e padrões do programa, que pode seracessada a partir do menu Iniciar do Windows, permite quesejam especificados programas-padrão a serem utilizados pararealizar atividades como navegação na Internet e envio demensagens de e-mail, entre outras.

C No Windows, fazer logoff de usuário significa excluir dosistema um usuário cadastrado e seus arquivos armazenados,além de tornar sua senha inválida.

D Por meio da janela Meu Computador, é possível acessar odiretório Arquivos de programas, que consiste em um diretórioespecial do Windows para instalação e remoção de programas.Para a remoção correta de determinado programa, é suficienteexcluir desse diretório o programa desejado.

E O Windows disponibiliza ao usuário uma lista de documentosrecentes, que pode ser utilizada para acesso rápido a arquivos,desde que estes arquivos tenham sido salvos recentemente nocomputador em uso.

QUESTÃO 53

Acerca do sistema operacional Linux, assinale a opção correta.

A No Linux, pode-se definir um caminho de diretórios a partir douso de barras invertidas (\), diferentemente do Windows, emque são utilizadas barras não invertidas (/).

B O Linux disponibiliza, na barra de inicialização rápida, recursopara ligar ou desligar o computador com maior velocidade,empregando o conceito de boot parcial da máquina.

C O Linux tem a desvantagem, com relação ao Windows, de sermais vulnerável a vírus de computador, que se propagam comrapidez nesse ambiente, tornando os aplicativos lentos einfectando os arquivos.

D Em ambiente operacional, o gerenciador de arquivos éutilizado para se visualizar a estrutura de diretórios erespectivos arquivos. No Linux, o Konqueror constituiexemplo de gerenciador de arquivos.

E O diretório raiz do Linux é o C:\.

www.pciconcursos.com.br

14

Agnaldo
Nota
C
Agnaldo
Nota
C
Agnaldo
Nota
C
Agnaldo
Nota
C
Page 16: Exercicios Informatica CESPE 2010 250 Questoes

UnB/CESPE – CAIXA/NM1

Cargo: Técnico Bancário Novo Caderno ALFA – 13 –

QUESTÃO 54

Com relação a conceitos de Internet e intranet, assinale a opçãocorreta.

A O acesso ao que se denomina intranet deve ser feito pormeio de uma rede local, não sendo possível esse acesso apartir de um computador conectado à Internet, garantindo-se, assim, segurança.

B A Internet e as intranets diferenciam-se pelos tipos deprotocolos de comunicação utilizados: a Internet éembasada no protocolo TCP/IP e as intranets, no protocoloTelnet.

C Na Internet, o protocolo de comunicação padrão paraacesso ao serviço de correio eletrônico é o http.

D Os mecanismos de busca atualmente utilizados na Internet,como os utilizados pelo Google, por exemplo, permitem oacesso a páginas de intranets de empresas.

E O acesso à Internet por meio de redes ADSL, queempregam a linha telefônica e modems como recursostecnológicos para a transmissão de dados, é possível epermite obter taxas de transmissão superiores a 10 Mbps.

QUESTÃO 55

Com relação aos aplicativos para acesso à Internet, assinale aopção que apresenta apenas navegadores web.

A Netscape Navigator, Internet Explorer, Mozzila Firefox,Opera

B Thunderbird, Netscape Navigator, Internet Explorer,Outlook

C Opera, Internet Explorer, Painel de Controle, MozzilaFirefox

D Outlook Express, Internet Explorer, Netscape Navigator,Internet Explorer

E Windows Explorer, Internet Explorer, Thunderbird,Mozzila Firefox, Outlook

QUESTÃO 56

Com relação à Internet, assinale a opção correta.

A O cliente de e-mail consiste em um programa que permite acessoà caixa postal do usuário de correio eletrônico; para essaatividade, dispensa-se o servidor.

B Uma VPN é uma rede virtual privada utilizada como alternativasegura para usuários que não desejam utilizar a Internet.

C VoIP é uma tecnologia atualmente promissora que, ao otimizar ouso da linha telefônica residencial ou empresarial, permite arealização de ligações telefônicas em tempo real e com baixocusto.

D A Internet emprega o modelo de comunicação cliente-servidor.E Denomina-se domínio da Internet o servidor que contém as

informações que se deseja acessar para diversas finalidades,tais como correio eletrônico, transferência de arquivos, acesso àWeb etc.

QUESTÃO 57

Acerca dos conceitos de organização de informações, assinale a opçãocorreta.

A Para o armazenamento de arquivos, o Windows possui estruturade diretórios rígida, sendo desnecessário e impossível ao usuáriocriar diretórios próprios.

B No Linux, arquivos com terminações diferentes, indicando teremsido gerados por programas diferentes, devem ser armazenadosem diretórios específicos e distintos.

C Uma das formas para otimizar o uso do espaço em disco é acompactação de arquivos, que pode ser feita por meio deprogramas específicos para tal finalidade, tais como o WinZip eo RAR.

D No Linux, os arquivos são armazenados com prazo de validade.Ao se encerrar o prazo de armazenamento demandado, caso nãoseja feito o backup, o arquivo é excluído do sistema dearmazenamento.

E O Linux não permite a criação de mais de cinco subdiretóriosdentro de um diretório raiz, nem um caminho superior a cincodiretórios.

www.pciconcursos.com.br

15

Agnaldo
Nota
C
Agnaldo
Nota
C
Agnaldo
Nota
C
Page 17: Exercicios Informatica CESPE 2010 250 Questoes

UnB/CESPE – CAIXA/NM1

Cargo: Técnico Bancário Novo Caderno ALFA – 14 –

QUESTÃO 58

Tendo como base a janela ilustrada acima, assinale a opção correta acerca do Windows Explorer.

A Denomina-se desktop o diretório raiz do Windows, no qual estão armazenados todos os outros diretórios do sistema operacional.B Em , estão listados os sítios da Internet recentemente acessados pelo usuário.C Para se alterar o modo de exibição da lista de arquivos e diretórios exibidos na janela, deve-se acionar o botão .D O botão é utilizado para se abrir, com o mouse, um diretório selecionado pelo usuário. Esse procedimento para se abrir um

diretório pode também ser realizado por meio de clique duplo.E A barra de endereço permite tanto navegar pela estrutura de diretórios quanto acessar a Web ou outros serviços de Internet, tais como

o FTP.qQUESTÃO 59

Com relação a certificação e assinatura digital, assinale a opçãocorreta.

A A assinatura digital facilita a identificação de umacomunicação, pois baseia-se em criptografia simétrica de umaúnica chave.

B Quando um usuário com assinatura digital envia e-mail paraum destinatário, a mensagem será assinada por uma chavepública do destinatário, para garantir que seja aberta apenaspelo destinatário.

C No Windows, não é necessário utilizar assinatura digital, poistodos os aplicativos, principalmente os navegadores web,utilizam essa tecnologia de maneira automática e transparenteao usuário.

D Uma desvantagem dos aplicativos da suíte BR Office, emrelação aos da suíte Microsoft Office, é não possuir suporte aassinaturas digitais nem exibir certificados digitais criados paraos usuários.

E O destinatário de uma mensagem assinada utiliza a chavepública do remetente para garantir que essa mensagem tenhasido enviada pelo próprio remetente.

QUESTÃO 60

Acerca de certificação digital, assinale a opção correta.

A A infraestrutura de chaves públicas é uma rede privada quegarante que seus usuários possuem login e senha pessoais eintransferíveis.

B Uma autoridade de registro emite o par de chaves do usuárioque podem ser utilizadas tanto para criptografia como paraassinatura de mensagens eletrônicas.

C A autoridade certificadora raiz emite certificados para usuáriosde mais alto nível de sigilo em uma organização com umachave de criptografia de 128 bits.

D A autoridade de registro recebe as solicitações de certificadosdos usuários e as envia à autoridade certificadora que os emite.

E O uso de certificado digital garante o repúdio de comunicaçõesoriundas de usuários ou sítios que possuem certificados válidose emitidos por entidades confiáveis.

www.pciconcursos.com.br

16

Agnaldo
Nota
C
Agnaldo
Nota
C
Agnaldo
Nota
C
Page 18: Exercicios Informatica CESPE 2010 250 Questoes

������������������ ������ ���������������������� �!��"�#$��%�� �&������ ���'(��� � �%�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a6R,/,bcdefg+68[,./3:7950-1/,H-0-,H+66/-6,1+680-6R,+_H0+66Y+6./2/,h>ih@D;,h>ijkCAEilF>CEA+,h>ijkCABkF>=A0+:+0+26+,-,.M745+6,./2,/,m/8\/,+645+01/,1/,bcdefR,GCh>@DA./00+6H/31+,n,/H+0-X\/,1+H0+667/3-0,52-,8+.M-,+;,0-H71-2+38+;,M7m+0[M-;,-.7/3-31/-,-H+3-6,52-,W+oU,p/3671+0+,8-2mq2,45+,3\/,[,0+6807XY+6,1+,H0/8+X\/;,1+:53.7/3-2+38/,+,1+,56/,+2,0+M-X\/,-/6,H0/90-2-6;,-0457W/6;,170+8J07/6;,0+.506/6,+,+457H-2+38/6,2+3.7/3-1/6Urstuvrwxvtysz{|v}~wz�=D�@iE�Al@�h?CGCEACA��lCD=E�AEC�@l=EAEClFDCAklAF=kh=ABCEh=�<i@B=E���,�/03-76,�,72H0+66/6,/5,+M+80�37./6,�,./6852-2;� H+07/17.-2+38+;,�0+80-8-0�,/,1+6+2H+3/,1-,+./3/27-,+,1/9/W+03/U,V-0-,766/;,H0+.76-2,80-m-M-0,./2,2+171-6;7317.-1/0+6,45-3878-87W/6U,p/3851/;,/6,7317.-1/0+6,45+� ./36805�2/6,�3J6,/5,+M+6�,3\/,6\/,3+580/6;,3\/,6\/731+H+31+38+6,1/,M59-0,+2,45+,+68-2/6;,3-,6/.7+1-1+U,�,3\/6\/,731+H+31+38+6,1/,052/,45+,H0+8+31+2/6,1-0,n,6/.7+1-1+� �,/,3/66/,H0/L+8/;,-,3/66-,71+/M/97-U,�-o,8+2H/,45+,+68-W-,-2-150+.+31/,-,H/Ma27.-,6/m0++66+6,7317.-1/0+6,�,.0+6.72+38/,+./3�27./,+,+2H0+9/U,�[�� H/5./6,2+6+6;,3/,.-1+03/,1+,+./3/27-,1/,L/03-M,QANEG@B=ABC���@k>=;,52,-0879/,1+,./M53768-,/:7.7-M,1/,L/03-M;,m-68-38+./36+0W-1/0-;,.0787.-W-,/,1+6+2H+3/,1-,+./3/27-,+,1/�� 9/W+03/;,-:702-31/;,H/0,+_+2HM/;,45+,-,+./3/27-,7-,2-M,+,/1+6+2H0+9/,.0+6.7-U,]-,2+62-,+17X\/;,2-6,+2,/580/,.-1+03/;52,0/2-3.768-,+,./M53768-,1+,L/03-76,+,0+W768-6;,�/\/,�m-M1/�� �7m+70/;,17o7-,-M9/,6727M-0I,3-1-,.0+6.7-,3/,�0-67M;,-,3\/,6+0,/672H/68/6U,�,-66/2m0/6/,q,45+,-,H073.7H-M,2-8q07-,1+66+.-1+03/,1+,+./3/27-;,3+66-,2+62-,+17X\/,1/,NEG@B�=;,./2�� 90-31+,1+68-45+;,+0-,-M9/,-6672I,p0+6.+,/,+2H0+9/,�+,+2H0+9/:/02-M�;,+,-,2-66-,6-M-07-M,-52+38-,3-,W+M/.71-1+,1+,� ,m7,H/02a6U¡¡ �,M+78/0,1+6./3:7-1/,H+09538-07-I,/,45+,-./38+.+5¢�6,./M53768-6,3\/,M++2,/,H0JH07/,L/03-M¢,]\/;,3\/,q,766/U£,./M53768-,3\/,+68-W-,H0/15o731/,73:/02-X\/;,+68-W-¡¤ H0/15o731/,52-,738+0W+3X\/,3/,1+m-8+;,52-,738+0W+3X\/2/87W-1-;,52-,73:/02-X\/,738+0+66-1-;,2/87W-1-,H+M-,65-H/67X\/,H/M�87.-U,�,/,0/2-3.768-,+68-W-,6+,0+:+0731/;¡¥ 72HM7.78-2+38+;,n,¦+171-,V0/W76J07-,�¦V�,3§,�;,45+-52+38-07-,72H/68/6,H-0-,-6,8-76,�+2H0+6-6,H0+68-1/0-6,1+6+0W7X/6�;,3-6,45-76;,H0/W-W+M2+38+;,+M+,707-,6+0,+345-10-1/U�� ]-1-,2-76,3-850-M,45+,+6H+03+7+2,+,45+,W+L-2,/,2531/,1+/580-,2-3+70-U,©,52-,H+0.+HX\/,1/,2531/;,2-0.-1-,H+M/,M59-0+2,45+,+M+6,+68\/,+,H+M/6,:-8/6,45+,H+0./00+2,65-,+_768a3.7-�� 17[07-U£,¦V,3§,�;,8\/,6500-1-,H+M-,2�17-;,172735�-,/72H/68/,1+,0+31-,�ª��,H-0-,/6,-66-M-07-1/6,+,H+367/3768-6,+;�� H0/H/0.7/3-M2+38+;,-52+38-W-,/,72H/68/,1-6,8-76,+2H0+6-6U�66-6;,3/6,ZM872/6,-3/6;,873-2,671/,/,176H/6787W/,H+M/,45-M+2H0+6-6,+,:53.7/3[07/6,45-M7:7.-1/6,873-2,+2H0++3171/,-�� 8+0.+707o-X\/,1/,-66-M-07-1/;,./2,67937:7.-87W-,0+15X\/,1+.-09-,807m58[07-U,£M952,L/03-M,8/./5,3+66-,0+M-X\/¢,]\/,6+,:-M-1+,./01-,+2,.-6-,1+,+3:/0.-1/U�� «¬­®°±²³¬µ°¶·¬³·¬¹°º»»»µ¼®³®·®·­µ²³­µ½³¾°¿À²Á°Â¼ ÃĬŵ�5M95+,/6,78+36,1+,Æ,-,Ç;,-.+0.-,1-6,H0/H07+1-1+6,8+_85-76,+90-2-87.-76,1/,8+_8/,-.72-UÈ �,+2H0+9/,1-6,-6H-6,3-,M73-,,-L51-,-,./3680570,/,8/2,70�37./45+,/,-58/0,72H072+,-/,8+_8/,+,+68[,0+M-.7/3-1/,n,71+7-,.+380-MU�66-,0+M-X\/,H/1+,6+0,+_H0+66-,1-,6+95738+,:/02-I,65HY+6+,45+/6,0+80-8/6,6+L-2,.JH7-6,1-,0+-M71-1+;,2-6,/,-58/0,H0/.50-2/680-0,45+,-,0+-M71-1+,H/1+,6+0,1768/0.71-,H+M-,H/67X\/,45+,-6H+66/-6,/.5H-2,3-,6/.7+1-1+UÉ ]-,M73-,Ê;,6+,-,H-M-W0-,�8+2H/�,:/66+,65m68785�1-,H+M-+_H0+66\/,B=iEA@�=E;,-,:/02-,W+0m-M,�:-o�,1+W+07-,6+065m68785�1-,H/0,<@ËClU Ì ]/,80+./,��,-66/2m0/6/,q,45+,-,H073.7H-M,2-8q07-,1+66+.-1+03/,1+,+./3/27-�,�ÍUÎÏÎÊ�;,-,H-M-W0-,�-66/2m0/6/�,+68[+2H0+9-1-,./2/,52,65m68-387W/;,/,45+,6+,./2H0/W-,H+M-H0+6+3X-,1/,-0879/,45+,-,-38+.+1+UÐ *+,-./01/,./2,/,-58/0;,-,./M53768-,1/,.-1+03/,1+,+./3/27--W7-,M71/,-6,2-8q07-6,+6.078-6,H+M/6,./M+9-6,1+,L/03-M,+,6+/H53-,-,+M+6;,2-6,+M+6,3\/,87W+0-2,-.+66/,n,/H737\/,1-,./M+9--38+6,1-,H5mM7.-X\/,+,1-,W+7.5M-X\/,1/,L/03-MUÑ �,./3+.87W/,45+,737.7-,/,80+./,��,/,0/2-3.768-,+68-W-,6+0+:+0731/�,�ÍUÏ�,8+2,W-M/0,+_HM7.-87W/,+,H/1+07-,6+0,65m68785�1/H/0,�=iE;,6+2,H0+L5�o/,H-0-,/6,6+3871/6,/07973-76,1/,8+_8/UÒ �,+2H0+9/,1/,-.+38/,90-W+,+2,��,/,0/2-3.768-,+68-W-,6+0+:+0731/;,72HM7.78-2+38+;,n,¦+171-,V0/W76J07-,�¦V�,3U§,���ÍUÏ ÊÓAL5687:7.-6+,H+M-,0+9a3.7-,1+,�0+:+0731/�,+,H+M/+2H0+9/,1+,-0879/,:+27373/,-38+.+1+31/,�¦+171-,V0/W76J07-�U�5M95+,/6,78+36,45+,6+,6+95+2;,-,0+6H+78/,1+,./3.+78/6,1+73:/02[87.-UÔ V-0-,-,./3+_\/,1+,52,./2H58-1/0,n,ª38+03+8;,q,3+.+66[07/,8+0;+380+,/580/6,./2H/3+38+6,1+,ÕÖ×ØÙÖ×f;,52-,HM-.-,1+,0+1+,/552,bcØfbU,£,HM-.-,q,56-1-,3/,.-6/,1+,/,./2H58-1/0H+08+3.+0,-,52-,0+1+,1+,./2537.-X\/;,+345-38/,45+,/,bcØfbq,3+.+66[07/,45-31/,-,./3+_\/,q,0+-M7o-1-,W7-,M73-,8+M+:�37.-UÚ ]/,-HM7.-87W/,�_.+M;,52,673-M,1+,.7:0\/,�Û�,1+W+,6+0,587M7o-1/72+17-8-2+38+,-38+6,1+,52-,0+:+0a3.7-,-m6/M58-,-,6+0,:7_-1-U�66+,H0/.+172+38/,+W78-,45+,-,0+:+0a3.7-,1-,.qM5M-,H/66-,6+0-M8+0-1-,-/,6+0,56-1-,52-,-MX-,1+,H0++3.72+38/,/5,./2-31/6;./2/,./H7-0,+,./M-0UÜ V/0,H-10\/;,/,6768+2-,/H+0-.7/3-M,Ý731/Þ6,176H/37m7M7o-;,H-0--,0+-M7o-X\/,1+,.[M.5M/6,672HM+6;,52-,.-M.5M-1/0-,.7+38�:7.-45+,/:+0+.+,/HXY+6,H0[87.-6,H-0-,2-37H5M-XY+6,1/6,0+65M8-1/61-6,/H+0-XY+6;,/6,45-76,H/1+2,6+0,6-MW/6,+2,-0457W/;,H-0-,56/H/0,/580/6,H0/90-2-6;,/5,736+071/6,+2,HM-37M-6,1/,�_.+MUÈß p/2,/,56/,1+,H0/90-2-6,-H0/H07-1/6;,q,H/66�W+M,7368-M-0,2-761+,52,6768+2-,/H+0-.7/3-M,+2,52,./2H58-1/0;,8-M,45+;,-/,6+737.7-0,52-,6+X\/,1+,80-m-M/;,H/1+6+,6+M+.7/3-0,45-M,6768+2-/H+0-.7/3-M,6+0[,587M7o-1/UÈÈ �,-.+66/,-,52,-0457W/,45+,+68+L-,-02-o+3-1/,+2,52,170+8J07//.5M8/,/5,M/.-M7o-1/,+2,52-,5371-1+,H+08+3.+38+,-,52-,0+1+1+,./2H58-1/0+6,H/1+,6+0,0+-M7o-1/,170+8-2+38+,H/0,2+7/,1+52,�./3+,1+,-8-M/,./3:7950-1/,3-,[0+-,1+,80-36:+0a3.7-U,]+66+.-6/;,/,-.+66/,-/,-0457W/,q,/m871/,./2,-,-HM7.-X\/,1+,52.M745+,15HM/,./2,/,m/8\/,170+78/,1/,bcdef,6/m0+,/0+:+071/,�./3+U17

marcusbelfort
Rectangle
marcusbelfort
Rectangle
Agnaldo
Nota
C
Agnaldo
Nota
C
Agnaldo
Nota
E
Agnaldo
Nota
E
Page 19: Exercicios Informatica CESPE 2010 250 Questoes

��������������� ������������������ ����������� !"#$ %&'()*+,-./0*1*23-&4-15&6/&789:;;<=>?8@AB>C8D>9E;FG:8HIDFJFA?8>K8JL>MI>N8J:K8>OJ>PQ:8D:;8D>9E;FG:;8HIDFJFAF;8>>OGCAHIDFJFAF;8RQ:8GCFSIGTCF:;8C>?AGF=:;8U8VRFQ:8>8:;8GCFSIGTCF:;8>8RQ:8GCFSIGTCF:;8C>?AGF=:;8A@IRD:;89WS?FJ:;N8AIGACMIFA;N8@IRDAPX>;89WS?FJA;8>8D>KAF;8>RGFDAD>;8@>D>CAF;8FRG>YCARG>;D:;8:CPAK>RG:;8@F;JAF;8>8DA8;>YICFDAD>8;:JFA?N8D>;D>8MI>8AG>RDA8A:;8;>YIFRG>;8C>MIF;FG:;Z888;>HA8@>FG:8R:8=A?:C8>OAG:8DA8YIFA8D>8D>9E;FG:[888;>HA8>KFGFD:89>?:8D>9:;FGARG>N89ACG>;8:I8C>9C>;>RGARG>8?>YA?[888;>HA8R:CKAGF=:8U8\]_]8:I8A:8ECYQ:8J:K9>RG>ababab cdefghijklmgjfnmnoipqlmnmrismtnuqknivqhwlmnqsmnxmgkvmnjfnyfljnz{{|nrfsnqsnjfrqskgwfnksnkji}~ftnmhhigmvknmnf�}~fnrfllkwmnmnlkh�kiwfnjkrfgrkiwfhnknsfjfhnjknqwivi�m}~fnjfhnm�virmwi�fhn�irlfhfown�ooirknkn��n�ooirk�� �njfrqskgwfnksnkji}~fn�fjknhklnrmllkpmjfnknmvwklmjfn�kvfnm�virmwi�fnyliwktnjfn��n�ooirktnjkhjknuqknmnokllmskgwmn����������nkhwkxmighwmvmjmngfnrfs�qwmjflnksnqhf�n� �flnskifnjknoqgrifgmvijmjkhnjih�fgi�ivi�mjmhnmfnhknrvirmlngmnhkwmnksn tn�n�fhh��kvnmvwklmlnfnwi�fnjknsmlrmjflnqhmjfngfjfrqskgwfnksnkji}~f�� �njfrqskgwfnksnkji}~fng~fnrfgw�sn�mvflkhngqs�lirfh�n�flwmgwftng~fn�fjknhklnwlmghoklijfn�mlmnqsmn�vmgiv�mnjfn��rkv�n� emhfnfnjfrqskgwfnksnkji}~fnhkxmnrf�imjfnknrfvmjfngfn�f�kl�figwtnhkl~fnrlimjfhnwl�hn��� ¡�¢nqsn�mlmnfnw�wqvfnjfnjfrqskgwf�nfqwlf�mlmnfn�ml£plmof�nknfnwklrkilfn�mlmnmhnwl�hnvig�mhnrfsnsmlrmjflkh�¤ �mlmnoflsmwmlnrfsnlkrqfn¥nkhuqkljmnfn�ml£plmofnigirimjfn�fln¦§n�fhh��kvtn�nhqoirikgwknhkvkrifg£©vfnktnmnhkpqiltnrvirmlnfn�fw~fn ������ !"#ª«rklrmnjfhnhihwksmhnf�klmrifgmihnyigjf�hnkn¬igq�tnmhhigmvknmnf�}~frfllkwm�� «luqi�fhnrlimjfhngfn¬igq�n�fjksnhklnvijfhngfnyigjf�hn�flskifnjmnokllmskgwmn­�®�°�±²�³�³®µ®��¶ tnk�ihwkgwkngfn·¡¹º�µ»µ�njfnyigjf�h�n� ¼fn¬igq�tnfn�lfplmsmn�vmgemvrn�klsiwknmnkvm�flm}~fnjk�vmgiv�mhnkvkwl½girmhtnjknoflsmnkuqi�mvkgwknmfn��rkvngfyigjf�h�� ¼fnhihwksmnyigjf�htnfnm�virmwi�fnyigjf�hn���vflklnwksnmoqg}~fnk�rvqhi�mnjknomriviwmlnfnpklkgrimskgwfnjmhnigoflsm}¾khksnqsnrfs�qwmjfltn�klsiwigjfnrlimltnk�rvqilnknlkgfskmlmluqi�fhnkn�mhwmh�nkguqmgwfnfn¿gwklgkwn���vflkln�nqsnÀÁÂÃ�¡Áuqkn�klsiwknmngm�kpm}~fngmn¿gwklgkw�� �flnhkln�ÂÄÅÃÆÁ¡nvi�lktnfnqhq£lifnwksnmnvi�kljmjknjknrf�imlnksfjioirmlnqsmnjihwli�qi}~fnjfn¬igq�tnhksnhfviriwmlnuqmvuqklnwi�fjkn�klsihh~f�¤ �flnskifnjmnf�}~fnǵ��È®³ÉÊË ��tn�n�fhh��kvnmxqhwmlnjmwmt�flmnknoqhfn�fl£lifnjfnrfs�qwmjfl�n ����� !"#Ì«nlkh�kiwfnjknrfgrkiwfhnknsfjfhnjknqwivi�m}~fnjknwkrgfvfpimhmhhfrimjmhn¥n¿gwklgkwnkn¥n�ÅÁÆ¡Åtnmhhigmvknmnf�}~fnrfllkwm�n� �ngm�kpmjfln¿gwklgkwn���vflkln�klsiwknom�kln ÂÃ�ÂÆ �Íjkmluqi�fhnknhmv�£©vfhnksn�mhwmhnknhq��mhwmhnjfnjihrfnl�pijfngfrfs�qwmjflnvfrmv�n� «nqwivi�m}~fnjfn�qwvffÎn���lkhhnwksnmn�mgwmpkstnksnlkvm}~fmfn�irlfhfown�qwvffÎtnjkng~fngkrkhhiwmlnjknrfgoipqlm}~f�l��imn�mlmnfnkg�ifnknlkrk�iskgwfnjknskghmpkghnjknrfllkifkvkwl½girf�� �mlmnhknom�kln¹Ï�ÂÆ njknqsnmluqi�fnmlsm�kgmjfngfnjihrfl�pijfnjfnrfs�qwmjfltn�ngkrkhh£lifnmgk�mlnfnmluqi�fnmnqsmskghmpksnjfnrvikgwknjkn¡Ð·Æ���n� Ñsnmluqi�fnuqknoflnlkrk�ijfnksnmgk�fnmnqsmnskghmpksnjkrfllkifnkvkwl½girfn�fjknhklnmlsm�kgmjfnksnqsmn�mhwmuqmvuqklnjfnjihrfnl�pijfnjfnrfs�qwmjfltnjkhjknuqknfngfskjfnlkoklijfnmluqi�fng~fnhkxmnmvwklmjf�¤ ¼mn�ÅÁÆ¡Åtnfhnmluqi�fhnh~fnmlsm�kgmjfhnksnhkl�ijflkhvfrmvi�mjfhnoihirmskgwkngmnks�lkhm�nkguqmgwftngmn¿gwklgkwtnfhmluqi�fhnh~fnmlsm�kgmjfhnksnhkl�ijflkhnk�wklgfh�nÒÒÒÓÔÕÖÕ×ØÕÙÚÛ×ÛÓÕ×ÜÓÝÚÞ Þ18

B
Agnaldo
Nota
C
Agnaldo
Nota
C
Agnaldo
Nota
C
Page 20: Exercicios Informatica CESPE 2010 250 Questoes

UnB/CESPE – MJ/DPF Cargo: Agente de Polícia Federal – 3 –

Com referência à redação de correspondências oficiais, julgue ositens a seguir.�� Documentos oficiais em forma de ofício, memorando, avisoe exposição de motivos têm em comum, entre outrascaracterísticas, a aposição da data de sua assinatura eemissão, que deve estar alinhada à direita, logo após aidentificação do documento com o tipo, o número doexpediente e a sigla do órgão que o emite.�� Desconsiderando-se as margens e os espaços adequados,respeitam as normas de redação de um documento oficialencaminhado por um chefe de seção a seu diretor o seguintetrecho, contendo o parágrafo final e fecho de um ofício.

(...)4. Por fim, por oportuno informamos que asprovidências tomadas, e aqui mencionadas,também já são do conhecimento das partesenvolvidas. Atenciosamente[assinatura]Pedro Álvares CabralChefe da seção de logísticae distribuição de pessoal (SLDP).Julgue os seguintes itens, acerca da Organização Internacional dePolícia Criminal (INTERPOL).�� Trata-se de organização subordinada ao secretário-geral daOrganização das Nações Unidas.�� A INTERPOL administra cinco presídios, um em cadacontinente, utilizados na guarda provisória de presosprocurados pelas polícias nacionais e no encarceramento decriminosos de guerra.� A autoridade legal da INTERPOL para realizar prisões estácondicionada à autorização prévia da polícia nacional.� A INTERPOL, que funciona como agência de ligação entreos organismos policiais dos países-membros, mantém basede dados que os auxilia no combate ao crime.�� Tal organização tem sede em Lyon, na França, e é chefiadapor um secretário-geral eleito por assembleia geral dospaíses-membros, para um mandato de cinco anos.Com relação à Usina Hidrelétrica de Itaipu e ao acordo firmadoentre Brasil e Paraguai, em julho de 2009, no qual são revistascláusulas do Tratado de Itaipu, julgue os itens que se seguem.�� Segundo o acordo, o Paraguai pode vender parte da energiagerada por Itaipu diretamente no mercado de energiabrasileiro. �� Localizada na fronteira brasileiro-paraguaia, Itaipu é a maiorusina hidrelétrica do mundo em capacidade de geração deenergia elétrica.�� Em contrapartida às concessões brasileiras, o Paraguaiautorizou no acordo a instalação, em seu território, dogasoduto Patagônia-São Paulo.�� Os diretores e os conselheiros de Itaipu são indicados, emigual número, pelo governo brasileiro e pelo paraguaio.Julgue os itens subsequentes, a respeito de Internet e intranet.�� As intranets, por serem redes com acesso restrito aosusuários de empresas, não utilizam os mesmos protocolos decomunicação usados na Internet, como o TCP/IP.�� Um cookie é um arquivo passível de ser armazenado nocomputador de um usuário, que pode conter informaçõesutilizáveis por um website quando este for acessado pelousuário. O usuário deve ser cuidadoso ao aceitar um cookie,já que os navegadores da Web não oferecem opções paraexcluí-lo.

A figura acima mostra a parte superior de uma janela do InternetExplorer 7 (IE7), em execução em um computador com sistemaoperacional Windows Vista, em que a página da Webhttp://www.google.com.br está sendo acessada. Com relação aessa janela, ao IE7 e a conceitos de Internet, julgue os itens quese seguem.

�� Ao se clicar o botão , a página que estiver sendo exibidano navegador passará a ser a página inicial do IE7 sempreque este navegador for aberto.

� O Google é um instrumento de busca que pode auxiliar aexecução de diversas atividades, como, por exemplo,pesquisas escolares.

Julgue os itens a seguir, acerca de hardware e de software usadosem computadores pessoais.� ROM é um tipo de memória não volátil, tal que os dados

nela armazenados não são apagados quando há falha deenergia ou quando a energia do computador é desligada.

�� Existem dispositivos do tipo pendrive que possuemcapacidade de armazenamento de dados superior a 1 bilhãode bytes. Esses dispositivos podem comunicar-se com ocomputador por meio de porta USB.

Considerando a figura acima, que mostra o canto inferioresquerdo do desktop do Windows XP, julgue os itens seguintes.

�� Ao se clicar, com o botão direito do mouse, o ícone , éexibida uma lista de opções, entre elas, a que permite iniciaração para esvaziar a Lixeira do Windows.

�� Ao se clicar uma vez sobre o ícone , a respectivapasta será aberta, e os ícones associados aos arquivos nelacontidos poderão ser visualizados.

www.pciconcursos.com.br

19

marcusbelfort
Rectangle
Agnaldo
Nota
E
Agnaldo
Nota
E
Agnaldo
Nota
E
Agnaldo
Nota
C
Agnaldo
Nota
C
Agnaldo
Nota
C
Agnaldo
Nota
C
Agnaldo
Nota
E
Page 21: Exercicios Informatica CESPE 2010 250 Questoes

UnB/CESPE – MJ/DPF Cargo: Agente de Polícia Federal – 4 –

Considerando a figura acima, que mostra uma janela do Word2002, com um texto em edição, em que nenhuma parte estáformatada como negrito, julgue os próximos itens.�� Ao se clicar à direita da palavra “devidamente” e, em

seguida, clicar o botão , o símbolo será exibido àdireita da referida palavra.

�� Ao se aplicar um clique duplo em um local da barra de títuloque não contenha botão ou ícone, a janela mostrada serámaximizada.

�� O conteúdo da primeira linha do texto mostrado serácentralizado, após a realização da seguinte sequência deações: selecionar a referida linha; pressionar e manterpressionada a tecla �; acionar a tecla �, pressionando-ae liberando-a; liberar a tecla �.

Julgue os itens a seguir, considerando a figura acima, que mostrauma planilha em edição no Excel 2002, com uma lista de preçosunitários de mesas e cadeiras, bem como a quantidade a seradquirida de cada item.�� Para se calcular o preço total das oito cadeiras a serem

adquiridas, é suficiente clicar a célula D3, digitar =B3*C3 e,em seguida, teclar �.

�� Para se inserir uma nova linha entre as linhas 1 e 2,movendo os conteúdos das linhas 2 e 3 para baixo, ésuficiente clicar no cabeçalho da linha 2 — — e, emseguida, clicar o botão .

Com relação a conceitos de Internet, julgue o item abaixo.� A sigla FTP designa um protocolo que pode ser usado para

a transferência de arquivos de dados na Internet.Julgue os itens a seguir, com base nos conceitos e normasaplicáveis à escrituração contábil.� Os livros diário e razão, por constituírem os registros

permanentes de uma entidade e por serem obrigatórios,devem ser registrados no registro público competente.

�� Em decorrência da aplicação do método das partidasdobradas, as contas retificadoras do patrimônio líquido têmseu saldo aumentado quando são debitadas, e diminuídoquando são creditadas.

�� O fato contábil registrado no lançamento de 4.ª fórmulaabaixo pode ser entendido como recebimento por uma vendaa prazo, após o vencimento da obrigação, e concessão deabatimento por problemas com a mercadoria ou sua entrega.

D bancosD abatimentosC clienteC juros

�� De acordo com a doutrina e a legislação contábeis, aprescrição de uma dívida e o perecimento de um direitocorrespondem, respectivamente, a uma realização de receitae à incorrência de uma despesa.

A respeito da composição e da avaliação do patrimônio, julgue ositens seguintes.�� Suponha que uma empresa mineradora tenha adquirido os

direitos de exploração de uma mina por R$ 5 milhões, pormeio de um contrato com cinco anos de vigência. Nessecaso, após dois anos de exploração, se tiverem sido extraídos30% da possança da mina, o referido ativo, classificado noimobilizado, deverá estar avaliado no balanço da empresapor R$ 3 milhões.

�� Constituem eventos que afetam o patrimônio líquido, comoacréscimo ou redução, ajustes de exercícios anteriores,resultado líquido do exercício e resultado na venda de açõesem tesouraria.

�� Suponha que o saldo da conta de resultado do exercício,antes do cálculo do imposto de renda devido, seja positivoem R$ 1,5 milhão e o valor desse imposto, de R$ 2 milhões.Nesse caso, o fato estará adequadamente representado peloseguinte lançamento em reais.

D resultado do exercício 1.500.000D reservas de lucros 500.000C provisão para imposto de renda 2.000.000

�� Na demonstração dos fluxos de caixa, classificam-se nofluxo de financiamento, entre outras operações, a colocaçãode debêntures, a distribuição de juros sobre o capital próprioe a integralização de capital em espécie.

www.pciconcursos.com.br

20

marcusbelfort
Rectangle
Agnaldo
Nota
E
Agnaldo
Nota
C
Agnaldo
Nota
E
Agnaldo
Nota
E
Page 22: Exercicios Informatica CESPE 2010 250 Questoes

UnB/CESPE – BRB

����������� �������������������������� �� ������

Texto II

nome científico: Ginkgo biloba L.nome popular: nogueira-do-japãoorigem: Extremo Orienteaspecto: as folhas dispõem-se em leque e são semelhantes ao trevo;a altura da árvore pode chegar a 40 metros; o fruto lembra umaameixa e contém uma noz que pode ser assada e comidaTexto III

Dá para confiar?Nas farmácias brasileiras, os comprimidos de extrato de

ginkgo vendidos só com receita médica competem com cápsulas depó moído e folhas, em embalagens expostas nas prateleiras aoalcance do consumidor. Muita gente relata efeitos benéficosadvindos dessas fórmulas alternativas. Mas seriam elas tão eficazesquanto os comprimidos vendidos com receita? A resposta é não.Pesquisadores da UFSC fizeram testes para saber quanto há decomponentes do extrato EGb 761 nessas cápsulas e nas folhas daplanta. Conclusão: para obter a mesma quantidade de um únicocomprimido de 120 mg, seriam necessárias 20 cápsulas de 200 mgde pó moído. Quanto ao chá, a eficácia depende da qualidade damatéria-prima. “Mas seria preciso ingerir grande quantidade, já queos teores das substâncias ativas no chá caseiro são baixos”, afirmaa autora do trabalho e pesquisadora da UFSC. Segundo ela, aproporção ideal só é obtida com os extratos secos padronizados.

Internet: <www.saude.abril.com.br> (com adaptações).

Acerca da interpretação e dos aspectos gramaticais dos textos II eIII e considerando as informações do texto I, julgue os itenssubsequentes.�� Os textos II e III são predominantemente descritivos.�� O conteúdo do texto III não contradiz as afirmações do texto

I em relação às recomendações sobre os tipos de extratos doGinkgo biloba a serem usados para fins terapêuticos.

�� Infere-se do texto III que há diferentes formas de se encontraro G. biloba nas farmácias brasileiras, mas nem todas sãoigualmente eficazes.

�� No primeiro período do texto III, mantém-se a correçãogramatical caso se coloque uma vírgula logo após “só comreceita médica”, a fim de se ressaltar o caráter explicativo dotermo.

Com relação aos conceitos de informática, ambientes operacionaise aplicativos usados para a criação de textos, planilhas eletrônicase apresentações, julgue os itens a seguir.

�� O Calc é um aplicativo incluído na suíte de pacote de softwaredo BROffice e disponibilizado gratuitamente para uso a partirde um modelo de troca, no qual, se o usuário concordar emcontribuir com adaptações e mudanças nos programas, elepoderá então receber um conjunto de cópias assim que asmudanças forem aceitas.

�� O editor de texto Writer do BROffice, disponível paradownload na Internet, tem a desvantagem de não permitir agravação de dados em formatos comerciais, tal como o PDF,uma vez que não é possível integrar funcionalidades desseambiente com outros softwares proprietários.

�� O Impress é um software que permite a edição deapresentações de slides, os quais, de acordo com a preferênciado usuário, podem ser adequados à audiência para a qual seráapresentado o conteúdo, podendo-se escolher o formato comoretrato ou paisagem, e variar os tipos de design e leiaute, osestilos e a formatação.

A respeito dos conceitos relacionados à Internet, julgue os itens quese seguem.

�� O acesso à Internet em alta velocidade por meio de conexãodial-up, via linha telefônica, também conhecido como serviçoADSL, dispensa o uso de modem, visto que, nesse caso, aconexão ocorre diretamente a partir de infraestrutura dasempresas de telefonia fixa ou móvel (celular).

� Para que o SMTP, um protocolo de recebimento de mensagensde e-mail pela Internet, seja utilizado, é necessário umendereço IP do servidor que armazene as mensagens de correioeletrônico do usuário do serviço.

� Com o surgimento da WWW (world wide web), o acesso aarquivos de conteúdo apenas textual evoluiu para arquivos queagregam diversos formatos, com destaque para os documentoshipermídia, que são a união de hipertexto com multimídia, ouseja, textos com links, imagens, sons, vídeos, entre outrosrecursos.

�� No Internet Explorer (IE), a partir de Opções da Internet,encontrada no menu Ferramentas, é possível configurar umasérie de parâmetros como, por exemplo, a página inicial queserá acessada toda vez que o IE for aberto e em que pastadevem ser armazenados os arquivos temporários da Internet.

www.pciconcursos.com.br

21

marcusbelfort
Rectangle
Agnaldo
Nota
E
Agnaldo
Nota
E
Agnaldo
Nota
C
Agnaldo
Nota
E
Agnaldo
Nota
E
Agnaldo
Nota
C
Agnaldo
Nota
C
Page 23: Exercicios Informatica CESPE 2010 250 Questoes

UnB/CESPE – BRB

����������� �������������������������� �� ������

Acerca da organização e da segurança da informação em meioeletrônico, julgue os próximos itens.�� Confidencialidade, um dos princípios básicos da segurança da

informação, tem como característica garantir que umainformação não seja alterada durante o seu trânsito entre oemissor e o destinatário.

�� Os worms são pouco ofensivos, pois referem-se ao envioautomático de mensagens indesejadas de correio eletrônico aum grande número de destinatários, que não as solicitaram ouque tiveram seus endereços eletrônicos copiados de um sítiopirata.

�� Em um ambiente computacional, a perda das informações porestragos causados por vírus, invasões indevidas ou intempériespode ser amenizada por meio da realização de cópias desegurança (backup) periódicas das informações, as quaispodem ser feitas da máquina do usuário, de servidores e detodos os demais dispositivos de armazenamento, local ouremoto, de dados.

Sérgio e Carla chegam ao autoatendimento de uma agênciabancária para sacarem, respectivamente, R$ 430,00 e R$ 210,00.Nessa agência, estão em operação 10 caixas automáticos, todosindicando, na tela, que contêm notas de 5, 10, 20 e 50 reais. Noentanto, efetivamente, 2 deles contêm apenas notas de 10 reais, 3contêm somente notas de 20 reais, e os demais contêm notas detodos os valores indicados na tela. Nos caixas, existem notassuficientes para os saques, cada um deles tem fila individual, eSérgio e Carla tomaram filas de caixas diferentes. Considerando a situação hipotética apresentada, julgue os itens quese seguem.�� A quantidade de escolhas de pares de filas diferentes

disponíveis para o casal Sérgio e Carla é superior a 100.�� A quantidade de escolhas de pares de filas diferentes

disponíveis para Sérgio e Carla tal que ambos consigamrealizar os saques desejados é maior que 20.

�� A quantidade de escolhas de pares de filas diferentesdisponíveis para Sérgio e Carla tal que pelo menos um delesnão consiga fazer o saque é menor que 20.

�� Considere que as senhas de banco de Sérgio e de Carla sejamcompostas de uma primeira parte numérica de 6 algarismos queassumem valores de 0 a 9 e uma segunda parte constituída detrês letras entre as 26 letras do alfabeto. Considere ainda queas partes alfabéticas das senhas de Sérgio e Carla sejam,respectivamente, TMW e SLZ, e que não sejam permitidassenhas numéricas com todos os números iguais. Nessasituação, o número total de senhas possíveis nesse banco cujaparte alfabética não contenha nenhuma das letras existentes nasenha de Sérgio ou na de Carla é menor que 8 bilhões.

A esposa, o filho e a filha de Marcos são correntistas deuma mesma agência do BRB. Certo dia, entregaram os cartõesmagnéticos a Marcos para sacar dinheiro de suas contas, que têm assenhas de números 201001, 201002 e 201003, e os códigos de trêsletras BRB, RBB e BBR. Marcos sabia a quem pertencia cadacartão e lembrava-se das senhas e dos códigos, mas não dasassociações entre cartões, senhas e códigos. Ele recordava apenasque a senha do cartão da esposa era 201001 e o código de três letrasassociado à senha 201002 era BBR. Marcos decidiu telefonar paracasa e obteve a informação de que o código do cartão da conta dofilho era RBB.Com base nessas informações, julgue os itens a seguir.� A senha do cartão do filho de Marcos é 201003.� O código do cartão da filha de Marcos é BRB.�� Marcos não tinha necessidade de telefonar para casa. As

informações de que dispunha eram suficientes para queencontrasse as associações, sem a referida informação docódigo do cartão do filho.

�� Considerando-se que, no banco de dados dos clientes do bancoBRB, existam pelo menos 35 contas-correntes cujos códigos detrês letras usam apenas as letras B e R, que apenas umcorrentista use o código BBB e que, no máximo, trêscorrentistas usem o código BRB, então existem pelo menoscinco correntistas do BRB com o mesmo código de três letras,usando apenas as letras B e R.

��������

www.pciconcursos.com.br

22

marcusbelfort
Rectangle
marcusbelfort
Rectangle
Agnaldo
Nota
E
Agnaldo
Nota
E
Agnaldo
Nota
C
Page 24: Exercicios Informatica CESPE 2010 250 Questoes

UnB/CESPE – BRB

�������������� ������

A respeito dos conceitos de tecnologias e protocolos utilizados na Web,julgue os itens a seguir.�� O conjunto de técnicas usadas no protocolo WWW permite a

criptografia das conexões de rede realizadas por meio dosprotocolos XML, HTTP e FTP.

�� O DNS (domain name system) é um protocolo de aplicação que faza conversão de um nome em um endereço IP.

�� O uso de HTTPS (HTTP seguro) permite que as informaçõesenviadas e recebidas em uma conexão na Internet estejamprotegidas por meio de certificados digitais.

�� O protocolo FTP, utilizado para videoconferência na Internet, temum modo de conexão ativo, em que o cliente negocia osparâmetros da conexão.

�� A linguagem HTML é utilizada para formatar o conteúdo de umapágina da Internet e pode ser editada com o uso de qualquer editorde texto.

Acerca de segurança e proteção de computadores, julgue os próximositens.�� Um worm se aloja no servidor de e-mail e infecta automaticamente

o computador do usuário sempre que este realiza uma conexão aoserviço de correio eletrônico.

�� O firewall, mecanismo que auxilia na proteção de um computador,permite ou impede que pacotes IP, TCP e UDP possam entrar ousair da interface de rede do computador.

�� Um vírus mutante de computador é capaz de assumir múltiplasformas com o intuito de burlar o software de antivírus.

� Quando o firewall padrão do sistema operacional Windows detectauma tentativa de ataque por meio do protocolo MSN, eleautomaticamente desabilita o software, aciona o antivírus ebloqueia o usuário do MSN para evitar novas tentativas de ataque.

� Vírus de computador são capazes de usar arquivos executáveispara se espalhar. Alguns tipos de vírus detectam e contaminamautomaticamente arquivos e pastas que podem estar armazenadosem um pendrive.

Com relação aos aplicativos BrOffice Writer, Calc e Impress, julgue ositens subsequentes.�� O Writer não permite a geração de arquivos do tipo RTF (rich text

format), entretanto suporta a criação e a manipulação de tabelas.�� O Impress é capaz de exportar uma apresentação para o formato

PDF.A seguir, são apresentadas proposições relativas a um cliente de umainstituição financeira.< Se Carlos fizer um empréstimo na instituição financeira, então ele

não viajará.< Se Carlos não viajar, então ele comprará um carro novo.< Se Carlos comprar uma moto ou usar o cartão de crédito, então ele

não comprará um carro novo.< Se Carlos viajar, então ele usará o cartão de crédito.Considerando que essas proposições sejam verdadeiras, julgue osseguintes itens. �� A proposição “se Carlos viajar, então ele não fará um empréstimo

na instituição financeira” é verdadeira.�� A proposição “se Carlos comprar um carro novo, então ele não

comprará uma moto nem usará o cartão de crédito” é falsa.�� A proposição “se Carlos não usar o cartão de crédito, então ele

comprará um carro novo” é verdadeira.�� A proposição “se Carlos fizer um empréstimo no banco, então ele

comprará uma moto” é verdadeira.

Em determinado dia, dois amigos foram os últimosclientes a chegarem ao atendimento de uma agência bancária,no momento em que quatro operadores de caixa estavamfazendo o atendimento. Nas filas desses caixas estavam, naquelemomento, 11, 14, 12 e 10 clientes. Os tempos máximos deatendimento de cada cliente por esses operadores de caixa sãoiguais, respectivamente, a 3, 2, 2,5 e 2,8 minutos. Até o final doexpediente, não ocorreu atendimento especial e os clientes nãomudaram de fila até serem atendidos.Considerando as informações acima e que os dois amigostambém foram atendidos, julgue os itens a seguir.�� O tempo máximo esperado para que todos os clientes

sejam atendidos é superior a 40 minutos.�� Se todos os clientes estivessem organizados em uma fila

única, então o tempo máximo para o atendimento de todoseles seria inferior a 34 minutos.

� Existem 16 maneiras distintas de os dois amigos seposicionarem ao final dessas quatro filas.

� Considerando todas as possibilidades de escolha de filaspelos dois amigos, o tempo máximo em que um deles, apóster sido atendido, ficará esperando até que termine oatendimento do outro é inferior a 4 minutos.

��������

www.pciconcursos.com.br www.pciconcursos.com.br

23

marcusbelfort
Rectangle
marcusbelfort
Rectangle
Agnaldo
Nota
E
Agnaldo
Nota
C
Agnaldo
Nota
C
Agnaldo
Nota
E
Agnaldo
Nota
C
Agnaldo
Nota
E
Agnaldo
Nota
C
Agnaldo
Nota
C
Agnaldo
Nota
E
Agnaldo
Nota
C
Agnaldo
Nota
E
Agnaldo
Nota
C
Page 25: Exercicios Informatica CESPE 2010 250 Questoes

UnB/CESPE – IBAMA Caderno LTema 5: Conservação, Manejo e Proteção das Espécies da Fauna – 2 –

Considerando aspectos linguísticos do texto Reparação duasdécadas depois, julgue os itens a seguir.� O texto caracteriza-se como essencialmente informativo.� O emprego do sinal indicativo de crase em “à luta de

classes” (R.2) justifica-se pela regência dos termos“subversão” e “incitamento” e pelo gênero do substantivo“classe”.

� O emprego da vírgula após “latifundiários” (R.3) justifica-sepor isolar o termo explicativo.

As palavras “amazônico” e “viúva” acentuam-se de acordocom a mesma regra de acentuação gráfica.

� No segmento “Faltava reparar a injustiça cometida pelosmilitares” (R.6) o complemento do verbo “reparar” poderiaestar precedido da preposição em, com a devida contraçãocom o artigo “a”, sem prejuízo para o sentido e a correçãogramatical do texto.

�� No período que se inicia na linha 8, o sujeito da oraçãoprincipal está posposto ao verbo.

�� O termo “onde” (R.19) introduz oração adjetiva de sentidoexplicativo.

�� O verbo “participar” (R.24) está empregado, no período,como termo substantivo.

�� Os termos “portanto” (R.28) e “enquanto” (R.29), estabelecemidênticas relações de sentido.

Ainda com base no texto de Leandro Fortes e considerandoaspectos textuais e gramaticais, julgue os próximos itens.�� A expressão “Na época”, no início do último parágrafo do

texto, refere-se ao período em que Chico Mendes foiperseguido pela ditadura militar.

�� Pelas opiniões apresentadas no texto, verifica-se que oministro da Justiça e a conselheira possuem posições opostasno que se refere à atuação política de Chico Mendes.

�� Estaria de acordo com o que estabelece a prescriçãogramatical para textos escritos no nível formal da linguagem,tais como documentos oficiais, a substituição da expressão“dali para a frente” (R.3) por dali pra frente.

�� A conjunção “E” (R.7), por ter, no período, valor adversativo,pode ser substituída pela conjunção Mas, sem prejuízo paraas informações do texto.

� Na linha 8, o vocábulo “cujo” estabelece relação sintático-semântica entre os termos “resultado” e “Comissão deAnistia”.

� O termo “o documento” (R.14) refere-se a “portaria deanistia” (R.12).

Julgue os itens a seguir acerca de conceitos de informática.�� Provedores de e-mails na Internet criam, no momento do

cadastro, sítios postais para os usuários, que são capazes dearmazenar mensagens recebidas com limite de tamanhodiversificado.

�� Existem sítios na Internet que disponibilizam a opção paraentrar na intranet mediante a exigência de usuário e senhapreviamente cadastrados.

�� O Internet Explorer apresenta ferramenta que permite criaruma nova mensagem como um cliente Microsoft OfficeOutlook, desde que este esteja configurado como clientepadrão.

�� Software livre é o mesmo que software aberto, e o seuobjetivo é popularizar o uso do computador promovendo ainclusão digital.

Com base na figura acima, que contém uma planilha em ediçãono Excel 2003, julgue os itens de 25 a 30.�� A formatação dos valores da coluna “Desmatamento” como

percentual foi obtida selecionando a referida coluna e

clicando a ferramenta .�� Para calcular a média aritmética do desmatamento nos 4

trimestres é suficiente digitar a fórmula =Média(B2:B5) epressionar a tecla .

�� O gráfico é do tipo personalizado, disponível na ferramenta, denominada assistente de gráfico.

www.pciconcursos.com.br

24

marcusbelfort
Rectangle
Agnaldo
Nota
E
Agnaldo
Nota
C
Agnaldo
Nota
C
Agnaldo
Nota
E
Agnaldo
Nota
E
Agnaldo
Nota
E
Agnaldo
Nota
C
Page 26: Exercicios Informatica CESPE 2010 250 Questoes

UnB/CESPE – IBAMA Caderno LTema 5: Conservação, Manejo e Proteção das Espécies da Fauna – 3 –

�� Para copiar o gráfico e a planilha, ao mesmo tempo, para oWord 2003, é suficiente selecionar os números de linhas de1 a 20, clicar a opção Copiar do menu , abrir o

documento do Word e clicar a ferramenta .

� Considerando-se que o tamanho do arquivo seja igual a1 MB, ele poderá ser enviado tanto para o disquete quantopara um pen drive com capacidade de 1 GB.

� Para criar uma cópia de segurança da planilha, tambémconhecida como backup, é suficiente clicar a ferramenta

.Julgue os itens subsequentes acerca da política nacional do meioambiente (PNMA).�� Segundo a lei de PNMA, o IBAMA terá competência para

o licenciamento ambiental de empreendimentos poluidoressempre que as atividades e obras a serem realizadas geraremsignificativo impacto ambiental de âmbito nacional ouregional.

�� O Sistema Nacional do Meio Ambiente (SISNAMA) éconstituído por órgãos e entidades da União, estados,Distrito Federal, municípios e territórios, responsáveis pelaproteção e melhoria da qualidade ambiental, e suacomposição conta com um órgão superior, que é o conselhode governo; um órgão consultivo e deliberativo, que é oConselho Nacional do Meio Ambiente (CONAMA); bemcomo com um órgão central, um órgão executor, órgãosseccionais e locais.

Paulo, proprietário rural, decidiu instituir uma servidãoambiental em sua propriedade, por 10 anos. Procurou o órgãoambiental competente, que deu anuência à instituição da servidão.Com cinco anos do início da servidão, Paulo precisou vender oimóvel rural e encontrou um comprador, que exigiu amodificação da destinação da área, pois pretendia explorar acultura de soja na fazenda.Considerando essa situação hipotética, julgue os itens a seguir. �� A servidão ambiental instituída por Paulo será averbada na

matrícula do imóvel, no cartório de registro de imóveiscompetente, para que, na venda da propriedade aocomprador, se verifique a existência da servidão e se alterea destinação da área.

�� A servidão ambiental instituída por Paulo se estende a todaa área de sua propriedade, incluídas as de preservaçãopermanente e de reserva legal, se houver.

Marcos, analista ambiental do IBAMA, lotado emBrasília, precisou afastar-se de seu local de trabalho paracolaborar em uma grande operação de fiscalização no estado doPará, por 10 dias. O IBAMA custeou os gastos de hotelaria,alimentação e locomoção urbana de Marcos. Com o início dostrabalhos, percebeu-se que a equipe conseguiria terminar astarefas antes e, de fato, no quinto dia, Marcos retornou a Brasília.Considerando essa situação hipotética e o que estabelece aLei n.º 8.112/1990, julgue os itens que se seguem no que se referea diárias.�� Como Marcos retornou a sua sede em prazo inferior ao

inicialmente previsto, deve restituir as diárias recebidas emexcesso, no prazo de 5 dias.

�� Marcos tem direito a receber as passagens de ida e volta euma diária por dia de trabalho.

Maria, servidora do IBAMA, recebeu a notícia de quesua mãe estava com câncer e que precisaria se submeter atratamento radioterápico em cidade diversa da residência atual deambas. Como era filha única, afastou-se das suas atividades noIBAMA para acompanhar a mãe no tratamento, requerendolicença por motivo de doença em pessoa da família. Em razão dosgastos com esse tratamento, que eram excessivos, Maria decidiuse dedicar a alguma atividade que pudesse aumentar a rendafamiliar. Passou então, a realizar trabalhos de consultoria em suaárea de formação durante o período de licença.Considerando essa situação hipotética, julgue os itens seguintesacerca da licença por motivo de doença em pessoa da família,segundo a Lei n.º 8.112/1990. �� É legítimo o exercício da atividade de consultoria por Maria,

já que se destina ao custeio do tratamento em razão do qualestá licenciada.

�� A licença de Maria será precedida do exame de sua mãe pormédico ou junta médica oficial e, constatado o câncer nesta,serão desnecessários novos exames a cada pedido deprorrogação da licença, se houver.

Julgue os itens de 39 a 42, em conformidade com as normasprevistas na Lei n.º 8.666/1993, que trata de licitações econtratos.� O processamento das compras por meio de sistema de

registro de preços deve ser realizado sempre que se mostrepossível. O sistema de registro de preços demanda prévia eampla pesquisa de mercado, publicação trimestral dos preçosregistrados por meio da imprensa oficial e permite quequalquer cidadão impugne preço constante do quadro geral,caso verifique a incompatibilidade desse preço com ovigente no mercado.

� O autor do projeto básico ou executivo da obra, serviço oufornecimento de bens que estejam sendo alvo de licitaçãonão pode dela participar na fase licitatória, mas pode atuarna fase da execução do contrato, como consultor ou técnico,em funções específicas e exclusivamente a serviço daadministração pública.

www.pciconcursos.com.br

25

marcusbelfort
Rectangle
marcusbelfort
Rectangle
Agnaldo
Nota
C
Agnaldo
Nota
C
Agnaldo
Nota
E
Page 27: Exercicios Informatica CESPE 2010 250 Questoes

UnB/CESPE – TRE/BA

����������� �������������������������� ��������� ������

• De acordo com o comando a que cada um dos itens a seguir se refira, marque na folha de respostas, para cada item: o campodesignado com o código C, caso julgue o item CERTO; ou o campo designado com o código E, caso julgue o item ERRADO. Paraas devidas marcações, use a folha de respostas, único documento válido para a correção das suas respostas.

• Nos itens que avaliam conhecimentos de informática, a menos que seja explicitamente informado o contrário, considere que: todosos programas mencionados estão em configuração-padrão, em português; o mouse está configurado para pessoas destras; expressõescomo clicar, clique simples e clique duplo referem-se a cliques com o botão esquerdo do mouse; teclar corresponde à operação depressionar uma tecla e, rapidamente, liberá-la, acionando-a apenas uma vez. Considere também que não há restrições de proteção, defuncionamento e de uso em relação aos programas, arquivos, diretórios, recursos e equipamentos mencionados.

�����������������Quase todo mundo conhece os riscos de se ter os1

documentos usados de forma indevida por outra pessoa, depoisde tê-los perdido ou de ter sido vítima de assalto. Mas umsistema que começou a ser implantado na Bahia pode resolver4

o problema em todo o país.A tecnologia usada atualmente para a emissão de

carteiras de identidade na Bahia pode evitar esse tipo de7

transtorno. A foto digital, impressa no documento, dificultaadulterações.

A principal novidade do sistema é o envio imediato10

das impressões digitais, por computador, para o banco dedados da Polícia Federal em Brasília. Dessa forma, elaspodem ser comparadas com as de outros brasileiros e13

estrangeiros cadastrados. Se tudo estiver em ordem, o documento é entregue em

cinco dias. Ao ser retirada a carteira, as digitais são conferidas16

novamente.“Você pode até ter a certidão de nascimento de outra

pessoa, mas, quando tentar tirar a carteira por ela, a19

comparação das impressões digitais vai revelar quem é você”,diz a diretora do Instituto de Identificação da Bahia.

Na Bahia, a troca pelo modelo novo será feita aos22

poucos. As atuais carteiras de identidade vão continuar valendoe serão substituídas quando houver necessidade de emitir-se asegunda via. Por enquanto, só a Bahia está enviando os dados25

para a Polícia Federal.Segundo o Ministério da Justiça, a partir de 2011,

outros estados devem integrar-se gradativamente ao sistema.28

A previsão é que, em nove anos, todos os brasileiros estejamcadastrados em uma base de dados unificada na PolíciaFederal.31

Internet: <www.g1.globo.com> (com adaptações).

Com relação ao texto acima apresentado, julgue os itens de 1 a 12.� A nova tecnologia de emissão de carteira de identidade, criada

na Bahia, reduz o risco de fraudes e adulterações.� No texto, tanto o termo “todo” (R.1) quanto “todo o” (R.5)

expressam totalidade. � O texto, que é, predominantemente, descritivo, apresenta

detalhes do funcionamento do sistema de identificação quedeve ser implantado em todo o Brasil.

� Os vocábulos “impressa” (R.8) e “entregue” (R.15) sãoparticípios irregulares dos verbos imprimir e entregar,respectivamente; tais verbos admitem, também, as formasparticipiais regulares: imprimido e entregado.

� A palavra “mas” (R.19), no texto, tem sentido semelhante aoexpresso pelo conectivo e no seguinte período: Assinou odocumento, e se esqueceu de levá-lo.

� Depreende-se do texto que a implantação da nova carteira deidentidade proporcionará mais agilidade aos serviços prestadospelos institutos de identificação do Brasil.

� A supressão da vírgula que sucede a palavra “ordem” (R.15)não acarreta prejuízo à correção gramatical do período emquestão.

� Infere-se do texto que o processo de emissão da nova carteirade identidade será menos dispendioso para o cidadão, visto queas fotos necessárias para o documento serão feitas pelo próprioinstituto de identificação.

O emprego das expressões “vão continuar valendo” (R.23) e“está enviando” (R.25), as quais indicam haver uma ação emcurso, usualmente, deve ser considerado vício de linguagem.

� Do trecho “Por enquanto, só a Bahia está enviando os dadospara a Polícia Federal” (R.25-26) infere-se que, pelo menos, umoutro estado brasileiro também adotou o novo sistema deidentificação, mas não enviou, ainda, as impressões digitaispara atualização do banco de dados da Polícia Federal.

�� Na linha 28, o emprego da preposição a na combinação “ao”é exigência sintática do verbo “integrar”.

�� Infere-se do texto que o novo sistema de identificaçãorepresenta um avanço para o trabalho da polícia brasileira.

A respeito dos conceitos e aplicativos dos ambientes MicrosoftOffice e BROffice, julgue os itens a seguir.�� O Microsoft Word possui opções que permitem

maior agilidade durante a edição de documentos, porexemplo, evitando perdas de informações digitadasou formatações realizadas. Entre essas opções, os botões

permitem, respectivamente, recortar um objetoqualquer no documento, como um trecho do texto ouuma imagem, copiar esse objeto para a área de transferênciae colar tal objeto em determinado local no documento.Essas ações também podem ser realizadas com o uso das teclas� + �, � + � e � + �, respectivamente.

�� No Microsoft Word, ao se selecionar um trecho de um textodigitado, esse trecho aparecerá na tela do monitor com umamarcação, que pode ser uma tarja preta sobre ele. Nessascondições, caso se pressione a tecla �, o trecho selecionadoserá substituído por completo pelo caractere referente à teclapressionada.

�� No BROffice Writer, para se desfazer ou restaurar umadigitação, é necessário, inicialmente, selecionar com o mouseo trecho do texto que se deseja alterar e, em seguida, clicar osbotões adequados na barra de ferramentas ou acionar,respectivamente, as teclas � + � e � + �.

www.pciconcursos.com.br

26

marcusbelfort
Rectangle
marcusbelfort
Rectangle
Agnaldo
Nota
C
Agnaldo
Nota
C
Agnaldo
Nota
E
Page 28: Exercicios Informatica CESPE 2010 250 Questoes

UnB/CESPE – TRE/BA

����������� �������������������������� ��������� ������

Acerca de navegação, correio eletrônico, grupos de discussão eferramentas de busca e pesquisa na Internet, julgue os itens que seseguem.�� Ao verificar a caixa postal de correio eletrônico, na realidade, o

usuário acessa o servidor central de e-mail da Internet, chamado decliente de e-mail, o qual direciona as mensagens que possuem oendereço do usuário reconhecido por sua senha pessoal eintransferível.

�� Uma das formas de busca de informações na Internet utilizando ossítios de busca, como o Google, é por meio da utilização deoperadores booleanos, os quais podem variar dependendo daferramenta de busca utilizada.

�� Um sítio de chat ou de bate-papo é um exemplo típico de grupo dediscussão em que os assuntos são debatidos em tempo real. Para essafinalidade, a comunicação pode ser de forma assíncrona, o quesignifica que é desnecessária a conexão simultânea de todos osusuários.

Com relação aos sistemas operacionais Windows XP e Linux, julgue ospróximos itens.� As informações de espaço livre em um disco rígido de uma máquina

que tenha instalado o sistema Windows XP podem ser obtidas apartir do menu Arquivo do Windows Explorer, acessando-se a opçãoPropriedades, que exibe informações específicas sobre a unidadeselecionada.

� O Linux é um sistema operacional que pode ser usado apenas emservidores, não sendo adequado para a utilização em estações detrabalho do tipo PC. No entanto, é um sistema cujo código-fonte ficadisponível para alterações, permitindo que os usuários contribuampara a sua melhoria.

Quanto ao uso seguro das tecnologias de informação e comunicação,julgue os itens que se seguem.�� As intranets são estruturadas de maneira que as organizações possam

disponibilizar suas informações internas de forma segura, irrestrita epública, sem que os usuários necessitem de autenticação, ou seja, defornecimento de nome de login e senha.

�� Uma das formas de bloquear o acesso a locais não autorizados erestringir acessos a uma rede de computadores é por meio dainstalação de firewall, o qual pode ser instalado na rede como umtodo, ou apenas em servidores ou nas estações de trabalho.

O jogo de dominó tradicional é jogado com 28peças, igualmente divididas entre 4 jogadores sentados facea face em torno de uma mesa retangular. As peças sãoretangulares e possuem uma marcação que as divide em duasmetades iguais; em cada metade: ou não há nada gravado, ouestá gravado um determinado número de buracos querepresentam números. As metades representam 7 números: 1,2, 3, 4, 5, 6 e 0, sendo este último representado por umametade sem marcação. Cada número ocorre em 7 peçasdistintas. Em 7 peças, denominadas buchas, o número aparecenas duas metades. Existe também uma variação de dominóconhecida como double nine, em que as metades representamos números 0, 1, 2, 3, 4, 5, 6, 7, 8 e 9, em um total de 55peças.

M. Lugo. How to play better dominoes. New York:Sterling Publishing Company, 2002 (com adaptações).

A partir dessas informações, julgue os itens subsequentes.

�� Uma variação de dominó cujas metades representem osnúmeros 0, 1, 2, 3, 4, 5, 6, 7, 8, 9, 10, 11 e 12 terá umtotal de 82 peças.

�� No dominó tradicional, os 4 jogadores podem se sentarà mesa de 6 maneiras distintas.

�� Considere que cada jogador, na sua vez, retire as 7 peçasao mesmo tempo. Nesse caso, as peças de um dominótradicional poderão ser divididas entre os 4 jogadores de maneiras distintas.

�� Entre todas as possíveis divisões das peças de umdominó tradicional entre os 4 jogadores, em mais de100 milhões delas algum deles começará o jogo comtodas as 7 buchas.

��������

www.pciconcursos.com.br

27

marcusbelfort
Rectangle
Agnaldo
Nota
E
Agnaldo
Nota
C
Agnaldo
Nota
E
Agnaldo
Nota
C
Agnaldo
Nota
E
Agnaldo
Nota
C
Agnaldo
Nota
C
Page 29: Exercicios Informatica CESPE 2010 250 Questoes

UnB/CESPE – TRE/BA

������������� ���������������������������������������� ������

�� As expressões “Meu caro Paz” (R.2) e “o velho amigo” (R.10)correspondem, respectivamente, ao vocativo e à epígrafe dacomunicação particular em questão.

�� Em “senti-lhes o afeto antigo” (R.4), a forma pronominal “lhes”refere-se às expressões “tuas palavras” e a “teu abraço” ambasna linha 3.

�� Na oração “se a solidão me abate” (R.6), a substituição doconector “se” por acaso não prejudicaria o sentido expressonessa oração e a correção gramatical.

�� As formas verbais “anuncias” (R.8), “concluas” (R.9) e “aludes”(R.9) estão conjugadas na segunda pessoa do singular domesmo tempo e do mesmo modo verbal.

�� Em “oxalá concluas a viagem” (R.8-9), o vocábulo “oxalá”pode ser substituído por tomara que, mantendo-se, assim, osentido do trecho em que se insere.

Acerca da redação de correspondências oficiais, julgue o itemabaixo.� Como vocativo das comunicações oficiais destinadas a

senadores, juízes, ministros e governadores, recomenda-seevitar o título acadêmico de Doutor e usar o pronome detratamento Senhor.

Com relação aos conceitos e aplicativos dos ambientes MicrosoftOffice e BROffice, julgue os itens de que se seguem.� A impressão de um documento do Word pode ser feita a partir

do botão de impressora — —, localizado na barra deferramentas, ou acionando-se as teclas � + �.A diferença entre essas duas ações é que a primeira permite aconfiguração de impressão de acordo com as opções desejadas,enquanto a segunda envia o documento diretamente para aimpressora padrão que estiver configurada no computadorem uso.

�� No Microsoft Word, os botões permitem,respectivamente, desfazer uma ação indesejada e refazer umaação desfeita. Esses efeitos também podem ser obtidos pormeio da combinação de teclas � + � e � + �,respectivamente.

�� Para se inserir dados em uma planilha do Microsoft Excel,deve-se, inicialmente, selecionar a célula onde os dados serãoinseridos. Esse procedimento pode ser realizado com o uso domouse, posicionando o cursor na célula desejada, ou a partirdas setas do teclado, ou teclando �, para, em seguida, sedigitar os dados na célula e, por fim, confirmar a operaçãocom �.

�� No BROffice Writer, a partir do menu Exibir, é possível inserir,no documento em edição, um objeto do tipo gráfico, fórmula,som ou vídeo.

Acerca de navegação, correio eletrônico, grupos de discussão eferramentas de busca e pesquisa, julgue os próximos itens.�� A caixa postal de correio eletrônico é um diretório criado no

servidor de e-mail, o qual fica localizado no computador dousuário. Ao ser ligada a máquina, esse servidor recebe daInternet, via provedor de acesso, as mensagens que foramenviadas para o endereço do usuário.

�� Um grupo de discussão é uma maneira fácil e imediata de seinteragir na Internet, a exemplo dos chats que, em tempo real,possibilitam a duas ou mais pessoas se comunicarem de formaassíncrona. Os blogs e os fotologs também constituemexemplos de grupos de discussão.

Com relação ao uso seguro das tecnologias de informação ecomunicação, julgue os itens subsequentes.�� No acesso à Internet por meio de uma linha digital assimétrica

de assinante (ADSL), a conexão é feita usando-se uma linha detelefone ligada a um modem e os dados trafegam em altavelocidade.

�� Firewall é um recurso utilizado para a segurança tanto deestações de trabalho como de servidores ou de toda uma redede comunicação de dados. Esse recurso possibilita o bloqueiode acessos indevidos a partir de regras preestabelecidas.

�� Confidencialidade, disponibilidade e integridade dainformação são princípios básicos que orientam a definição depolíticas de uso dos ambientes computacionais. Essesprincípios são aplicados exclusivamente às tecnologias deinformação, pois não podem ser seguidos por seres humanos.

Julgue os itens que se seguem, a respeito dos sistemas operacionaisWindows XP e Linux.� No Windows XP, é possível tornar um diretório restrito,

usando-se funcionalidade encontrada na aba Compartilhamento,que é acessada a partir da opção Propriedades do menu Arquivodo Windows Explorer.

� O Linux tem a característica de ser um sistema desenvolvidode maneira colaborativa, tal que os software da suíte BROffice,por exemplo, podem ser alterados durante o uso, depois deinstalados. Assim, é possível modificar os comandos referentesaos ícones dispostos na barra de ferramentas, alterar as teclasde atalho e também o idioma do aplicativo.

��������

www.pciconcursos.com.br www.pciconcursos.com.br

28

marcusbelfort
Rectangle
Agnaldo
Nota
C
Agnaldo
Nota
C
Agnaldo
Nota
E
Agnaldo
Nota
E
Agnaldo
Nota
C
Agnaldo
Nota
C
Agnaldo
Nota
E
Agnaldo
Nota
E
Agnaldo
Nota
E
Page 30: Exercicios Informatica CESPE 2010 250 Questoes

UnB/CESPE

Banco da Amazônia S.A. – Cargo 23: Técnico Bancário – 2 –

Texto para os itens de 15 a 20Entrevista: Marina Silva

Protagonismo feminino na Amazônia é muito forteAs mulheres, em todo o mundo, têm de passar por1

muitos obstáculos — entre eles o preconceito. Para você, o queé ser uma mulher na região amazônica? Quais os desafios e vantagens?

Marina Silva — Primeiro que ser uma mulher na4

Amazônia, ainda que com suas peculiaridades, guardasemelhanças com ser uma mulher no Brasil. As dificuldades, ospreconceitos, que muitas vezes elas têm de enfrentar, não são7

diferentes porque se trata da Amazônia. Isso não vai serdiferente do que a gente vai encontrar nas diversas regiões dopaís.10

Acho que uma característica importante é que naAmazônia elas foram assumindo um protagonismo muito forteem todos os sentidos. Se você pega a luta dos seringueiros,13

você vai ver figuras femininas. A primeira presidente doSindicato dos Trabalhadores Rurais de Xapuri, na época doChico Mendes, era uma mulher [Dona Raimunda], que agora16

novamente está no sindicato. A formação do ConselhoNacional dos Seringueiros tem uma forte participação dasmulheres, inclusive da Dona Raimunda.19

Você também pode observar isso na política, naacademia. A presidente do museu Goeldi também é umamulher. Você tem uma forte participação da mulher. Então ser22

uma mulher na Amazônia comporta a dor e as delícias de seruma mulher no Brasil, com as dificuldades típicas de cada região.

E quais seriam essas dificuldades típicas da região25

amazônica? Marina Silva — Acho que o atendimento das

demandas de Saúde e Educação, aquelas demandas que são28

básicas e essenciais e que para uma grande parte das mulheresna Amazônia são algo muito distante. Você tem uma ausênciado Estado muito grande na prestação de serviços elementares:31

do atendimento da saúde da mulher, planejamento familiar,atendimento da infância e é algo que sobrecarrega muito asmulheres.34

Thais Iervolino. Internet: <www.portala m a z o n i a . o r g > ( c o m a d a p t a ç õ e s ) .

Tendo em vista as informações veiculadas no texto, além de seusaspectos gramaticais e textuais, julgue os itens seguintes.15 Seria mantida a concordância verbal no trecho “As mulheres,

em todo o mundo, têm de passar por muitos obstáculos” (R.1-2)retirando-se o acento circunflexo da forma verbal “têm”.

16 O pronome “elas” em “elas têm de enfrentar” (R.7) refere-se àmulher que vive na Amazônia.

17 Para a entrevistada, as dificuldades da mulher que vive naAmazônia são semelhantes às que mulheres de outras regiõesencontram para se afirmar na sociedade, embora ocorram emescala menor.

18 O trecho “Se você pega a luta dos seringueiros” (R.13) temcaracterísticas da linguagem informal e poderia ser reescrito,mantendo-se o significado, da seguinte forma: Se tomarmoscomo exemplo a luta dos seringueiros.

Considerando o texto e os seguintes requisitos: “A redação oficialdeve caracterizar-se pela impessoalidade, uso do padrão culto delinguagem, clareza, concisão, formalidade e uniformidade”(Manual de Redação da Presidência da República, 2002), julgueos itens que se seguem.19 O quarto parágrafo — “Você também (...) cada região”

(R.20-24) — não seria adequado para compor um documentocom o padrão ofício.

20 As respostas da entrevistada, na ordem em que sãoapresentadas, poderiam fazer parte do corpo de um relatório.

Em fevereiro de 1999, Hugo Chávez iniciou o seu primeiromandato como presidente venezuelano. A respeito de seusonze anos de governo, julgue os itens a seguir.21 Chávez defende novas formas de cooperação internacional

entre os países latino-americanos, destacando-se a criação daAliança Bolivariana para as Américas, em oposição ao Tratadode Livre Comércio das Américas.

22 A nacionalização de empresas estrangeiras de diversos setorese o maior controle estatal da economia têm sido marcas dogoverno de Hugo Chávez.

23 A política externa de Hugo Chávez é marcada pela criação defortes laços com a Colômbia, base de seu projeto debolivarianismo do século XXI.

Com relação ao impacto da ocupação humana na Amazônia, julgueos itens que se seguem.24 Na Amazônia, as usinas termelétricas produzem quantidades

desprezíveis de CO2, quando comparadas às usinashidrelétricas.

25 Menos de 10% das terras da Amazônia Legal têm títulos depropriedade válidos, uma das consequências da ocupaçãodesordenada da região.

26 A principal área de devastação ambiental da Amazônia é ochamado arco do desmatamento, região que se estende deRondônia ao Maranhão.

27 As cidades têm pouco impacto ambiental porque, ao contráriodo restante do Brasil, a maior parte da população da Amazôniaainda vive no campo e está envolvida em atividades típicas dodesenvolvimento sustentável.

O Painel Intergovernamental sobre Mudanças Climáticas (IPCC) setornou referência nas discussões sobre possíveis mudanças no climado planeta. Acerca do IPCC e de seus relatórios, julgue os itenssubsequentes.28 No quarto relatório produzido pelo órgão, em 2007, foi

avaliado que, desde meados do século XX, o aquecimentoglobal é inequívoco e quase certamente causado pela ação dohomem.

29 O IPCC é ligado ao governo dos Estados Unidos da América,o que, por vezes, tem posto em dúvidas os relatórios do órgão.

30 Em 2009, o IPCC foi agraciado, juntamente com o presidenteamericano Barack Obama, com o prêmio Nobel da Paz, emrazão dos serviços prestados para a melhoria do conhecimentoclimático do planeta.

A respeito dos ambientes Windows XP e Linux, julgue os itens aseguir.31 Os sistemas Windows XP e Linux têm kernel comum, aberto,

que pode ser facilmente customizado pelo usuário.32 O Linux, um sistema multitarefa e multiusuário, é disponível

em várias distribuições, entre as quais, Debian, Ubuntu,Mandriva e Fedora.

33 O Windows XP possui, por padrão, uma interface gráfica,enquanto o Linux tem disponíveis várias interfaces gráficas,que podem ser instaladas e customizadas segundo anecessidade do usuário.

34 Tanto o Linux quanto o Windows XP possuem suporte nativoao protocolo TCP/IP para acesso à Internet.

www.pciconcursos.com.br www.pciconcursos.com.br

29

marcusbelfort
Rectangle
marcusbelfort
Rectangle
Agnaldo
Nota
E
Agnaldo
Nota
C
Agnaldo
Nota
C
Agnaldo
Nota
C
Page 31: Exercicios Informatica CESPE 2010 250 Questoes

UnB/CESPE

Banco da Amazônia S.A. – Cargo 23: Técnico Bancário – 3 –

Considerando que, em uma empresa, um computador seja usado porvárias pessoas para acesso à Intranet, julgue os itens que se seguem.35 Em uma situação normal, caso um usuário deseje acessar a

Internet e a intranet da empresa ao mesmo tempo, ele deveusar duas versões diferentes de navegador, por questão deincompatibilidade de tecnologia entre a Internet e a intranet.

36 Na situação em questão, o uso de correio eletrônico pode serfeito por meio de um cliente de correio, como o MicrosoftOutlook, ou com o uso de uma interface web. Em ambos oscasos, é possível que as mensagens de correio do usuáriofiquem armazenadas no servidor de e-mail da empresa.

37 Para melhorar o nível de controle de acesso dos usuários àInternet, é possível o uso de mecanismos de segurança comofirewalls e antivírus.

38 Caso se deseje usar o referido computador para gerardocumentos no Windows XP, é possível salvar essesdocumentos na pasta Meus Documentos, mas, por padrão,nenhuma pasta ou subpasta pode ser excluída dessecomputador, e também não é possível enviar documentoanexado a correio eletrônico.

39 Se o acesso à Internet for feito mediante um servidor proxy,pode ser impedido o acesso a determinadas páginas e tipos desoftware, em função de configurações de restrição que podemser realizadas.

40 TCP/IP é o protocolo padrão para o acesso à Internet. No casoda intranet, o acesso é realizado, por padrão, com o uso doprotocolo IPv6.

Os saldos dos depósitos em poupança dos correntistasMiguel, Norberto, Orlando e Paulo, em uma agência bancária,somavam R$ 64.000,00. Em um mesmo dia, foram realizadas asseguintes operações nessas contas de poupança:< Miguel fez um depósito de R$ 3.000,00;< Norberto fez um saque de R$ 3.000,00;< Orlando depositou o dobro do montante que já possuía de

saldo;< Paulo sacou dois terços do seu saldo.

Após essas operações, os quatro clientes ficaram com omesmo saldo em suas respectivas contas de poupança.Com referência a essa situação hipotética, julgue os itens a seguir,a respeito dos saldos das contas desses correntistas antes derealizadas as citadas operações.41 O saldo da conta de Paulo era igual ao triplo do saldo da conta

de Orlando.42 O saldo da conta de Miguel era igual à metade do saldo da

conta de Norberto.43 A soma dos saldos das contas de Miguel e de Norberto era

igual a dois terços do saldo da conta de Paulo.44 O saldo da conta de Orlando era igual a R$ 4.000,00.45 O saldo da conta de Norberto era superior a R$ 16.000,00.

Acerca de matemática financeira, julgue os itens subsequentes.46 Considerando 1,1 e 1,0489 como valores aproximados de

1,0128 e 1,0124, respectivamente, é correto afirmar que a taxaanual de juros equivalente à taxa de juros compostos de 1,2%ao mês é inferior a 15%.

47 Se um empréstimo de R$ 1.000,00 for quitado em 10prestações, mensais e consecutivas, a juros de 2% ao mês, pelosistema de amortização constante (SAC), e se a primeiraprestação vencerá um mês após a contratação do empréstimo,então o valor da terceira prestação será igual a R$ 116,00.

48 Considerando que a instituição financeira X ofereça aosclientes a taxa de desconto de 2,4% ao mês para desconto detítulos, e que a instituição concorrente Y ofereça uma reduçãode 25% na taxa praticada pela X, para descontos dos títuloscom vencimentos em até 90 dias, então o valor atual, comdesconto simples por fora, pago pela Y para um título comvalor de face de R$ 1.000,00 e que vence em 2 meses é inferiora R$ 960,00.

49 Caso uma loja de roupas ofereça o desconto de 5% sobre opreço de cada peça para pagamento à vista, ou o pagamento emduas parcelas, mensais e iguais, sem acréscimo, com a primeiradevendo ser paga no ato da compra, então a taxa mensal dejuros que a loja embute nos preços para vendas a prazo ésuperior a 10%.

50 Se um cliente aplicou seu dinheiro em uma instituiçãofinanceira à taxa de juros (aparente) de 7,52% ao ano, durantedeterminado ano em que a inflação oficial apurada foi de 5%,então o valor aplicado por esse cliente, nesse ano, rendeu jurosreais acima de 2,5%.

��������

www.pciconcursos.com.br www.pciconcursos.com.br

30

marcusbelfort
Rectangle
marcusbelfort
Rectangle
Agnaldo
Nota
E
Agnaldo
Nota
C
Agnaldo
Nota
C
Agnaldo
Nota
E
Agnaldo
Nota
C
Agnaldo
Nota
E
Page 32: Exercicios Informatica CESPE 2010 250 Questoes

UnB/CESPE

Banco da Amazônia S.A. — Cargo 1: Técnico Científico — Área: Administração – 3 –

Acerca de informática, julgue os itens a seguir.

31 As informações processadas nos computadores são compostaspor caracteres, sendo que cada caractere, representado por 0 ou1, é chamado de byte, e um conjunto de oito bytes constitui umbit.

32 A memória cache do computador é um tipo de memóriaintermediária que guarda as informações oriundas da memóriaprincipal, com a finalidade de agilizar o acesso do processadora essas informações.

33 As placas de rede do tipo Wi-Fi operam sob uma arquiteturado tipo Ethernet e servem para conectar computadores a redesdo tipo WAN (wide area network), por cabo de par trançado.

34 A principal característica das impressoras multifuncionais éreunir, em um único equipamento, diversas funcionalidadesque antes eram segregadas em dispositivos independentes,como impressora, copiadora e escâner.

Com relação a sistemas operacionais, julgue os itens que se seguem.

35 No Windows XP, a barra de inicialização rápida oferece umespaço para ícones associados a programas utilizados commais frequência, os quais podem ser acionados com apenas umclique do mouse.

36 No Windows XP, a janela Meu Computador, que pode estaracessível tanto pelo desktop quanto pelo menu Programas,permite acesso ao ambiente de gerenciamento de pastas earquivos, o qual, quando acionada a opção de visualização depastas, apresenta a mesma interface do Windows Explorer.

37 A partir do menu Arquivo do Windows Explorer, o WindowsXP oferece a opção de se criar um arquivo em formato editávelno Excel, no Word e no PowerPoint, e também de criararquivos da suíte BROffice, desde que esses software estejaminstalados no computador em uso.

38 No Linux, o aplicativo KDE Controle Center temfuncionalidades equivalentes ao Painel de controle do Windows,ambos permitindo o gerenciamento de pastas e arquivos e aconfiguração para a permissão de acesso aos usuários docomputador.

A respeito da utilização de aplicativos dos ambientes MicrosoftOffice e BROffice, julgue os itens de 39 a 43.

39 A barra de ferramentas de formatação do Excel contém opçõesque permitem inserir, em uma planilha, figuras, formas e linhase também configurar cores e autoformas.

40 No Word, o recurso de autocorreção do menu Ferramentas é útilpara a correção gráfica e sintática de palavras e frases digitadasem um documento em edição.

41 No Excel, a alça de preenchimento é utilizada para aduplicação de um dado inserido em uma célula para as demaiscélulas na direção em que o usuário arrastar o mouse, seja decima para baixo, da direita para a esquerda ou na diagonal.

42 No Writer do BROffice, a opção Alterar capitalização, disponívelno menu Formatar, permite inverter a fonte usada no texto entremaiúsculas e minúsculas.

43 Para a criação de apresentações de slides com dadosmatemáticos, o Calc do BROffice oferece a possibilidade deinserção de uma planilha com tabelas e fórmulas de formataçãoautomática, por meio de opção encontrada no menu Arquivo.

No que se refere a tecnologias de Internet e intranet e à segurançada informação, julgue os itens subsequentes.

44 A Internet funciona a partir do modelo cliente/servidor, noqual os computadores dos usuários operam como clientesconectados aos servidores que funcionam como provedoresde acesso e de serviços de correio eletrônico, transferênciade arquivos e acesso a páginas web.

45 Um servidor de saída de e-mails, ou servidor POP, éobrigatório para que um serviço de correio eletrônico sejaestabelecido em um servidor, o qual deve ser responsável porenviar os e-mails para usuários cadastrados.

46 O serviço de acesso à Internet por ADSL não necessita demodem para estabelecer uma conexão, que é realizada por umcabo UTP dedicado, ligado entre o computador do usuário e oprovedor de acesso.

47 A Internet por rádio, no Brasil, ainda é um serviço de baixavelocidade, sujeito a intempéries e inoperante no caso de diasnublados, porque utiliza infraestrutura por satélite.

48 Cliente web ou WWW, a exemplo do Internet Explorer e doMozilla Firefox, é um programa utilizado para acessar osservidores que armazenam, na Internet, as páginas de usuáriosou organizações.

49 Uma rede do tipo VPN (virtual private network) é fundamentalpara evitar que vírus ou programas maliciosos entrem noscomputadores de determinada empresa, já que esse tipo derede é configurado de modo a bloquear qualquer arquivo quenão seja reconhecido pelo firewall nela instalado.

50 Port scanner é um programa que, se instalado em umcomputador, permite que um invasor volte a acessá-lo quandoquiser para invadi-lo. Geralmente é instalado a partir deprogramas do tipo cavalo-de-troia ou até por e-mail.

www.pciconcursos.com.br

31

Agnaldo
Nota
E
Agnaldo
Nota
C
Agnaldo
Nota
E
Agnaldo
Nota
C
Agnaldo
Nota
C
Agnaldo
Nota
C
Agnaldo
Nota
C
Agnaldo
Nota
E
Agnaldo
Nota
C
Agnaldo
Nota
E
Agnaldo
Nota
E
Agnaldo
Nota
E
Agnaldo
Nota
C
Agnaldo
Nota
E
Agnaldo
Nota
E
Agnaldo
Nota
E
Agnaldo
Nota
C
Agnaldo
Nota
E
Agnaldo
Nota
E
Page 33: Exercicios Informatica CESPE 2010 250 Questoes

UnB/CESPE – MSConhecimentos Básicos (para todos os cargos de nível superior) – 2 –

Em relação aos sistemas operacionais Windows XP e Linux,julgue os itens a seguir.

�� No Windows XP, as teclas � e �, quandopressionadas simultaneamente, permitem alternarrapidamente entre os programas abertos.

�� O Windows XP disponibiliza, por meio do menu Iniciar>Todos os Programas> Acessórios> Ferramentas do Sistema, aativação do programa Desfragmentador de disco, que pode serutilizado para recuperar fragmentos de arquivos danificadospor falhas de disco.

�� No sistema operacional Linux típico, o subdiretório /dev dodiretório raiz contém os arquivos executáveis (binários) decomandos essenciais pertencentes ao sistema, e que sãousados com frequência pelas aplicações.

�� No sistema operacional Linux, um ponto (.) no início donome identifica os arquivos ocultos.

Quanto aos aplicativos do Microsoft Office e do BrOffice.org,julgue os itens que se seguem.�� No aplicativo Writer do BrOffice.org, para se fechar um

arquivo que esteja sendo editado, é possível usar qualqueruma das duas seguintes combinações de teclas: � + �ou� + �.

�� O aplicativo Microsoft Word 2003, na sua configuraçãopadrão, permite, a partir do menu Exibir, quatro modos devisualização do texto: Normal, Layout da web, Layout deimpressão e Estrutura de tópicos.

�� No aplicativo Microsoft PowerPoint 2003, uma das maneiraspossíveis de se iniciar a apresentação dos slides de umarquivo em edição é clicar no menu Apresentações eselecionar a opção Exibir Apresentação.

Acerca de conceitos de organização de arquivos e Internet, julgueos itens seguintes.�� Firewall é o mecanismo usado em redes de computadores

para controlar e autorizar o tráfego de informações, por meiodo uso de filtros que são configurados de acordo com aspolíticas de segurança estabelecidas.

� No Internet Explorer, ao se clicar o botão , o usuário tempossibilidade de configurar o endereço URL do site que seráexibido ao iniciar o navegador Internet Explorer.

� No Windows Explorer, para se excluir um arquivodefinitivamente, sem que ele seja enviado para a Lixeira, énecessário clicar sobre o nome do arquivo com o botãodireito do mouse, manter pressionada a tecla � e, na listadisponibilizada, clicar a opção Excluir.

Quanto às competências e atribuições dos entes da Federação emmatéria de saúde, julgue os itens seguintes.

�� Compete à União, aos estados, ao Distrito Federal (DF) eaos municípios, no âmbito administrativo, elaborar aproposta orçamentária do Sistema Único de Saúde (SUS).

�� Conforme legislação do SUS, constitui competênciaexclusiva da União a realização de operações externas denatureza financeira de interesse da saúde, autorizadas peloSenado Federal.

�� Cabe à União, com seus recursos próprios, financiar oSubsistema de Atenção à Saúde Indígena.

�� Entre as atribuições estabelecidas pela lei à direção estadualdo SUS, destaca-se a de formar consórcios intermunicipais,em benefício da população dos municípios inseridos na suaesfera territorial.

No que se refere aos serviços privados de assistência à saúde,julgue o item subsequente.

�� O SUS pode recorrer aos serviços prestados pela iniciativaprivada quando suas disponibilidades forem insuficientespara garantir a cobertura assistencial à população dedeterminada área, sempre por intermédio de entidadesfilantrópicas sem fins lucrativos.

Acerca da participação da comunidade na gestão do SUS, julgueos itens que se seguem.

�� O SUS conta, em cada esfera de governo, com o Conselhode Saúde, órgão colegiado composto exclusivamente pelosrepresentantes do governo e pelos usuários, o qual atua naformulação de estratégias, no controle da execução dapolítica de saúde na instância correspondente e nos aspectoseconômicos e financeiros.

�� A Conferência de Saúde, que se reúne a cada quatro anospara avaliar a situação de saúde e propor diretrizes para aformulação da política de saúde nos níveis correspondentes,pode ser convocada pelo Conselho de Saúde.

www.pciconcursos.com.br

32

marcusbelfort
Rectangle
Agnaldo
Nota
E
Agnaldo
Nota
E
Page 34: Exercicios Informatica CESPE 2010 250 Questoes

��������������� ��������������������������������� �!���"#��#�$� ��%��&'()*+,-./0123456780260926:5;<6=;>092?02@;<3090A=2BAA;<4C2699;D6542620:780<0334=6EF G12?0<H14D=02456I036?02D02J03?2:0?429432965>02412H163KH;>02<012H12D014249:4<LA;<02<0D=4D?0262=431;D67802E?0<C941262H=;5;M67802?42<636<=4349249:4<;6;92=6;92<0102:0D=024>L3NH562OPQ242I63362ORQE2S994263KH;>02:0?429432N36>6?02412H16:69=62<3;6?62D621TKH;D62412H9020H2412T3462?;9:0DL>45241234?4?42<01:H=6?0349C2942U0H>43EV /01202H902?02W0X43W0;D=C2Y2:099L>452<3;6326:3494D=67Z49<0D=4D?02H12<0D[HD=02?42\]_\2412H12aD;<0263KH;>0242;D943;3D0>092\]_\2:3499;0D6D?0b942692=4<5692 2c2 20H<5;<6D?02620:7802defeghijklC2D02mno2pqjrjsEt B2\uvwxyz2S{<452?;9:0D;I;5;M62AHD<;0D65;?6?492:6362<3;632H1I6D<02?42?6?092KH42:431;=626990<;632=6I4569242<05HD692?4A0316203N6D;M6?6C2;D=4N36?624249=3H=H36?6E| B234<H3902}~hlsjsg��rli�h2?02J03?2:431;=4262<3;67802?42=6I4569KH426990<;6125;DU69242<05HD69C2D692KH6;92:0?4294323465;M6?0<T5<H5026H=01T=;<02?42?6?09C2<0102412H162:56D;5U62454=3�D;<6C9412KH4294[62D4<499T3;02H=;5;M63C2:636545614D=4C2=6I45692S{<45E� W6362942A6M43262=6IH567802412H12?0<H14D=0C241249:4<;652D64?;7802?42?0<H14D=0920A;<;6;92<0121H;=092:63TN36A09C2;=4D9249HI;=4D9C2?4>4b942:3499;0D63262=4<562 24262I63362?449:6709C202KH42N636D=42H1265;DU614D=02?42=4{=0242:63TN36A09?4216D4;362:6?30D;M6?62429412?49>;09E2&'()*+,-���H6D=0260926:5;<6=;>092?0261I;4D=42��BAA;<4C2699;D6542620:780<0334=6EF �20:7802��if�sg��ke2?02J3;=432:431;=42965>632412H12aD;<0?0<H14D=02=0?69269265=4367Z492KH42A03612A4;=692D4542:032H120H16;92H9HT3;09EV �026:5;<6=;>02/65<C2Y2:099L>45210D=632A�31H5692?42<T5<H50216;96>6D76?69242<0120:7Z492KH42D8024{;9=412D02S{<45C2>;9=02KH402:3;14;302Y2H12\uvwxyz25;>342?494D>05>;?0242?;9:0D;I;5;M6?0:6362H9021H;=026D=492?02a5=;10E2t �029426=H65;M632H12?0<H14D=02D02J3;=43C2Y2:099L>452?49A6M4369265=4367Z492A4;=69C2<0D9;?436D?020249=TN;02412KH424542A0;965>02:4562a5=;162>4MC262:63=;32?02<016D?02�l��ssl��s2?02mno�s��jfeE| G162?692>6D=6N4D92?0926:5;<6=;>092?629HL=42��BAA;<42Y266H9�D<;62?42699;9=4D=492?42:30?H7802?42?0<H14D=09C2:0;9245499802?42:0H<626[H?6242<0DAHD?41202H9HT3;02?H36D=42624?;7802?4?0<H14D=0920H26:3494D=67Z49C2>;9=02KH42D802:099H412;D=43A6<461;NT>4520H2H1625;DNH6N412:3�{;162?02H9HT3;0E� W6362942;D943;32H162D0>62A�31H562D02/65<C2?4>4b942H9632620:780��i��i�s2?02mno2�lss��l~��hE&'()*+,-���99;D6542620:7802KH42<0D=Y126:4D692A433614D=692?42D6>4N6780242?4<0334;02454=3�D;<02H=;5;M6?69241234?492?42<01:H=6?0349EF �D=43D4=2S{:50343C2@0M;5562�;34A0{C2BH=500�C2BH=500�2S{:3499V /U6=C2�/�C2@��C2�X;==43t J���C2J4IC2JJJC2�;DN43| �0U43C2G94D4=2�4X9C2J6;9C2�4=D4X9� ��3H1C2�014W6N4C2W699X03?C2G94D4= &'()*+,-���99;D6542620:7802<0334=62<012345678026029;9=41620:436<;0D65J;D?0X92�WEF B2J;D?0X92�W2:099H;C26:634D=42412=0?0920926:5;<6=;>092KH498024{4<H=6?092D499429;9=41620:436<;0D65C2H162I63362?4A433614D=692?42[6D4562:6?3802<0120:7Z492?426I3;320H2A4<U6363KH;>0920H2:69=69C26<499632HD;?6?42?4234?42425;N6320H2?495;N63EV �20:7802 �¡l�sg�~jk�klgklgslklC26<4996?62?;34=614D=42:450L<0D42 C2:431;=42;?4D=;A;<6320D?4249=T250<65;M6?02H163KH;>02KH42942?494[624D<0D=3632D621TKH;D62412H90Et W03214;02?620:7802¢se¡sjlk�klh2?62I63362?42=634A69242?02mno}~j�j�s2?02J;D?0X92�WC2Y2:099L>452=4326<499023T:;?0242;14?;6=0£920:7Z492?42��if�sC2}�¡sj�jsC2}�¡es��s242pq¡es��s209263KH;>092KH49802H=;5;M6?092:4502H9HT3;0E| S12¢se¡sjlk�klhgklg¤jh�egie��ig¥�¦§2?02J;D?0X92�WC2Y2:099L>45=4326<4990262;DA03167Z4926<43<62?62KH6D=;?6?42?4249:6702H96?042?4249:67025;>342D02� E2�61IY12Y2:099L>4526<4996326A433614D=62©j�¡lª�gklgkjh�e2:63620=;1;M63262?;9=3;IH;7802?463KH;>092D02� E� B2¢�j~ligklg�e~�seilg?02J;D?0X92�W20A434<42H12<0D[HD=02?4A433614D=692a=4;92:6362KH42092H9HT3;092<0DA;NH34129H692=634A69IT9;<69C2<010234<4I432424D>;632«my]\C26<499632092:30N3616942692A433614D=692?424D=34=4D;14D=02426<49963262:69=62 l�hke���l~�ehC24D=3420H=369E2&'()*+,-�¬/01234567802602H902?62�D=43D4=242?62nwzynwC2699;D6542620:780<0334=6EF �92nwzynw\29802?49=;D6?692602H90241250<6;923410=09C20D?42D80942?;9:Z42?426<4990262:30>4?03492?426<49902£2�D=43D4=EV W63629423465;M632H162<0D4{802£2nwzynwC2Y2D4<499T3;02H1:30>4?032?426<49902;D9=656?02D621TKH;D62?02H9HT3;0242H12<6I0?42I6D?62563N62:636262=36D9A43�D<;62?4263KH;>092?42?6?09Et �62�D=43D4=C2026<49902?4?;<6?02Y2A4;=02:03214;02?42H1625;DU6:3;>6=;>6C2<012<0D4{802?;9:0DL>452­®2U03692:032?;6E| �2<0D4{802£2�D=43D4=2:0320D?692?423T?;02:431;=4262H=;5;M67802?4=4<D050N;6929412A;0C2:6362KH42942:09962<6:=6329;D652426<49963?;34=614D=4269234?492?42<01HD;<6780262:63=;32?42M0D6923H36;9C169234KH432KH42U6[625;DU692=454A�D;<692A;{692;D9=656?692D69:30{;1;?6?49E� �2�D=43D4=C2=61IY12?4D01;D6?62JJJC2Y2H162aD;<6234?42?4<01:H=6?0349C2?426I36DN�D<;621HD?;65C2A0316?62:032;Da14369nwzynw\C2692KH6;92?;9:0D;I;5;M612692;DA03167Z492KH42=36A4N61D62�D=43D4=2412=41:023465E¯ °±²³²µ²¶·´°²¹°º·» »33

Page 35: Exercicios Informatica CESPE 2010 250 Questoes

��������������� ���������������������������� �!�����"���������#� ��$��%&'()*+,*)*+*-+.(/-01/-+2/+33+*+3456789:;<6<=8><6>?@A;<6:><6ABC<6D?6EFGHIJKLMN;O778=B6P;8AB6QRS=?TABTD?6:>6ABUA?6B>6V;?=B@@?6DB6BD8WX?RYZ[\] _ aab?T@8DB;<TD?6<6@8A:<WX?6>?@A;<D<6T<6789:;<c6<@@8T<CB6<6?VWX?=?;;BA<Rd e<;<6@B6=BTA;<C8f<;6?6AgA:C?6D?6ABUA?c6h6@:78=8BTAB6=C8=<;6@?i;BBCB6Bc6B>6@B9:8D<c6@BCB=8?T<;6<6?VWX?6jklmno6T?6=<>V?Rp e<;<6@B6=?>V<;A8Cq<;6?6<;r:8s?6=?>6:@:t;8?@6D<6;BDB6<6r:B6?=?>V:A<D?;6B@ABu<6=?TB=A<D?6h6@:78=8BTAB6=C8=<;6<7B;;<>BTA<6 6B6D898A<;6?@6vFwxyE6D<@6VB@@?<@6<:A?;8f<D<@Rz 5?6@B6V?@8=8?T<;6?6=:;@?;6C?9?6<V{@6<6V<C<s;<6|sB;B<D?;}6Bc6B>@B9:8D<c6V;B@@8?T<;6<6AB=C<6~c6?6=?;V?6D?6ABUA?c6>?@A;<D?6T<ABC<c6@B;t6;BD:f8D?6<6:>6�T8=?6V<;t9;<7?R� 5?6@B6><;=<;6<6V<C<s;<6|CB9BTD<@}6Bc6B>6@B9:8D<c6<VC8=<;6:>A;8VC?6=C8r:B6@?i;B6BC<c6?6=?;V?6D?6ABUA?6@B;t6@BCB=8?T<D?R6� e<;<6@B6@:iC8Tq<;6?6AgA:C?c6h6@:78=8BTAB6@BCB=8?Tt�C?6B6=C8=<;T<67B;;<>BTA<6 RYZ[\] _ a�b?>6;BC<WX?6�6789:;<6B6�6:A8C8f<WX?6D?6N;O778=B6P;8AB;6QRSc6<@@8T<CB<6?VWX?6=?;;BA<Rd 56789:;<6<V;B@BTA<6<6Vt98T<6�6DB6:>6ABUA?6r:B6=?TAh>6�SVt98T<@R6p 568TD8=<WX?6 6<V;B@BTA<D<6T?6=<TA?68T7B;8?;D8;B8A?6D<6ABC<6Bs8DBT=8<6r:B6:><6@BTq<6V<;<6<=B@@?6B@At6@BTD?;Br:8@8A<D<R6z 56?VWX?6 6T<6V<;AB68T7B;8?;6D<6ABC<68TD8=<6r:B6?6ABUA?67?88T@B;8D?6<6V<;A8;6DB6:><6Vt98T<6D<6�TAB;TBAR6� e<;<6@B6BUV?;A<;6?6ABUA?6V<;<6?67?;><A?6e��6h6@:78=8BTAB6=C8=<;@?i;B6<6?VWX?6 c6D898A<;6?6T?>B6D?6<;r:8s?6<V;?V;8<D?6B=C8=<;6<6?VWX?6�����nR6� e<;<6@B6C8>V<;6<67?;><A<WX?6D?6ABUA?6h6@:78=8BTAB6@BCB=8?Tt�C?B6=C8=<;6<67B;;<>BTA<6 R YZ[\] _ a�b?T@8DB;<TD?6<8TD<6<6789:;<6D?6N;O778=B6P;8AB;6QRSc6<@@8T<CB6<?VWX?6r:B68TD8=<6?6V;?=BD8>BTA?6=?;;BA?6V<;<6BUV?;A<;6?6ABUA?6V<;<?6P?;DRd bC8=<;6<6?VWX?6���onm�n��n����o�D?6�Ly�6 c@BCB=8?T<;6<6V<@A<6DB@Bu<D<6B6V?;678>6=C8=<;6��Rp bC8=<;6<6?VWX?6�on�6D?6�Ly�6 c6@BCB=8?T<;6<6V<@A<DB@Bu<D<6Bc6B>6@B9:8D<c6=C8=<;6��R6z 5?6@B6=C8=<;6@?i;B6<6?VWX?6 c6@B;t6<V;B@BTA<D<6<6?VWX?;B7B;BTAB6<?6A8V?6DB6<;r:8s?6V<;<6BUV?;A<WX?R65V{@6=C8=<;6?6A8V?DB6<;r:8s?6DB@Bu<D?c6h6@:78=8BTAB6@BCB=8?T<;6<6?VWX?6�on�6B=C8=<;6��R� bC8=<;6?6�Ly�6 6B6<;;<@A<;6?6ABUA?6V<;<6:><6u<TBC<6D?P?;Dc6TB=B@@8A<TD?6DB@@<67?;><6<i;8;6:><6T?s<6ABC<6D?N;O778=B6P;8AB;6QRSR6� bC8=<;6<6?VWX?6�����n�jo�o6D?6�Ly�6�n����oc6@BCB=8?T<;6<6?VWX?���no�o m��on��¡�kn�¢o��£����¤�B6=C8=<;6?6i?AX?6�����nR¥/¦0§+/+&'()*+,*)*+*-+.(/-01/-+3©+/+3ª56ABC<6<=8><67?869B;<D<6<6V<;A8;6D?6<VC8=<A8s?6N;O778=B6b<C=R6QRS6B<V;B@BTA<6:><6VC<T8Cq<6<6V<;A8;6D<6r:<C67?869B;<D?6?69;t78=?>?@A;<D?RYZ[\] _ a«b?T@8DB;<TD?6<6789:;<c6<@@8T<CB6<6?VWX?6=?;;BA<Rd O69;t78=?6>?@A;<D?67?86=;8<D?6=?>6i<@B6T<@6=hC:C<@6DB6�¬6<6­®Rp 567B;;<>BTA<6 68TD8=<6r:B6?69;t78=?6TX?6h6D8T>8=?Re?;A<TA?c6<?6@B6<CAB;<;6?@6s<C?;B@6D<6VC<T8Cq<6?69;t78=?6TX?6@B;t<CAB;<D?R6z e<;<6@B6<CAB;<;6<@6=?;B@6D?69;t78=?c6h6@:78=8BTAB6@BCB=8?Tt�C?6B=C8=<;6@?i;B6<@6=?;B@6DB@Bu<D<@6T?6=<>V?6 R� e<;<6@B6=BTA;<C8f<;6?6ABUA?6T<6=hC:C<c6h6@:78=8BTAB6=C8=t�C<6B<u:@A<;6<6?VWX?6 R� 5TAB@6DB6=;8<;6?69;t78=?c67?86TB=B@@t;8?6DB78T8;6<6B@=<C<6<6@B;:A8C8f<D<6V<;<6?6B8U?6°R±±±²³µ¶· ¹º¶º²¶»²¼¹½ ½34

Page 36: Exercicios Informatica CESPE 2010 250 Questoes

��������������� ���������������������������� �!�����"���������#� ��$��%&'()*+,-./0123456714089:58028474028178;<713<=78478>3?:678@AB<:<728A0C47402D8E3F5225C823408?567402878;76E368408GHA5<8IJJKL8722317<5870;MN089:587;65251E7L8652;5AE3F7C51E5L8728>O6C:<728A0665E728;767825A7<A:<76808E0E7<85878CB437840285<53E06528108GHA5<8IJJKPQ RSTUVWXYZ[\]_S58RSabcWYZ[d]_e RfU WY]gd]_S58RSabcWY]gd]_h RTUVWXY]gd]_S58RSabcWSY]g\]_i RfU WYZ[jS]_S58RSabcWYZ[k]_l RfU WYZ[d]_S58RSabcWYZ[d]_%&'()*+,-mn22317<58780;MN089:587;65251E78:C8;60E0A0<08652;012oF5<8;5<051F308458C5127?51285<5E6p13A728178q1E5615EP8Q rste tutvh wxyti wyztl {zt%&'()*+,-|/012345671408028232E5C7280;567A3017328}3140~28�t858�31:HL722317<58780;MN08A0665E7PQ �10C58B808232E5C78?5651A374068458:2:o6308408�31:HP8e n80;MN08�j�S�U ��VWbU�S108}3140~28�t87;65251E7872A767AE56�2E3A728408:2:o63087E:7<Ph x08�31:HL8;76782587A52276878q1E5615E8B82:>3A351E5851E676810}3140~28GH;<0656Pi u8�Wc�jXSbjS�U�V�UXjS408�31:H8;0223�3<3E7878A637MN08458769:3F0258;72E72Pl x7:E3<:28B8:C8;60?67C7825C5<=71E58708}3140~28GH;<065689:5;56C3E58?5651A3768769:3F02P8%&'()*+,-�/012345671408028A01A53E028�o23A028458E5A10<0?372858>5667C51E727220A374728�8q1E5615E858��������L8722317<58780;MN08A0665E7PQ t76782587A52276878q1E5615E80:8:C78��������L8B82:>3A351E589:580:2:o6308E51=7808q1E5615E8GH;<06568312E7<74085C825:8A0C;:E7406Pe n8E5A10<0?378v�8432;013�3<3�78256F3M028458E5<5>013785E6712C322N0845847402878<01?728432E�1A372L85C8:C87C�351E5COF5<L831A<:31408087A5220878q1E5615EP8h u8u:E00�8GH;65228;022:38C7328>:1A3017<34745284089:580y3A6020>E8u:E<00�L8A0C0L8;0685H5C;<0L8Z�j�bW858�U�VWVU�Pi n8��������8432;013�3<3�78256F3M02825C5<=71E5287028478q1E5615E451E608458:C7865458<0A7<L8C7281N08;56C3E589:58522528256F3M0225�7C87A522740284580:E6028<0A732P8l nsw�8B8:C8256F3M083C;<5C51E7408;5<08q1E5615E8GH;<065689:5;56C3E587:C51E76878F5<0A3474584587A5220878q1E5615EP8 %&'()*+,-�/012345671408A01A53E02845825?:671M7847831>06C7MN0L8722317<5870;MN08A0665E7P8Q n825?:671M78472831>06C7M�5289:58E67123E7C8;5<78q1E5615E8B845E0E7<8652;0127�3<34745840874C3132E6740684586545Pe q12E7<76858:E3<3�76871E3F�6:285C8:C8A0C;:E74068B8:C787MN0;65F51E3F789:585<3C3178A0C;<5E7C51E5878;0223�3<347458457E79:52878769:3F02858;72E72P8h n08258:E3<3�768��������8B8?7671E340808�<09:5308458F�6:2858��� ¡L;0328782:78;631A3;7<8>:1MN08B83451E3>3A768585<3C31768769:3F02A0660C;3402P8i {5A:6202858312E7<7M�528458;60A5227C51E0845831>06C7M�52A6�E3A7280:82512�F532840815?OA30845F5C82568C71E347285C8o657225?:672L8;60E5?34728;068:C8;56�C5E60845825?:671M7845>31340LA0C8�76653672845825?:671M787;60;637472858A01E60<584587A5220Pl u28232E5C7280;567A3017328C04561028;022:5C8C5A7132C0289:55F3E7C878;60;7?7MN08458F�6:2858A7F7<028458E6037P8z732C5A7132C02845F5C825687E3F74028;068C5308408?5651A37406845769:3F0280:8;5<08?5651A3740684587;<3A7E3F02P8%&'()*+,¢£/0123456714080287;<3A7E3F028408y3A6020>E8u>>3A5L8722317<58780;MN0A0665E7P8Q n8452F71E7?5C8458258:E3<3�76808yw8}0648;767878543MN0845E7�5<728B8783C;0223�3<347458458A63768>O6C:<728;7678E0E7<3�76F7<0652P8e n08258A63768:C787;65251E7MN08108yw8t0~568t031EL8B8;022�F5<31256368E5HE028408yw8}06480:8478q1E5615E8587314783125636;<713<=78408yw8GHA5<8�5C8A0C083C7?512858F�4502845843F56202E3;02P8h x08yw8GHA5<8IJJKL878A637MN08458C7A6028B8;022�F5<8A0C87312E7<7MN08408¤�¥¦��§W��U W�jPi n08258A0;3768:C8652:<E7408458:C78>O6C:<78A637478108yw8GHA5<58A0<o©<085C8:C865<7EO6308A637408108yw8}064L89:71407<E5674028028474028108yw8GHA5<L808F7<0687;65251E7408108yw}0648256o87<E5674087:E0C7E3A7C51E5Pl t7678258A63768:C806?710?67C78108yw8}064L8B815A522o630312E7<76808y3A6020>E8u6?71325Pªªª«¬­®­°­±²³³«­´«µ²¶ ¶35

Page 37: Exercicios Informatica CESPE 2010 250 Questoes

UnB/CESPE – Banco do Brasil S.A. Caderno ACargo: Escriturário – 6 –

CONHECIMENTOS ESPECÍFICOSCentral de Segurança do Windows

Ajude a proteger o seu PC

A Microsoft tem um compromisso com a sua privacidade. Leia nossa.

Atualizações automáticas VERIFIQUE AS CONFIGURAÇÕES

NÃO MONITORADOProteção contra vírus

Gerenciar configurações de segurança para:Opções da Internet Firewall do Windows

Atualizações Automáticas

DESATIVADO

Dados Básicos de SegurançaRecursos A Central de Segurança ajuda a gerenciar as configurações de segurança do Windows. Para ajudar a proteger seucomputador, verifique-se de que os três elementos básicos de segurança estão ATIVADOS. Se as configurações desegurança não estiverem ATIVADAS, siga as recomendações. Para voltar à Central de Segurança mais tarde, abra o painelde Controle.O que há de novo no Windows para ajudar a proteger meu computador?

Obter as iniformações maisrecentes sobre segurança e vírusda Microsoft

Procurar as atualizações maisrecentes no Windows Update

Obter ajuda sobre a Central deSegurançaAlterar o modo de alerta daCentral de Segurança

Obter suporte para problemasrelativos a segurança

As Atualizações Automáticas estão configuradas para somente instalar atualizações após sua confirmação. Clique emAtivar Atualizações Automáticas para que o Windows automaticamente mantenha seu computador em dia com asatualizações mais importantes (recomendado). Como as Atualizações Automáticas ajudam a proteger meu computador?

Você nos informou que está usando um programa antivírus que você mesmo monitorará. Para ajudar a proteger seucomputador contra ameaças de segurança, certifique-se de que seu programa antivírus está ativado e atualizado. Comoprogramas antivírus ajudam a proteger meu computador?

Obs.: O Windows não detecta todos os firewalls.Recomendações...

R comendações...e

A ivar Atualizações Automáticast

Considerando a figura acima, que apresenta a janela da Central deSegurança sendo executada em um computador com o sistemaoperacional Windows XP, julgue os itens seguintes, a respeito deconceitos de proteção e segurança.�� O computador em uso não possui firewall instalado, as

atualizações do sistema operacional Windows não estão sendobaixadas do sítio da Microsoft e não existe antivírus emfuncionamento.

�� O firewall do Windows é um tipo de aplicativo funcionalmentesimilar ao iptables do Linux, no que concerne ao monitoramentoda execução de aplicativos no computador e à filtragem depacotes na saída dos adaptadores de rede do computador.

Cada um dos itens de 73 a 75 contém associações relativas a conceitosde informática, hardware, software e sistemas operacionais Windowse Linux usados no mercado. Julgue se essas associações estão corretascom base na tecnologia atual.

��

tecnologia empregadaem barramento de

computadoresconceito relativoa computadores

SATA disco rígidoAGP placa de expansãoIDE disco rígidoSCSI disco rígidoPCI placa de expansãoUSB periféricoDDR memóriafirewire periférico

��

dispositivocomputacional medida associada

clock de processador 2 giga-hertzmemória secundária 4 giga-bytesmemória primária 500 giga-bytesregistrador 64 bitsbarramento USB 500 mega-hertzbarramento de memória 500 megabits/segundofast ethernet 1 gigabit/segundo

��

comando sistema operacionalls Linuxcd Windows e Linuxps Linuxchmod Linuxchown Linuxcopy Windows e Linuxset Windows e Linuxdir Windows e Linuxattrib Windows

Considerando que a planilha eletrônica mostrada na figuraacima esteja sendo editada no programa Excel, julgue ositens que se seguem.�� Se as células C5, C6, C7 e C8 contiverem as instruções

a seguir, então a soma do conteúdo das células C5, C6,C7 e C8 será igual a 132.em C5: =SOMA(C2:C4)/3em C6: =MÉDIA(C2:C4)em C7: =SOMASE(C2:C4;"<50")/3em C8: =SE(C7=C6;SE(C5=C6;C6;SOMA(C2:C7)/6)

SOMA(C2:C7)/6)�� Se a sequência de operações a seguir for realizada na

planilha mostrada, nesse caso, a soma do conteúdo dascélulas D2, D3 e D4 será igual a 99.• atribuir o rótulo CLIENTE à célula B2;• atribuir a instrução =SE(B2=CLIENTE;C2;0) à

célula D2;• copiar, por meio da operação de Copiar e

Colar padronizada, o valor de D2 para as células D3e D4.

��������

36

Page 38: Exercicios Informatica CESPE 2010 250 Questoes

UnB/CESPE – Banco do Brasil S.A. Caderno ACargo: Escriturário – 7 –

A figura acima apresenta a configuração de um sistemacomputacional em determinado instante de tempo.No diretório c:\dir\a\aa, encontram-se os seguintes arquivos:a1.mp4, a2.mp3, a3.jpg, a4.gif, a5.png, a6.wav, a7.wmv e a8.xls.Nessa situação, julgue os próximos itens.�� Os arquivos contidos no diretório c:\dir\a\aa são compatíveis com

os seguintes usos, respectivamente por ordem de citação:áudio/vídeo digital; áudio digital; fotografia digital; figura gráfica;figura gráfica; áudio digital; áudio/vídeo digital; e planilha decálculo.

� Considere que o usuário tenha utilizado o console para digitar umcomando, conforme apresentado na figura a seguir. Nessasituação, se o comando apresentado nessa figura for efetuado nomomento em que o arquivo a8.xls estiver aberto, essa açãoresultará em uma mensagem de erro e nenhum dos arquivoscontidos no diretório de origem será movido para o diretório dedestino.

Julgue os itens de 80 a 85, relativos a conceitos de aplicativos deinformática utilizados em ambiente de escritório.� Microsoft Word e OpenOffice Writer são processadores de texto

do tipo WYSIWYG (what you see is what you get), e ambos sãocapazes de associar metadados aos documentos editados por meiodeles, tais como título, assunto, palavras-chave e comentáriosacerca do documento, entre outros.

�� Entre as funções providas pelos programas Microsoft PowerPointe BrOffice Impress, destacam-se a provisão de assistentes paraelaboração de apresentações em estilos padronizados, aexportação para formatos HTML, PDF e SVG e a inserção deobjetos de vídeo digital.

�� Ao estabelecer conexão com servidores web, os navegadores, taiscomo o Microsoft Internet Explorer e o Mozilla Firefox,empregam com grande frequência o protocolo HTTP. Nesseprotocolo, um pedido é enviado do navegador para o servidor e,em seguida, uma resposta é enviada do servidor para o cliente.Essa resposta, usualmente, contém no seu corpo um documentodigital.

�� Entre os programas para correio eletrônico atualmenteem uso, destacam-se aqueles embasados em softwareclientes, como o Microsoft Outlook e o MozillaThunderbird, além dos serviços de webmail. O usuáriode um software cliente precisa conhecer um maiornúmero de detalhes técnicos acerca da configuração desoftware, em comparação aos usuários de webmail.Esses detalhes são especialmente relativos aosservidores de entrada e saída de e-mails. POP3 é o tipode servidor de entrada de e-mails mais comum e possuicomo característica o uso frequente da porta 25 para oprovimento de serviços sem segurança criptográfica.SMTP é o tipo de servidor de saída de e-mails de usomais comum e possui como característica o usofrequente da porta 110 para provimento de serviçoscom segurança criptográfica.

�� São exemplos de tecnologias empregadas nasplataformas Windows e Linux para estabelecimento deconexões remotas com computadores: Telnet, SSH,VNC e WTS (Windows Terminal Services), entreoutras. A tecnologia Telnet é suportada nas plataformasWindows e Linux, enquanto a tecnologia WTS não ésuportada na plataforma Linux. Já a SSH não é umaferramenta nativa para a plataforma Windows.

�� Comparativamente à educação presencial, asabordagens de educação a distância exigem do aprendizmais autodisciplina, motivação e esforço para o preparodos materiais educacionais. Porém, ambas asabordagens podem ser realizadas sem uso decomputador, são compatíveis com a aprendizagemcontinuada e nelas se podem adotar modelos deconversação e tutorial, entre outros.

Julgue os itens a seguir, acerca do atendimento prioritárioobrigatório nas instituições financeiras.�� O atendimento prioritário garantido por lei estende-se,

entre outros, aos maiores de sessenta anos de idade, àsgestantes, aos portadores de necessidades especiais, àspessoas com criança de colo e aos turistas estrangeirosoriundos de países de língua diversa da línguaportuguesa.

�� O atendimento prioritário regulamentado por leicompreende tratamento diferenciado e atendimentoimediato.

�� A permanência de cão-guia no interior de agênciabancária pode ser licitamente impedida por funcionárioresponsável, mesmo diante da apresentação da carteirade vacinação atualizada do animal.

37

marcusbelfort
Rectangle
Page 39: Exercicios Informatica CESPE 2010 250 Questoes

UnB/CESPE – ANATEL

Conhecimentos Básicos para os cargos de nível superior de 1 a 6 e de 11 a 15 – 4 –

A figura acima ilustra uma planilha em elaboração em uma janela do Microsoft Office Excel 2003, relacionando os números de

chamadas recebidas e realizadas por uma empresa fictícia, nos dez primeiros dias do ano de 2004, até onze horas. A respeito dessa

planilha e do Excel 2003, julgue os itens subsequentes.

21 O Excel 2003 permite o modo de exibição de diversas barras de ferramentas que contêm diferentes campos e

botões. Na janela acima, os campos ou botões ; ;

e fazem parte, respectivamente, das barras de

formatação, de fórmulas e padrão.

22 Só é possível encontrar o valor total das chamadas recebidas e realizadas na data/hora, 05/01/2004 11:00” mediante a realização

da seguinte sequência de ações: selecionar a célula H6; digitar = B6 + G6; teclar �.

Considerando a figura acima, que ilustra uma janela do Microsoft Outlook 2003 sendo executada, julgue os itens seguintes.

23 Se uma pessoa quiser enviar mensagem a destinatário específico com cópia para outros destinatários, deve considerar que o

Microsoft Outlook não permite que a mensagem seja enviada simultaneamente a terceiros sem que o destinatário tenha

conhecimento. Em contrapartida, o programa permite a inclusão de diversos endereços de e-mail no campo , que

receberão uma cópia da mesma mensagem com conhecimento do destinatário.

24 Caso um remetente, usando o Microsoft Outlook, queira enviar uma mensagem a um contato salvo em lista criada por ele, basta

clicar no ícone . A ferramenta encontrará o endereço eletrônico do contato a partir da digitação de seu nome.

www.pciconcursos.com.br

38

Page 40: Exercicios Informatica CESPE 2010 250 Questoes

UnB/CESPE – ANATEL

Conhecimentos Básicos para os cargos de nível superior de 1 a 6 e de 11 a 15 – 5 –

Sistema Linux já é coisa de gente grande: crescea adoção do software nas empresas brasileiras

O Linux, principal concorrente do MicrosoftWindows, já serve de base a um mercado bilionário no paíse dá suporte a atividades essenciais de gigantes nacionais.

O uso do Linux é tranquilo, estável e confiável.Além disso, permite reduções de 30% a 40% nosinvestimentos em equipamentos. Os terminais não têm discorígido e carregam os programas diretamente dos servidores.Com essa configuração rodando Linux, as redes varejistaspodem usar computadores bem mais simples e baratoscomo terminais, reduzindo os custos das máquinas e de suamanutenção.

O Estado de S. Paulo, 13/4/2004 (com adaptações).

Tendo o texto acima como referência inicial, julgue os itensseguintes, a respeito do sistema operacional Linux.

25 O sistema operacional Linux é considerado um softwarelivre, o que significa que não é propriedade exclusiva denenhuma empresa e que a seu usuário é assegurada aliberdade de usá-lo, manipulá-lo e redistribuí-loilimitadamente e sem restrições.

26 A redução de gastos com investimentos emequipamentos, a que se refere o texto, pode serrelacionada ao fato de o Linux ser um kernel modular, oque significa que as suas funções de agendamento deprocessos, gerenciamento de memória, operações deentrada e saída, acesso ao sistema de arquivos entreoutras, são executadas no espaço kernel.

Com referência ao funcionamento da Internet e das intranets,julgue os itens a seguir.

27 O funcionamento da Internet depende de três camadasde protocolos base: o protocolo de Internet IP, definidorde datagramas que carregam dados de um nó a outro darede; os protocolos TCP, UDP e ICMP, responsáveispela transmissão de dados; e, na camada final, osprotocolos definidores de mensagens específicas e deformatos digitais, como os DNS, POP3 e HTTP, entreoutros.

28 Baseada nos padrões de comunicação da Internet, umaintranet pode ser caracterizada como uma rede privadade computadores, acessível apenas a membros de umamesma organização. Mesmo assim, sua utilização requercomponentes básicos, como sistemas de proteção eservidores web, sem, no entanto, ser obrigatório o uso doprotocolo TCP/IP.

Acerca da segurança da informação, julgue os itens a seguir.

29 Segurança da Informação é a proteção contra um grandenúmero de ameaças às informações, de forma aassegurar a continuidade dos negócios, minimizandodanos comerciais e maximizando o retorno deinvestimentos. Ela pode ser garantida fazendo-se uso decontroles físicos da informação, de mecanismos decontrole de acesso, como firewalls e proxies, entreoutras medidas.

30 Uma organização, ao estabelecer seus requisitos desegurança da informação, deve avaliar riscos, a partir davulnerabilidade e da probabilidade de ocorrência deeventos de ameaça, sempre obtidas por meio de dadoshistóricos de incidentes e problemas registrados nosbancos de dados da central de serviços.

Julgue o item abaixo, acerca dos crimes contra a administração

pública.

31 Governador de estado que, após responder por ação de

improbidade administrativa perante o Poder Judiciário, for

condenado nas penas de suspensão dos direitos políticos e

indisponibilidade dos bens não responderá por crime no âmbito

penal.

Julgue os itens subsequentes, relativos a empresas públicas e

sociedades de economia mista.

32 Empresas públicas são pessoas jurídicas de direito privado

criadas mediante autorização legal, integrantes da administração

indireta do Estado.

33 Criadas mediante autorização legal sob a forma de sociedade

anônima, as sociedades de economia mista integram a

administração indireta do Estado.

Com referência ao processo administrativo na administração pública

federal, julgue o item que se segue.

34 Não cabe recurso das decisões administrativas proferidas pelos

servidores das agências reguladoras, conforme preceitua a Lei n.º

9.784/1999, que regula o processo administrativo no âmbito da

administração pública federal.

Julgue o item seguinte, referente aos direitos e deveres individuais e

coletivos.

35 Qualquer cidadão poderá impetrar habeas data no Poder

Judiciário para assegurar o conhecimento de informações

relativas a sua pessoa disponíveis na Agência Nacional de

Telecomunicações (ANATEL), independentemente de ter

formulado o pedido diretamente na agência.

Acerca da Lei Geral de Telecomunicações (Lei n.º 9.472/1997),

julgue o próximo item.

36 O peticionamento pelos usuários de serviços de

telecomunicações para reclamarem seus direitos contra

prestadores de serviço perante o órgão regulador não dependerá

da assistência de advogado.

Julgue o item seguinte, acerca dos bens públicos.

37 Considerando que um governador de estado prometa a

construção de uma praça para atividades esportivas para toda a

comunidade de seu estado, é correto afirmar que essa praça, tão

logo seja construída, será classificada no direito administrativo

brasileiro como bem de uso especial.

Acerca da responsabilidade civil do Estado, julgue o item que se

segue.

38 A responsabilidade civil do Estado poderá ser afastada se

comprovada a culpa exclusiva da vítima, ou mitigada a reparação

na hipótese de concorrência de culpa.

www.pciconcursos.com.br

39

marcusbelfort
Rectangle
Page 41: Exercicios Informatica CESPE 2010 250 Questoes

UnB/CESPE – TRE/GO Caderno DCargo 3: Analista Judiciário – Área: Apoio Especializado – Especialidade: Arquitetura – 4 –

��������

Julgue as seguintes propostas de continuidade para o texto.I Por isso, a hostilidade às formas de linguagem de grupos minoritários

constitui uma forma de preconceito. II Assim, qualquer “verdade” reprimida representa uma armadilha de

preconceitos e censuras.III Portanto, são verdades falsas porque são invisíveis. Há continuidade gramaticalmente correta e argumentativamente coerentepara o texto apenas A no item I.B no item III.C nos itens I e II. D nos itens II e III.Texto para as questões de 16 a 18Considere a configuração de um microcomputador do tipo notebookapresentada a seguir.

Intel Core Duo – 1,6 GHz e 533 MHz FSB;2 MB L2 cache;15,4 WXGA LCD;120 GB HDD; DVD-RW double layer;1 GB DDR2, 802.11 a/b/g wireless LAN.

���������

Com base na configuração apresentada, assinale a opção correta. A 1,6 GHz indica que o processador possui capacidade de

armazenamento de dados de 1.600 MB.B Intel Core Duo indica a existência de memória dupla, o que amplia

a velocidade de processamento das informações. C 533 MHz FSB indica a capacidade da memória RAM.D Intel Core Duo indica que há dois processadores no mesmo chip de

silício.���������

Com referência ao microcomputador apresentado, assinale a opçãocorreta.A 2 MB L2 cache indica a capacidade da memória cache, uma

memória estática que tem por finalidade aumentar o desempenho doprocessador ao realizar busca antecipada na memória RAM.

B A opção L2 significa que é possível instalar dois sistemasoperacionais, como o Linux e o Windows XP.

C A utilização de dispositivos do tipo pen drive permite ampliar, aindaque temporariamente, a capacidade da memória ROM docomputador.

D A capacidade do disco rígido, na referida configuração, é de 1 GB.���������

Ainda com base na configuração apresentada, assinale a opção correta.A DVD-RW double layer indica que o notebook possui leitora de

DVD dupla face, mas não permite gravação.B Com a configuração de hardware apresentada, não seria possível a

instalação do software Linux. Para essa instalação, seria necessárioampliar a capacidade de memória.

C O notebook com a configuração apresentada permite acesso a redessem fio das tecnologias 802.11 a/b/g wireless LAN.

D 15,4 WXGA LCD indica o modelo e o tamanho da placa-mãe daconfiguração apresentada.

Figura para as questões de 19 a 22

A figura a seguir mostra uma janela do software Writer,com um documento em processo de edição.

���������

Com base na figura apresentada, assinale a opção correta.

A O símbolo , no canto esquerdo do documentoem edição, indica que a imagem mostrada nodocumento é um hiperlink.

B Para mover a imagem mostrada no documento, ésuficiente clicá-la no centro e arrastá-la para aposição desejada.

C Para alterar as cores da imagem mostrada no

documento, é suficiente clicar a ferramenta e, na janela disponibilizada, escolher as cores.

D Para alterar a posição da imagem no documento, ésuficiente apontar o cursor para a borda da imagematé que ele se transforme em uma seta de dupla pontae arrastar a imagem para a posição desejada.

���������

Com relação à figura mostrada, assinale a opção correta.

A Considerando que o documento em edição tenha umtítulo com a fonte em letras minúsculas, para alterá-lopara letras maiúsculas é suficiente selecioná-lo; clicara opção Alterar Capitalização do menu ; e,posteriormente, na janela disponibilizada, clicarMaiúscula.

B Para criar afastamento à direita da imagem emrelação ao texto, é suficiente pressionar a tecla

.

C Ao pressionar a tecla , o texto será excluído.D Para formatar o texto com espaçamento duplo, é

suficiente clicar a ferramenta .

www.pciconcursos.com.br www.pciconcursos.com.br

40

marcusbelfort
Rectangle
Page 42: Exercicios Informatica CESPE 2010 250 Questoes

UnB/CESPE – TRE/GO Caderno DCargo 3: Analista Judiciário – Área: Apoio Especializado – Especialidade: Arquitetura – 5 –

��������

Com referência à figura apresentada, assinale a opção correta.A O estilo do texto no documento em edição é padrão. B O texto está formatado com fonte Arial e está centralizado.C A auto verificação ortográfica está ativada. D O documento em edição possui duas páginas.

��������

Considerando que o Microsoft Outlook seja o cliente de e-mailpadrão, assinale a opção correspondente à sequência correta deprocedimentos para o envio do documento em edição comoanexo de um e-mail.

A clicar a opção Envelope do menu , digitar oendereço eletrônico do destinatário no campo Para, clicarEnviar

B clicar a ferramenta , digitar o endereço eletrônico dodestinatário no campo Para, clicar Enviar

C clicar a opção Exportar do menu , digitar oendereço eletrônico do destinatário no campo Para, clicarEnviar

D clicar a ferramenta , abrir o Microsoft Outlook, digitaro endereço eletrônico do destinatário no campo Para, clicarEnviar

Figura para as questões de 23 a 26A figura a seguir ilustra uma janela do aplicativo Calc, com umaplanilha em processo de edição.

���������

Com relação à figura apresentada, assinale a opção correta.A Na situação da janela mostrada, para centralizar o título —

Gastos com Material — na planilha, é suficiente clicar o

botão .B Para centralizar os conteúdos das células A2, B2, C2 e D2, é

suficiente selecioná-las e clicar a ferramenta .C Para mesclar as células A1, B1, C1 e D1, é suficiente

selecioná-las e clicar o botão .D Na situação da janela mostrada, para mesclar e centralizar o

título da planilha — Gastos com Material —, é suficiente

clicar o botão .

���������

Com base na figura apresentada, assinale a opção correta.

A Para inserir bordas duplas na planilha, a sequência correta deprocedimentos é: selecionar a planilha; clicar a opção Célulasdo menu ; clicar a guia Borda, selecionar estiloduplo; clicar OK.

B Para alterar o tipo de letra da planilha, é suficiente selecioná-la, clicar a opção Fonte do menu , selecionar otipo de fonte desejado, clicar OK.

C Para autoformatar a planilha, é suficiente selecioná-la, clicar

a ferramenta , na janela disponibilizada, selecionaro modelo desejado, clicar OK.

D Para formatar os valores da coluna Valor Unitário como

moeda, é suficiente selecioná-los e clicar o botão .��������

Ainda com relação à figura apresentada, assinale a opção correta.

A Para calcular o valor total a ser gasto com a aquisição dosprodutos, é suficiente clicar a célula C7, clicar a ferramenta

e pressionar a tecla .B Para calcular o valor Total para o produto Micro, é suficiente

clicar a célula D5, digitar a fórmula =B5+C5 e pressionar atecla .

C Para calcular o valor Total para o produto Mesa é suficienteclicar a célula D3, digitar a fórmula =B3*C3 e pressionar a

tecla .D Para calcular o valor Total Geral, é suficiente calcular o

somatório da quantidade na célula B7 e multiplicar peloresultado do cálculo do somatório dos valores unitários.

���������

Considerando o gráfico apresentado na figura, assinale a opçãocorreta.

A Os valores no eixo horizontal apresentam casas decimaisporque ele foi construído com base na coluna D.

B Para alterar o tamanho do gráfico, é suficiente selecioná-lo,clicar a opção Tamanho do menu , selecionar,na janela disponibilizada, o tamanho desejado e clicar OK.

C Para dimensionar o gráfico com tamanho 50% da página, ésuficiente clicá-lo e selecionar 50 na ferramenta

.D O gráfico foi construído com base apenas nos valores das

colunas A e B.

www.pciconcursos.com.br www.pciconcursos.com.br

41

Page 43: Exercicios Informatica CESPE 2010 250 Questoes

UnB/CESPE – TRE/GO Caderno DCargo 3: Analista Judiciário – Área: Apoio Especializado – Especialidade: Arquitetura – 6 –

Figura para as questões 27 e 28

A figura a seguir ilustra uma janela do Internet Explorer 6.0 (IE6), que está em uso para acesso a uma página web.

���������

Considerando a pesquisa na Internet realizada, assinale a opçãocorreta.A Se a pesquisa fosse realizada com o termo tre-go colocado

entre aspas, então o último dos resultados mostrados na figuranão seria apresentado.

B A pesquisa apresenta o resultado de 30 sítios encontrados como tema tre-go.

C A partir da figura, é correto afirmar que a notícia “TRE-GOcassa dois vereadores de Anápolis”, encontra-se no sítio doTRE-GO.

D O terceiro resultado mostrado na figura apresenta a traduçãoda página oficial do TRE-GO para o idioma inglês.

���������

Considerando que o Microsoft Outlook seja o cliente padrão,assinale a opção correspondente à sequência correta deprocedimentos para encaminhar, por e-mail, o resultado dapesquisa.

A clicar a opção e-mail do menu , digitar, no campoPara, o endereço eletrônico do destinatário, clicar Enviar

B clicar a ferramenta , selecionar a opção Enviar Página,digitar, no campo Para, o endereço eletrônico do destinatário,clicar Enviar

C clicar a ferramenta , selecionar a opção Enviar Página,digitar, no campo Para, o endereço eletrônico do destinatário,clicar Enviar

D clicar o menu , selecionar a opção e- mail,digitar, no campo Para, o endereço eletrônico do destinatário,clicar Enviar

���������

Acerca do Internet Explorer e do sistema operacional Linux,assinale a opção correta.

A Para conectar à Internet um microcomputador que tenhainstalado o sistema operacional Linux, é necessária autilização de uma placa de rede específica.

B A conexão, à Internet, de um microcomputador que possuio sistema operacional Linux instalado é mais lenta quandocomparada com um que tenha instalado o Windows XP.

C Se um e-mail for criado a partir de algum aplicativo dosistema operacional Linux, ele não poderá ser lido pordestinatário que usa o Windows XP.

D Com o Linux é possível acessar a Internet usando uma redesem fio (wireless).

����������

Assinale a opção correspondente a características do sistemaoperacional Linux.

A multitarefa, multiusuário, open sourceB monotarefa, multiusuário, open sourceC multitarefa, monousuário, gratuitoD monotarefa, monousuário, gratuito

www.pciconcursos.com.br www.pciconcursos.com.br

42

Page 44: Exercicios Informatica CESPE 2010 250 Questoes

09.09.10 GABARITO  DAS  PROVAS  CESPE/2010 Prof.  Belfort

1 2 3 4 5 6 7 8 9 10 11 12 13 14 15 16 17 18 19 20 21 22 23 24 25 26 27 28 29 30 31 32 33 34 35 36 37 38 39 40 41 427 C8 C9 E X10 X C11 A B E E A X12 D D E D D13 D A B C E C E14 B B D C C A15 C D A E E C B16 E E C D D17 C C C E A18 E E C B C19 E C E D B20 C E E X B A21 C E E C C C22 E C C C E B23 E C E E X24 C E E E A25 E E E C26 C E C E D27 C C C E A28 E C E E C29 E C C E D30 C E E E E A31 C C E E E32 E C C C C33 C C C E C C E34 C E E C E C C35 E E C C E C36 E E E E E C C37 E E E E C C C38 E E C E C E E39 E E C E C E40 C C C C E E41 C C E E42 X C C C43 E E E44 X E C45 E C E46 E E E47 E X E48 C E C49 E C E50 E D E51 E A52 A B53 B D54 D E55 B A56 C D57 D X58 E E59 C E60 A D71 E72 E73 C74 E75 E76 X77 E78 C79 E80 C81 E82 C83 E84 C85 C

PÁGINAS

QUESTÕES

Page 45: Exercicios Informatica CESPE 2010 250 Questoes

!"#$$#$"%&'

()*+,-*%-+."$"/0-*12

-*+%'&+)**0

31'

%456#+789654:

;<=>?4@A

?48*5BA

?48&8@8C

DE:84E8:*8=F4?EG?

H54@+"!

-IJ89+"!%%5KE:+"!

H54@+"L

-IJ89+"L%%5KE:+"L

H4K:84

,?9JDM

;48CCHKE+N%

HKE+L

D-OD-P

'-

'F:955Q

RKEFI$

!S+T"!O+!P

!L!!U!O++!P

!L.

!!+!T+!V!S+T"

!L+!P!T+!O

!!+!OU!P!

$.$.

$$T

$T$!

$VV

$.$$+$.

O$!+$V

$TL

V$P

VTVT

V.V!

SVO

VVVL

VL$"

VPVS+O"

$$TPUV"

TTUTPTPUV"

T$UT!$.

!.+!TU!P!!

!!!"+!.

!T+!OU!P+!$

$!!$+!T

!T.O+.S+!"

.L+.P+!".O+.S

.P$T

V"V"

V$V$

V.V!

$VVT+VV+VO

VLVL

$OVS+O"

VP$L

L$"+$$

PS

$P$T

$V$!

$!$V

$ST"+T$

!O+!LT.+T!

!P+!S."

TSTTUTO

TL+TP.$

$P+.$$O

$V$L

.$..

..U.T.!

!$U!V!OUT"

T$T.

.T.T

.$U.T.VU.L

.!.V

.S!"

.P.P+!"

.O$!+$T

$V.L

$O+.$$L+$P+..

$S."

.P.T+.V+.S

.OU.P."+.$

...!

!".S

!$+!!+!T!$U!T

!"!V+!LUT"

!L!P

!O!$

!$+!.+!T!!++TS

TT+TOUTPTV+TS+V"

T"!S+T$

T.T!

!VU!P!P

!.$S

$P$O

$L$V

$$+$.$S+."

$!+$T!!

$$+$!S

S$"

$"$.

!T$!

$$U$!$T

!V$O+$P

$S."

."$V

$L$L

!OL!+LT

L$+L.LO+LL

L$+L.+LVLV

!LP.+PV

P.+P!P"

P$P"

P$LP+LS+PT

PT!P

.$+...!+.T

!S.L+.P

.S+!".V+.O

T"$O+$L+$P

$S+."T$

.$.!U.O

..T.

.S.L+.P

.P.S+!"

15:?9$.

T.

O$!!

."$O

T$

""

.LL

O!O

".

.."

T.P

WV

.$

.V$!

PL

."

""

$$!

.$V

"$

S"

.$$